N4301 ADULTS EXAM 2

Réussis tes devoirs et examens dès maintenant avec Quizwiz!

Cook beef to ____ degrees?

160

Which of the following drugs are used for the temporary treatment of hyperkalemia? A. Sodium polystyrene sulfonate (Kayexalate) B. Calcium acetate (PhosLo) C. IV glucose and insulin D. Sevelamer (Renagel) E. Sodium bicarbonate

A, C, E

A nurse at the beginning of a shift is assessing a client who has Cushing's disease. Which of the following is the priority assessment? A. Daily weights B. Fatigue C. Fragile skin D. Joint pain

A. Daily weights Fluid retention can lead to hypertension and heart failure

Which is the most likely cause of foodborne illness (food poisoning)? A. Prepackaged cookie dough left at room temperature B. Contaminated shellfish C. Frozen ice cream D. Unpasteurized milk

A. Prepackaged cookie dough left at room temperature

A nurse is caring for a client who has diabetes insipidus. Which of the following urinalysis laboratory findings should the nurse anticipate? A. Absence of glucose B. Decreased specific gravity C. Presence of ketones D. Presence of red blood cells

B. Decreased specific gravity The urine of a client who has diabetes insipidus will be dilute with a urine specific gravity of less than 1.005.

Which food item should a patient with celiac disease avoid? A. Steamed rice B. Yogurt C. Pancakes D. Raw pineapple

C. Pancakes

When caring for a patient during the oliguric phase of acute kidney injury, what would be an appropriate nursing intervention? A. Weigh patient three times weekly B. Increase dietary sodium and potassium C. Provide a low-protein, high-carbohydrate diet D. Restrict fluids according to the previous day's fluid loss

D. Restrict fluids according to the previous day's fluid loss

Why are adenomatous polyps removed during a colonoscopy? A. They eventually cause intestinal obstruction. B. They are prone to bleeding and lead to anemia. C. They lead to familial adenomatous polyposis (FAP). D. They are closely linked to colorectal cancer.

D. They are closely linked to colorectal cancer.

KIDENYS

KIDNEYS

Surgery is rarely used for PUD since ____ and ___ are so effective?

PPI, Antibiotics

Why is the fractional excretion of sodium (FENa) low in prerenal failure?

Prerenal failure results from hypoperfusion of the kidneys so this would also trigger the release of aldosterone, which retains sodium and water. Therefore, the FENa is low when its prerenal failure.

A nurse is teaching a client who has an autoimmune disease about the adverse effects of long-term corticosteroid therapy. Which of the following effects should the nurse include? (Select all that apply)? a. Osteoporosis b. Moon-shaped face c. Increased risk of infection d. Hearing loss e. Weight loss

a. Osteoporosis b. Moon-shaped face c. Increased risk of infection

A patient has hematemesis and melena. This is a sign of ___ , ___ GI bleeding

acute, upper

What are the 3 phases of acute kidney injury?

oliguric, diuretic and recovery

***SYNTHROID***

***SYNTHROID***

You are assessing a patient for a potential hyperthyroidism. What is the best assessment question? A. "Have you had any new tremors or palpitations recently?" B. "Are you having bronzing over your joints?" C. "How much calcium are you consuming?" D. "Were your children large babies?" Rationale

*A. "Have you had any new tremors or palpitations recently?"* Increased sympathetic nervous system activity, such as nervousness, palpitations, sweating, or tremors, can indicate hyperthyroidism. *Darkening pigmentation over joints is a sign of Addison's disease*. Calcium is not related to thyroid function; iodine is. Large babies are considered a sign of gestational diabetes.

A patient has been given instructions about levothyroxine (Synthroid). Which statement by the patient indicates understanding of these instructions? A. "I'll take this medication in the morning so as not to interfere with sleep." B. "I'll plan to double my dose if I gain more than 1 pound per day." C. "It is best to take the medication with food so I don't have any nausea." D. "I'll be glad when I don't have to take this medication in a few months."

*A. "I'll take this medication in the morning so as not to interfere with sleep."* Levothyroxine is used to treat hypothyroidism by increasing the basal metabolism and thus wakefulness. It is administered as a once-daily dose and is a lifelong therapy. It is best taken on an empty stomach to enhance absorption. Corticosteroids are taken with food to reduce the chance of peptic ulcers but Synthroid.

Which statement is the most important for a nurse to make to a patient who is taking methimazole? A. "You need to notify your doctor if you have a sore throat and fever." B. "Another medication can be given if you experience any nausea." C. "You may experience some muscle soreness with this medicine." D. "Headache and dizziness may occur but not very frequently."

*A. "You need to notify your doctor if you have a sore throat and fever."* Agranulocytosis (the absence of granulocytes to fight infection) is the most serious toxicity associated with methimazole. Sore throat and fever may be the earliest signs. Nausea, muscle soreness, and headache and dizziness are other adverse effects of methimazole that are not as serious as agranulocytosis.

Which of the following antacids is contraindicated for patients with heart failure or hypertension due to the risks of fluid retention? A. Alza-Seltzer (sodium bicarbonate) B. Tums (calcium carboante) C. Milk of Magnesia (magnesium hydroxide) D. Amphojel (aluminum hydroxide) E. Maalox or Mylanta (combination aluminum- magnesium antacids)

*A. Alza-Seltzer (sodium bicarbonate)* Note: Magnesium-containing antacids are contraindicated for patients with renal failure due to hypermagnesemia and Adverse effects -Aluminum and Calcium carbonate (remember how hypercalcemia causes constipation) can cause constipation -Magnesium antacids can cause diarrhea -Sodium bicarbonate (Alka-Seltzer) can cause fluid retention and is contraindicated in patients with heart failure or hypertension -Aluminum causes constipation and also binds with phosphate, causing HYPOphosphatemia.

A patient has developed a paralytic ileus following a recent abdominal surgery. What is the most important nursing consideration when caring for this patient? A. Ensure that the nasogastric tube (Salem Sump) is patent. B. Maintain the patient on a clear liquid diet. C. Maintain the patient on strict bed rest. D. Monitor motor strength in bilateral extremities.

*A. Ensure that the nasogastric tube (Salem Sump) is patent.*

Which are characteristic clinical manifestations of acute poststreptococal glomerulonephritis (APSGN)? A. Hematuria and dependent edema B. Anterior pelvic pain and glycosuria C. Lower back pain and inflamed kidneys D. Dyspnea and dull, aching pain over the urinary bladder

*A. Hematuria and dependent edema*

A patient is admitted to the emergency room with new onset abdominal pain. She is alert and oriented and she says the pain is in her upper right side. Which nursing intervention should be implemented first? A. Measure vital signs to detect hypovolemic changes. B. Administer the prescribed analgesics to promote patient comfort and relieve anxiety. C. Perform a thorough assessment of the onset, location, duration, and character of the pain. D. Perform a physical assessment of the abdomen for distension, masses, abnormal pulsations, and bowel sounds.

*A. Measure vital signs to detect hypovolemic changes.*

A patient is admitted to the emergency room with new onset abdominal pain. She is alert and oriented and she says the pain is in her upper right side. Which nursing intervention should be implemented first? A. Measure vital signs to detect hypovolemic changes. B. Administer the prescribed analgesics to promote patient comfort and relieve anxiety. C. Perform a thorough assessment of the onset, location, duration, and character of the pain. D. Perform a physical assessment of the abdomen for distension, masses, abnormal pulsations, and bowel sounds.

*A. Measure vital signs to detect hypovolemic changes.*

The elderly patient was informed that outpouches were found in the descending colon during the screening colonoscopy. The patient asks you what this finding means. What is the best explanation? A. Most people get these outpouchings as they age. B. These findings respond well to treatment with sulfa antibiotics. C. It is a precursor to colon cancer, and routine screening is essential. D. They contribute to malabsorption of cobalamin (vitamin B12) and fat.

*A. Most people get these outpouchings as they age.*

Which is the best indication that the patient has responded appropriately to levothyroxine? A. Normal thyroid-stimulating hormone (TSH) level B. Elevated free thyroxine (free T4) level C. Myxedema D. Cretinism

*A. Normal thyroid-stimulating hormone (TSH) level* Effective dosing with the thyroid replacement medication returns the patient to a euthyroid state with normal findings. Elevated TSH and decreased T4 levels are diagnostic findings for hypothyroidism. Myxedema is a severe hypothyroid coma, and cretinism is hypothyroidism in infants and young children.

Twenty-four hours after a surgical repair of a hiatal hernia, you notice bright red bloody drainage in the nasogastric tube. What should you do first? A. Notify the primary health care provider. B. Irrigate the nasogastric tube with ice water. C. Document the findings. D. Place the patient in a side-lying position.

*A. Notify the primary health care provider.* This is a sign of active gastrointestinal bleeding and needs further follow-up. Bloody drainage is expected for 8 to 12 hours. Because there is the potential for surgical complications or active ulcer bleeding, the health care provider must be consulted rather than only attempting to stop the bleeding with ice water irrigations. Action is required rather than documentation. Repositioning the patient will not resolve the problem

What is the most important finding in a patient who had a total thyroidectomy for thyroid cancer today? A. Positive Chvostek's sign B. Pain rated 8 C. Calcium level: 9 mg/dL D. White blood cell count: 11,500/μL Rationale

*A. Positive Chvostek's sign* Thyroid surgery can affect the parathyroid, which can cause hypocalcemia, indicated by a positive Chvostek's sign from the tetany caused by hypocalcemia. This systemic problem takes priority over pain. The calcium level is normal. The white blood cell count may be slightly elevated by the stress of surgery; it is probably too soon for infection.

For the patient with AKI, which laboratory result would cause you the greatest concern? A. Potassium level of 5.9 mEq/L B. BUN level of 25 mg/dL C. Sodium level of 144 mEq/L D. pH of 7.5

*A. Potassium level of 5.9 mEq/L* Hyperkalemia is one of the most serious complications in AKI because it can cause life-threatening cardiac dysrhythmias.

A patient with a history of peptic ulcer disease has presented to the emergency department with complaints of severe abdominal pain and a rigid, boardlike abdomen, prompting the health care team to suspect a perforated ulcer. Which of the following actions should the nurse anticipate? A. Providing IV fluids and inserting a nasogastric tube B. Administering oral bicarbonate and testing the patient's gastric pH level C. Performing a fecal occult blood test and administering IV calcium gluconate D. Starting parenteral nutrition and placing the patient in a high-Fowler's position

*A. Providing IV fluids and inserting a nasogastric tube* A perforated peptic ulcer requires IV replacement of fluid losses and continued gastric aspiration by NG tube. Nothing is given by mouth and gastric pH testing is not a priority. Calcium gluconate is not a medication directly relevant to the patient's suspected diagnosis and parenteral nutrition is not a priority in the short term.

A patient with a history of peptic ulcer disease is admitted to the emergency department with complaints of severe abdominal pain and a rigid, board-like abdomen. The health care team suspects a perforated ulcer. Which action should you anticipate? A. Providing IV fluids and monitoring nasogastric tube drainage B. Administering calcium gluconate and testing the patient's gastric pH level C. Performing a fecal occult blood test and administering IV calcium gluconate D. Starting parenteral nutrition and placing the patient in a high Fowler's position

*A. Providing IV fluids and monitoring nasogastric tube drainage*

The nurse is caring for a patient in the emergency department with complaints of acute abdominal pain, nausea, and vomiting. When the nurse palpates the patient's left lower abdominal quadrant, the patient complains of pain in the right lower quadrant. The nurse will document this as which of the following diagnostic signs of appendicitis? A. Rovsing sign B. Referred pain C. Chvostek's sign D. Rebound tenderness

*A. Rovsing sign* In patients with suspected appendicitis, Rovsing sign may be elicited by palpation of the left lower quadrant, causing pain to be felt in the right lower quadrant.

The patient has small cell lung cancer. Which finding indicates a complication of the condition? A. Serum sodium level: 128 mg/dL B. Serum potassium level: 4.5 mEq C. Urine output: 30 mL/hour D. Urine specific gravity: 1.012

*A. Serum sodium level: 128 mg/dL* Malignancy, especially small cell lung cancer, can cause syndrome of inappropriate antidiuretic hormone (SIADH), which results in increased reabsorption of water and dilutional hyponatremia. The other three options are normal findings. *lecture she talked about oat cell cancer causing this*

Which urinalysis results most likely indicate glomerular damage? A. Smoky color; 30 mg/dL of protein; pH of 6.2 B. Cloudy, yellow; 50 WBCs/hpf; pH of 8.2; numerous casts C. Cloudy, brown; ammonia odor; specific gravity of 1.030; 3 RBCs/hpf. D. Clear, colorless; trace of glucose; trace of ketones; osmolality of 500 mOsm/kg (500 mmol/kg)

*A. Smoky color; 30 mg/dL of protein; pH of 6.2* The clinical manifestations of acute poststreptococcal glomerulonephritis (APSGN) appear as a variety of signs and symptoms, including generalized body edema, hypertension, oliguria, hematuria with a smoky or rusty appearance, and proteinuria may occur.

The patient with Crohn's disease has had multiple intestinal resections. Which symptom indicates that short bowel syndrome has developed? A. Steatorrhea B. Constipation C. Hypercholesteremia D. Hypercalcemia

*A. Steatorrhea* The predominant manifestation is diarrhea or steatorrhea. Diarrhea, not constipation, is a concern because there is decreased intestinal surface to absorb fluid and nutrients. Decreased absorption of bile salts is the issue; increased cholesterol is not related to short bowel syndrome. The risk is deficiencies of cobalamin, zinc, and calcium

The nurse should question an order for glucocorticoids in the treatment of a patient with what? A. Systemic fungal infection B. Diabetes mellitus C. Myasthenia gravis D. Glaucoma

*A. Systemic fungal infection* Glucocorticoids are contraindicated in the treatment of a patient with a systemic fungal infection or in patients receiving live vaccines. Glucocorticoids should be used with caution in patients with diabetes mellitus, myasthenia gravis, and glaucoma. *This question is from Lehne and relates to material on a handout on Moodle

Which manifestations should a nurse investigate first when monitoring a patient who is taking levothyroxine (Synthroid)? A. Tachycardia B. Tremors C. Insomnia D. Irritability

*A. Tachycardia*

The nurse is caring for a patient with lung cancer that has metastasized to the bone. The patient's 08:00 serum calcium level is 12.8 mg/dL. A. The nurse will administer the scheduled Calcitonin-Salmon nasal spray B. The nurse will hold the scheduled Calcitonin- Salmon nasal spray

*A. The nurse will administer the scheduled Calcitonin-Salmon nasal spray* The normal total serum calcium level ranges from 8.6-10.2 so a level of 12.8 is hypercalcemia. Calcitonin "tones down" calcium so she would administer the nasal spray.

Which patient has the greatest risk for prerenal AKI? A. The patient is hypovolemic because of hemorrhage. B. The patient relates a history of chronic urinary tract obstruction. C. The patient has vascular changes related to coagulopathies. D. The patient is receiving antibiotics such as gentamicin

*A. The patient is hypovolemic because of hemorrhage.*

Which patient has the greatest risk for prerenal AKI? A. The patient is hypovolemic because of hemorrhage. B. The patient relates a history of chronic urinary tract obstruction. C. The patient has vascular changes related to coagulopathies. D. The patient is receiving antibiotics such as gentamicin.

*A. The patient is hypovolemic because of hemorrhage.* Prerenal causes of AKI are factors external to the kidneys. These factors reduce systemic circulation, causing a reduction in renal blood flow, and they lead to decreased glomerular perfusion and filtration of the kidneys. Reference: 1165

Diagnostic testing is planned for a patient with a suspected peptic ulcer. The nurse explains to the patient that the most reliable test for the presence and location of an ulcer is a(n): A. Upper GI endoscopy B. computed tomography (CT) scan C. barium enema D. serologic test for H. pylori

*A. Upper GI endoscopy*

What is the best test to diagnose stomach cancer? A. Upper endoscopic examination B. Barium enema C. Endoscopic retrograde cholangiopancreatogram (ERCP) D. Magnetic resonance imaging (MRI)

*A. Upper endoscopic examination*

You are caring for a patient with metastatic bone cancer. Which of the following clinical manifestations would alert you to the possibility of hypercalcemia in this patient? A. Weakness B. Paresthesia C. Facial spasms D. Muscle tremors

*A. Weakness* Signs of hypercalcemia are lethargy, headache, weakness, muscle flaccidity, heart block, anorexia, nausea, and vomiting. Paresthesia, facial spasms, and muscle tremors are symptoms of hypocalcemia.

Which is a characteristic of SIADH? A. Weight gain without edema B. Acute renal failure C. Hypernatremia D. Temperature

*A. Weight gain without edema* Rationale In SIADH, ADH is released despite normal or low plasma osmolarity. It is characterized by reabsorption of water that can lead to fluid retention, serum hypoosmolality, dilutional hyponatremia, and concentrated urine in the presence of normal intravascular volume and normal renal functions.

To help reduce the incidence of acute glomerulonephritis, public health teaching can stress the importance of A. early treatment of strep throat. B. obtaining a yearly screening urinalysis. C. proper urinary hygiene to prevent cystitis. D. early immunizations against streptococcal pneumonia

*A. early treatment of strep throat.*

A patient is admitted to the hospital with a diagnosis of Cushing syndrome. On physical assessment of the patient, the nurse would expect to find A. hypertension, peripheral edema, and petechiae. B. weight loss, buffalo hump, and moon face with acne. C. abdominal and buttock striae, truncal obesity, and hypotension. D. anorexia, signs of dehydration, and hyperpigmentation of the skin.

*A. hypertension, peripheral edema, and petechiae.* Rationale: Many of these are true but contain one that is not true. The effects of glucocorticoid excess include weight gain from accumulation and redistribution of adipose tissue, sodium and water retention, glucose intolerance, protein wasting, loss of bone structure, loss of collagen, and capillary fragility. Clinical manifestations of corticosteroid deficiency include hypotension, dehydration, weight loss, and hyperpigmentation of the skin.

During care of the patient with SIADH, the nurse should a. monitor neurologic status at least every 2 hours. b. keep the head of the bed elevated to prevent ADH release. c. teach the patient receiving treatment with diuretics to restrict sodium intake. d. notify the health care provider if the patient's blood pressure (BP) decreases more than 20 mm Hg from baseline

*A. monitor neurologic status at least every 2 hours.* Rationale: The patient with syndrome of inappropriate secretion of antidiuretic hormone (SIADH) has marked dilutional hyponatremia and should be monitored for decreased neurologic function and convulsions every 2 hours. ADH release is reduced by keeping the head of the bed flat to increase left atrial filling pressure and ANP is secreted as well promoting diuresis. Sodium intake is supplemented because of the hyponatremia and sodium loss caused by diuretics. A reduction in blood pressure (BP) indicates a reduction in total fluid volume and is an expected outcome of treatment. You want decreased blood pressure.

If patient has 1000 mL more fluid loss than intake when you check the I/O then you would expect a weight loss of? A. 1 pound B. 2 pounds C. 3 pounds D. 4 pounds

*B. 2 pounds* 1000 mL=1000 grams or 1 kg 1 kg=2.2 pounds so roughly 2 pounds *LECTURE EXAM HINT*

The nurse notes that a patient gained 1 lbs in 24 hours. This is equivalent to a fluid gain of approximately: A. 250 mL B. 500 mL C. 750 mL D. 1,000 mL

*B. 500 mL* 2.2 pounds=1kg 1 pound equals roughly 0.5 kg 0.5kg=500g 1 gram=1 millilter, so 500 mL *LECTURE EXAM HINT*

For which of the following clients would radioactive iodine, a treatment for hyperthyroidism, be contraindicated? A. A client who is over 40 years of age B. A client who is pregnant C. A client who has had a recent myocardial infarction D. A client who is scheduled for thyroid surgery

*B. A client who is pregnant* LECTURE: she mentioned that another drug for hyperthyroidism called Methimazole (Tapazole) is pregnancy category D. Methimazole also causes agranulocytosis *LO*

Which patient is at greatest risk for Goodpasture syndrome? A. An older adult with a history of urinary tract infections (UTIs) B. A young adult man who smokes C. A young adult woman who takes contraceptives D. An older adult male with benign prostatic hypertrophy Rationale

*B. A young adult man who smokes* Goodpasture syndrome is a rare disease that is seen mostly in young male smokers. The clinical manifestations include flu-like symptoms with pulmonary symptoms that include cough, mild shortness of breath, hemoptysis, crackles, rhonchi, and pulmonary insufficiency. Renal involvement causes hematuria, weakness, pallor, anemia, and renal failure. Pulmonary hemorrhage usually occurs and may precede glomerular abnormalities by weeks or months. Abnormal diagnostic findings include low hematocrit and hemoglobin levels, elevated BUN and serum creatinine levels, hematuria, and proteinuria. Circulating serum *anti-GBM antibodies* parallel the activity of the renal disease and are diagnostic of this syndrome.

To evaluate the effectiveness of treatment for the patient with nephrotic syndrome, you assess the A. Blood pressure q4h B. Abdominal girth daily C. Urine of each voiding for protein D. Daily dietary protein intake

*B. Abdominal girth daily*

To evaluate the effectiveness of treatment for the patient with nephrotic syndrome, you assess the A. Blood pressure q4h B. Abdominal girth daily C. Urine of each voiding for protein D. Daily dietary protein intake Rationale

*B. Abdominal girth daily* A major nursing intervention for a patient with nephrotic syndrome is related to edema. It is important to assess the edema by weighing the patient daily, accurately recording intake and output, and measuring abdominal girth or extremity size. Comparing this information daily provides you with a tool for assessing the effectiveness of treatment.

A patient is receiving glucocorticoids for the treatment of rheumatoid arthritis. The patient complains of having a headache. Which ordered medication should the nurse administer? A. Aspirin (Bayer) B. Acetaminophen (Tylenol) C. Ibuprofen (Advil) D. Naproxen sodium (Aleve)

*B. Acetaminophen (Tylenol) * The risk of gastrointestinal irritation and ulceration for a patient taking glucocorticoids is already increased so it is made worse by concurrent use of other medications, such as aspirin and nonsteroidal anti-inflammatory drugs (NSAIDs) that increase GI irritation and lead to ulcers.

Which behavior can help people with lactase deficiency to tolerate milk? A. Drink the milk on an empty stomach 30 minutes before meals. B. Add Lactaid to the consumed milk. C. Consume the milk with calcium supplement. D. Use on buttermilk.

*B. Add Lactaid to the consumed milk.* The lactase enzyme (Lactaid) is available as an over-the-counter product to mix with milk and break down the lactose before the milk is ingested. Some can tolerate the lactose better if lactase is taken with meals. Although lack of milk consumption can lead to a calcium deficiency, consuming it with calcium will not make a difference. The symptoms result from an inability to digest lactose, not a calcium deficiency. Buttermilk has additional fat, but it also has lactose.

Which nursing action should be done for a patient with DI? A. Monitor levels of urine ketones. B. Administer desmopressin acetate (DDAVP). C. Administer prednisone by intravenous push (IVP). D. Monitor blood glucose levels hourly. Rationale

*B. Administer desmopressin acetate (DDAVP).* Patients with DI have decreased production and secretion of ADH and increased urine output with low specific gravity. DDAVP is used for ADH replacement. DI is not related to glucose metabolism and ketones and does not need close monitoring. Prednisone is not used to treat DI.

Which of these prescriber interventions will the nurse implement first when caring for a patient who has just been diagnosed with peritonitis caused by a ruptured diverticulum? A. Administer the PRN ordered stool softener orally. B. Administer the metronidazole (Flagyl) 500 mg IV. C. Draw blood for a complete blood count (CBC) to determine the white blood cell count (WBC) D. Encourage the patient to drink fluids and eat foods that are high in fiber.

*B. Administer the metronidazole (Flagyl) 500 mg IV* Complicated diverticulitis (a burst infected diverticula) is treated with NPO status, a broad spectrum antibiotic and opioids for pain, followed by surgery. *Why not A?* The patient is NPO. A diet high in fiber or a stool softener is done to prevent this not treat it *Why not C?* This is done to diagnose diverticulitis. The patient has been diagnosed already and is now being treated. *Why not D?* The patient is NPO. This is preventative.

Which of these prescriber interventions will the nurse implement first when caring for a patient who has just been diagnosed with peritonitis caused by a ruptured diverticulum? A. Administer the PRN ordered stool softener orally. B. Administer the metronidazole (Flagyl) 500 mg IV. C. Draw blood for a complete blood count (CBC) to determine the white blood cell count (WBC) D. Encourage the patient to drink fluids and eat foods that are high in fiber.

*B. Administer the metronidazole (Flagyl) 500 mg IV.*

A colectomy is scheduled for a 68-year-old woman with an abdominal mass, possible bowel obstruction, and a history of rectal polyps. The nurse should plan to include which of the following prescribed measures in the preoperative preparation of this patient? A. Instruction on irrigating a colostomy B. Administration of a cleansing enema C. A high-fiber diet the day before surgery D. Administration of IV antibiotics for bowel preparation

*B. Administration of a cleansing enema* Preoperative preparation for bowel surgery typically includes bowel cleansing with antibiotics, such as oral neomycin and cleansing enemas, including Fleet enemas.

Which is correct information about the treatment of Crohn's disease? A. Surgery is the preferred treatment. B. Aminosalicylates are frequently used first. C. Corticosteroids are given for long-term therapy. D. High-fiber foods are encouraged to add bulk to diarrheal stool.

*B. Aminosalicylates are frequently used first.* -Surgery rarely works with Crohn's as it does with UC -Corticosteroids are used short term -High fiber/residue foods avoided -Aminosalicylates reduce inflammation

Which is correct information about the treatment of Crohn's disease? A. Surgery is the preferred treatment. B. Aminosalicylates are frequently used first. C. Corticosteroids are given for long-term therapy. D. High-fiber foods are encouraged to add bulk to diarrheal stool.

*B. Aminosalicylates are frequently used first.* Aminosalicylates (5-ASAs) are used first because they are less toxic, although there is a movement to using biologic and targeted therapy as first-line therapy. *Drugs with 5-ASA suppress the proinflammatory cytokines and inflammatory mediators*. Because there is a high rate of recurrence after surgical treatment of Crohn's disease, medication is the preferred treatment, although up to 75% of patients eventually must have surgery. *Corticosteroids are given for the shortest time possible* due to the complications of long-term therapy. *Patients are put on a low-residue diet.*

Which nursing diagnosis should you include when developing a plan of care for a patient with hypothyroidism? A. Diarrhea related to gastrointestinal hypermotility B. Anxiety related to forgetfulness and slowed speech C. Imbalanced nutrition: less than body requirements D. Activity intolerance related to increased metabolic rate Rationale

*B. Anxiety related to forgetfulness and slowed speech* Disturbed thought processes can cause the patient to be anxious. Hypothyroidism slows mental processes. The other options are appropriate for hyperthyroidism.

What is the most serious complication that can result from frequent, untreated gastric reflux? A. Dental enamel erosion B. Barrett's esophagus C. Leukoplakia D. Aspiration pneumonia

*B. Barrett's esophagus* A complication of frequent untreated gastric reflux is Barrett's esophagus or esophageal metaplasia, which can lead to esophageal adenocarcinoma. Enamel erosion results from frequent induced vomiting (e.g., bulimia) with gastric acid reflux, although the reflux does not ordinarily reach the mouth. Leukoplakia is the white patch in the oral cavity from constant irritation, such as from a pipe. Aspiration of the reflux is not a common problem in healthy individuals; it can be a problem with patients who are on mechanical ventilators.

A patient with a history of end-stage renal disease (ESRD) resulting from diabetes mellitus has presented to the outpatient dialysis unit for his scheduled hemodialysis. Which assessment should you prioritize before, during, and after his treatment? A. Level of consciousness B. Blood pressure and fluid balance C. Temperature, heart rate, and blood pressure D. Assessment for signs and symptoms of infection

*B. Blood pressure and fluid balance* hypotension and hypovolemia can occur with HD. Although all of the assessments are relevant to the care of a patient receiving hemodialysis, the nature of the procedure indicates a particular need to monitor blood pressure and fluid balance.

A patient with a history of end-stage renal disease secondary to diabetes mellitus has presented to the outpatient dialysis unit for his scheduled hemodialysis. Which of the following assessments should the nurse prioritize before, during, and after his treatment? A. Level of consciousness B. Blood pressure and fluid balance C. Temperature, heart rate, and blood pressure D. Assessment for signs and symptoms of infection

*B. Blood pressure and fluid balance* Although all of the assessments are relevant to the care of a patient receiving hemodialysis, the nature of procedure indicates a particular need to monitor patients' blood pressure and fluid balance.

A nurse in a provider's office is reviewing the health record of a client who is being evaluated for Graves' disease. Which of the following is an expected laboratory finding for this client? A. Decreased thyrotropin receptor antibodies B. Decreased thyroid stimulating hormone C. Decreased free thyroxine index D. Decreased triiodothyronine

*B. Decreased thyroid stimulating hormone* In the presence of Graves' disease, low thyroid stimulating hormone (TSH) is an expected finding. The pituitary gland decreases the production of TSH when thyroid hormone levels are elevated

What are the immunologic mechanisms involved in glomerulonephritis? A. Tubular blocking by precipitates of bacteria and antibody reactions B. Deposition of immune complexes and complement along the glomerular basement membrane (GBM) C. Thickening of the GBM from autoimmune microangiopathic changes D. Destruction of glomeruli by proteolytic enzymes contained in the GBM

*B. Deposition of immune complexes and complement along the glomerular basement membrane (GBM)* All forms of immune complex disease are characterized by an accumulation of antigen, antibody, and complement in the glomeruli, which can result in tissue injury. The immune complexes activate complement. Complement activation results in the release of chemotactic factors that attract polymorphonuclear leukocytes, histamine, and other inflammatory mediators. The result of these processes is glomerular injury.

What is a priority nursing diagnosis for the patient with nephrotic syndrome? A. Imbalanced nutrition: less than body requirements B. Disturbed body image C. Decreased cardiac output D. Acute pain

*B. Disturbed body image* Support for the patient, in terms of coping with an altered body image, is essential because there is often embarrassment and shame associated with the edematous appearance.

A patient is experiencing a gnawing, burning pain that radiates to the back about 2-5 hours after a meal. This is most likely a peptic ulcer in the? A. Stomach B. Duodenum C. Large intestine D. Colon

*B. Duodenum* A gnawing, burning pain that radiates to the back is a peptic ulcer. If it happens 1 hour after a meal it is most likely in the stomach, 2-5 its in the duodenum. They happen most commonly in the duodenum. It is often relieved by eating.

A nurse is caring for a client who has stage 4 chronic kidney disease. Which of the following is an expected laboratory finding? A. Blood urea nitrogen (BUN) 54 mg/dL B. Glomerular filtration rate (GRF) 20 mL/min C. Serum creatinine 1.2 mg/dL D. Serum potassium 5.0 mEq/L

*B. Glomerular filtration rate (GRF) 20 mL/min (normal is 125 mL/min)* A-a normal BUN is 7-20, it would be elevated very high in stage 4 B-A normal GFR is 125mL/min so 20 mL/min is VERY low C-a normal creatinine is 0.5-1.2 so 1.2 is still normal. It would be elevated very high in stage 4 D-normal serum potassium is 3.5-5. Hyperkalemia is 5 or higher and there would be hyperkalemia with stage 4 kidney failure

What finding is common when assessing a patient with a hiatal hernia? A. Pulsating abdominal mass B. Heartburn when lying supine C. Pernicious anemia D. Hypoactive bowel sounds

*B. Heartburn when lying supine*

The patient who is admitted with a diagnosis of diverticulitis and a history of irritable bowel disease and gastroesophageal reflux disease (GERD) has received a dose of Mylanta 30 ml PO. The nurse would evaluate its effectiveness by questioning the patient as to whether which of the following symptoms has been resolved? A. Diarrhea B. Heartburn C. Constipation D. Lower abdominal pain

*B. Heartburn* Mylanta is an antacid that contains both aluminum and magnesium. It is indicated for the relief of GI discomfort, such as with heartburn associated with GERD.

The nurse is planning care for a 68-year-old patient with an abdominal mass and suspected bowel obstruction. Which of the following factors in the patient's history increases the patient's risk for colorectal cancer? A. Osteoarthritis B. History of rectal polyps C. History of lactose intolerance D. Use of herbs as dietary supplements

*B. History of rectal polyps* A history of rectal polyps places this patient at risk for colorectal cancer. This tissue can degenerate over time and become malignant. The other factors identified do not pose additional risk to the patient.

Which of the following assessment findings is a consequence of the oliguric phase of acute kidney injury (AKI)? A. Hypovolemia B. Hyperkalemia C. Hypernatremia D. Thrombocytopenia

*B. Hyperkalemia* In AKI the serum potassium levels increase because the normal ability of the kidneys to excrete potassium is impaired. Sodium levels are typically normal or diminished, whereas fluid volume is normally increased because of decreased urine output. Thrombocytopenia is not a consequence of AKI, although altered platelet function may occur in AKI.

In addition to urinary symptoms, patients with APSGN may also present with which symptom? A. Rash over the upper trunk B. Hypertension C. Halos around lights D. Disorientation

*B. Hypertension* (oliguric stage?)

A nurse is caring for a patient with decreased triiodothyronine (T3) and thyroxine (T4) and elevated thyroid-stimulating hormone (TSH) levels. The nurse knows the patient is likely suffering from what? A. Thyrotoxicosis B. Hypothyroidism C. Hyperthyroidism D. Graves' disease

*B. Hypothyroidism* The anterior pituitary increases production of TSH when thyroid hormone levels of T3 and T4, are reduced, reflecting primary hypothyroidism. Patients may experience fatigue caused by a lowered basal metabolic rate. Thyrotoxicosis, hyperthyroidism, and Graves' disease are medical conditions indicative of excessive thyroid activity.

To normalize a low serum calcium level, the body releases parathyroid hormone (PTH); this results in which therapeutic effect? A. Decrease in the intestinal absorption of calcium B. Increase in bone resorption of calcium C. Increase in renal calcium excretion D. Increase in plasma levels of phosphate

*B. Increase in bone resorption of calcium* PTH restores normal calcium levels by promoting calcium resorption from bone and transferring it to the blood. PTH also activates vitamin D, which increases, not decreases, calcium absorption from the intestine. Renal losses of calcium are reduced by PTH. PTH reduces plasma levels of phosphate.

What screening test should you recommend to a white person with an average risk of colorectal cancer? A. Flexible sigmoidoscopy at age 21 B. Initial colonoscopy starting at age 50 C. Stool DNA every 10 years D. Carcinoembryonic antigen (CEA) yearly

*B. Initial colonoscopy starting at age 50* Whites with an average risk should have a colonoscopy every 10 years starting at age 50 (African Americans should have the first one at age 45). The use of a flexible sigmoidoscopy is decreasing because it evaluates only about 50% of the colon. When used, it is begun at age 50. Stool DNA is less favorable but acceptable. Stool tests must be done frequently since DNA shedding occur at intervals and may be easily missed. CEA is used to monitor disease recurrence after surgery or chemotherapy.

What is a key distinction between ulcerative colitis and Crohn's disease? A. Presence of intermittent constipation B. Pattern of inflammation C. Age of onset D. Prescribed diet

*B. Pattern of inflammation* The pattern of inflammation differs between Crohn's disease and ulcerative colitis. Crohn's disease involves all layers of the bowel wall anywhere in the gastrointestinal tract. Ulcerative colitis usually starts in the rectum and moves progressively toward the cecum, staying mainly located in the colon and rectum. Inflammation occurs in the mucosal layer. Intermittent diarrhea and constipation is a symptom of irritable bowel syndrome; diarrhea can occur in both diseases. Both commonly occur during the teenage years and early adulthood, with a second peak in the sixth decade. Both are treated similarly in terms of bowel rest, diet, and drugs.

When caring for a patient with acute kidney injury who is hyperkalemic, which of the following prescribed actions should the nurse take first? A. Administer Sevelamer (Renagel) B. Place the patient on a cardiac monitor. C. Administer sodium polystyrene sulfonate (Kayexalate). D. Look in the EHR for the patient's current blood urea nitrogen (BUN) and creatinine levels.

*B. Place the patient on a cardiac monitor.*

A nurse treating a patient in Addisonian crisis should avoid administering? A. Hydrocortisone B. Potassium chloride C. Vassopressors D. Dextrose

*B. Potassium chloride* They are severely hyperkalemic already. The treatment includes the administration of hydrocortisone, saline and dextrose.

Which medication usually is prescribed for patients with nephrotic syndrome? A. Sulfa B. Prednisone C. Amoxicillin D. Sulfisoxazole

*B. Prednisone* Corticosteroids and cyclophosphamide (Cytoxan) may be used for the treatment of severe cases of nephrotic syndrome. Prednisone has been effective to various degrees in persons with early-stage nephrosis, membranous glomerulonephritis, proliferative glomerulonephritis, and lupus nephritis.

What constituent is expected when evaluating the urinalysis of a patient with acute glomerulonephritis? A. Microscopic calculi B. Red blood cells and protein C. Escherichia coli D. Platelets

*B. Red blood cells and protein* Dipstick urinalysis and urine sediment microscopy reveal the presence of erythrocytes in significant numbers. Erythrocyte casts are highly suggestive of APSGN. Proteinuria may range from mild to severe. Screening blood tests include BUN and serum creatinine levels to assess the extent of renal impairment.

The patient with hyperparathyroidism has a calcium level of 12 mg/dL. What complication should you assess the patient for? A. Tetany B. Renal calculi C. Periorbital edema D. Hyperglycemia

*B. Renal calculi* Rationale The PTH causes hypercalcemia, and the kidneys cannot reabsorb the excess calcium, leading to hypercalciuria. Along with the large amount of urinary phosphate, this can lead to calculi formation. *Tetany is related to hypocalcemia*. Periorbital edema is related to hypothyroidism. Hyperglycemia is not a symptom related to hyperparathyroidism.

What is a priority nursing diagnosis for the patient with nephrotic syndrome? A. Activity intolerance B. Risk for infection C. Decreased cardiac output D. Imbalanced nutrition: less than body requirements

*B. Risk for infection* The patient with nephritic syndrome is susceptible to infection and should take measures to avoid exposure to persons known to have infections.

How does the drug sulfasalazine (Azulfidine) work in the treatment of IBD? A. Destroys bacteria B. Suppresses inflammatory mediators C. Slows gastric motility D. Promotes electrolyte exchange across intestinal membrane

*B. Suppresses inflammatory mediators* Sulfasalazine contains sulfapyridine and 5-aminosalicylic acid (5-ASA). Although the exact action is unknown, it works by suppressing inflammatory mediators. IBD is an autoimmune inflammatory disease; no specific infectious agent has been identified, although antimicrobials (Flagyl, Cipro) occasionally are used. The last two options are not related to this drug.

How does the drug sulfasalazine (Azulfidine) work in the treatment of IBD? A. Destroys bacteria B. Suppresses inflammatory mediators C. Slows gastric motility D. Promotes electrolyte exchange across intestinal membrane Rationale

*B. Suppresses inflammatory mediators* Sulfasalazine contains sulfapyridine and 5-aminosalicylic acid (5-ASA). Although the exact action is unknown, it works by suppressing inflammatory mediators. IBD is an autoimmune inflammatory disease; no specific infectious agent has been identified, although *antimicrobials (Flagyl, Cipro)* occasionally are used. The last two options are not related to this drug

The nurse reviews the patient's medication record and notes the following: Sucralfate (Carafate) 1 gram orally four times daily before meals (7:30 AM, 11:30 AM, and 4:30 PM) and at bedtime (10:00 PM); phenytoin (Dilantin) 200 mg orally daily at 8 AM. Which modifications, if any, should be made to the medication regimen? A. The medications can be administered as ordered. B. The nurse should obtain a prescriber order to administer the phenytoin at 9:30 AM daily. C. The nurse should obtain a prescriber order for intravenous phenytoin to avoid a drug interaction. D. The nurse should administer the phenytoin with the 7:30 AM dose of sucralfate (Carafate), because this is more time efficient.

*B. The nurse should obtain a prescriber order to administer the phenytoin at 9:30 AM daily.* Sucralfate can impede the absorption of other drugs; therefore, a period of 2 hours should separate these drugs. The nurse should consult the prescriber for a time administration change. Based on this information, all other options are incorrect. *LECTURE: interferes with the absorption of some medications*

The nurse is providing discharge instructions to a patient with diabetes insipidus. Which of the following instructions regarding desmopressin acetate (DDAVP) would be most appropriate? A. The patient can expect to experience weight loss resulting from increased diuresis. B. The patient should alternate nostrils during administration to prevent nasal irritation. C. The patient should monitor for symptoms of hypernatremia as a side effect of this drug. D. The patient should report any decrease in urinary elimination to the health care provider.

*B. The patient should alternate nostrils during administration to prevent nasal irritation.* DDAVP is used to treat diabetes insipidus by replacing the antidiuretic hormone (ADH) that the patient is lacking. DDAVP can cause nasal irritation, headache, nausea, and other signs of hyponatremia. A. Weight GAIN due to DECREASED dieresis B. correct C. HYPONATREMIA due to dilution with water at the distal tubules D. Decreased urinary elimination is expected.

Which is a complication in patients with ulcerative colitis? A. Hyperkalemia B. Toxic megacolon C. Pancreatitis D. Barrett's esophagus

*B. Toxic megacolon* Colonic dilation (toxic megacolon) can occur as a result of decreased tissue function, with lack of peristalsis and enlargement of the colon. The patient is at risk for perforation. Patients with inflammatory bowel disease (IBD) are at risk for *hypokalemia related to diarrhea*. Pancreatitis is not a risk related to IBD. Barrett's esophagus is related to gastroesophageal reflux disorder (GERD) and acid reflux.

Natriuretic peptides serve to protect the cardiovascular system under which condition? A. Hypovolemia B. Volume overload C. Myocardial infarction D. Hypotension

*B. Volume overload.* Natriuretic peptides protect the cardiovascular system in the event of volume overload. They work by reducing blood volume and promoting the dilation of arterioles and veins. ANP is released when blood pressure stretches the right atrium. For this reason promoting diuresis in patients with SIADH involves not elevating the HOB to above 10 degrees. ANP is the opposite 0f ADH. It promotes the excretion of sodium and water.

Which urinalysis result do you recognize as an abnormal finding? A. pH of 6.0 B. WBC count: 9/hpf C. Amber yellow color D. Specific gravity of 1.025

*B. WBC count: 9/hpf* Normal WBC levels in urine are below 5/hpf, and levels exceeding this indicate inflammation or urinary tract infection. Amber yellow is normal coloration, and a pH of 6.0 is average. The reference range for specific gravity is 1.003 to 1.030.

What should you teach a patient taking sucralfate (Carafate)? A. Watch for deficiency of fat-soluble vitamins. B. Will interfere with the absorption of many medications. C. Administer 1 hour after meals. D. Take with antacids for maximum effectiveness.

*B. Will interfere with the absorption of many medications.* Sucralfate is used for the short-term treatment of peptic ulcers because it forms an ulcer-adherent complex that covers the ulcer. It is known for binding with many drugs, including digoxin and warfarin. It does not affect fat-soluble vitamin absorption. It is most effective at a low pH and should be given at least 30 minutes before or after an antacid and meals.

You would increase the comfort of the patient with appendicitis by A. having the patient lie prone. B. flexing the patient's right knee. C. sitting the patient upright in a chair. D. turning the patient onto his or her left side.

*B. flexing the patient's right knee.*

A patient with renal disease has oliguria and a creatinine clearance of 40 mL/minute. You recognize that these findings most directly reflect abnormal function of A. tubular secretion. B. glomerular filtration. C. capillary permeability. D. concentration of filtrate.

*B. glomerular filtration.* The amount of blood filtered each minute by the glomeruli is expressed as the glomerular filtration rate (GFR). The normal GFR is about 125 mL/minute.

Several patients are seen at an urgent care center with symptoms of nausea, vomiting, and diarrhea that began 2 hours ago while attending a potluck dinner at a large family reunion. You question the patients specifically about foods they ingested containing A. beef. B. meat and milk. C. poultry and eggs. D. home-preserved vegetables.

*B. meat and milk.* Staphylococcus aureus toxins produce onset of symptoms (vomiting, nausea, abdominal cramping, and diarrhea) within 30 minutes and for up to 7 hours. Meat, bakery products, cream fillings, salad dressings, and milk are the usual sources of contamination from the skin and respiratory tract of food handlers

In contrast to diverticulitis, the patient with diverticulosis A. has rectal bleeding. B. often has no symptoms. C. has localized cramping pain. D. frequently develops peritonitis.

*B. often has no symptoms.*

A nurse teaches a patient who has ulcerative colitis about the side effects of the treatment medication, sulfasalazine (Azulfidine). Which statement by the patient would indicate understanding of the information? A. "My tongue may become discolored and my taste altered." B. "I may have constipation, so I'll increase my fluid intake." C. "I'll report any fatigue or sore throat and fever to my doctor." D. "I'll immediately report any chest pain or shortness of breath."

*C. "I'll report any fatigue or sore throat and fever to my doctor."* Sulfasalazine is used to treat ulcerative colitis by suppressing inflammation. It has the adverse hematologic effects of *agranulocytosis, hemolytic anemia*, and macrocytic anemia. Patients should report any signs of infection and/or fatigue. Altered taste, tongue discoloration, constipation, chest pain, and shortness of breath are not effects associated with sulfasalazine.

A patient with Graves' disease is treated with iodine-131 (Iodope) therapy. Which statement by the patient would indicate understanding of the treatment's effects? A. "I'll have to isolate myself from my family so I don't expose them to radiation." B. "I'm looking forward to feeling better immediately after this treatment." C. "I'll tell my doctor if I have fatigue, hair loss, or cold intolerance." D. "I'll need to take this drug on a daily basis for at least 1 year."

*C. "I'll tell my doctor if I have fatigue, hair loss, or cold intolerance."* Iodine-131 usually is given as a single treatment to produce remission of Graves' disease. Fatigue, hair loss, and cold intolerance are signs of hypothyroidism, which is a complication of the treatment. Iodine-131 has a quick radioactive decay and half-life; therefore, isolation is not needed, but it can take up to 2 months for the desired response to develop. In lecture the professor talked about how the urine is not very radioactive and the family is not at risk.

A nurse should give which nonmedication instruction to a patient who has peptic ulcers? A. "Reduce your intake of caffeine-containing beverages, such as coffee and colas." B. "Take a nonsteroidal anti-inflammatory drug once a day to help with pain." C. "It would be better to eat five or six small meals a day instead of three larger ones." D. "An ulcer diet of bland foods with milk and cream products will speed healing."

*C. "It would be better to eat five or six small meals a day instead of three larger ones."* Some optimal nondrug measures, in addition to drug management, to aid patients with peptic ulcers include changing the eating pattern to more frequent, smaller meals to avoid fluctuations in intragastric pH. No evidence indicates that beverages containing caffeine promote ulcer formation or that an "ulcer diet" improves healing. Nonsteroidal anti-inflammatory drugs (NSAIDs) inhibit the biosynthesis of prostaglandins, which reduce mucosal blood flow and promote the secretion of gastric acid.

A patient who has peptic ulcer disease and is receiving magnesium hydroxide (milk of magnesia) is experiencing an increased number of bowel movements. Which is the nurse's priority action? A. Ask the healthcare provider for a reduction in dose. B. Encourage the patient to increase dietary fiber. C. Administer the drug with an aluminum hydroxide antacid. D. Instruct the patient to keep an accurate stool count.

*C. Administer the drug with an aluminum hydroxide antacid.* Magnesium hydroxide is a rapid-acting antacid with a prominent adverse effect of diarrhea. To compensate, it usually is administered in combination with aluminum hydroxide, which promotes constipation. A reduction in dose might be necessary if the diarrhea is severe, but this is not the priority action. Increasing dietary fiber and keeping a stool count are appropriate actions to implement after adding an antacid to counteract the diarrhea effect. *Magnesium=diarrhea, Aluminum=constipation*

The nurse teaches the patient with gastroesophageal reflux disease (GERD) to control or reduce symptoms by doing the following? A. Lie down for 30 minutes after each meal B. Drink at least 8 ounces of water with each meal C. Avoid milk, especially at bedtime, because it increases gastric secretions D. Eat a diet that is high in fat and low in protein

*C. Avoid milk, especially at bedtime, because it increases gastric secretions*

When admitting a patient with a stroke who is unconscious and unresponsive to stimuli, the nurse learns from the patient's family that the patient has a history of significant gastroesophageal reflux disease (GERD). The nurse will plan to do frequent assessments of the patent's? A. Heart rate and BP B. Bowel sounds and abdominal girth C. Breath sounds and body temperature D. Serum sodium levels

*C. Breath sounds and body temperature*

A patient who has a peptic ulcer reports sudden, severe upper abdominal pain throughout the abdomen and radiating to the back. You notice a rigid abdomen with positive rebound tenderness. What action should you take? A. Continue to monitor, and repeat vital signs in 1 hour. B. Administer the prescribed prn antacid. C. Contact the primary health care provider. D. Position the patient on the left side and encourage slow breaths.

*C. Contact the primary health care provider* Perforation is the most lethal complication of peptic ulcer disease. The classic manifestation of perforation is a sudden, dramatic onset of pain that then spreads. Signs of peritoneal irritation are rigid abdomen and positive rebound tenderness. These findings require a medical evaluation to determine whether surgery or other treatment is needed. The other measures do not appropriately respond to this potential emergency.

When administering prednisone to a patient, the nurse will do what? A. Administer the prednisone in the evening to coincide with the natural secretion pattern of the adrenal cortex. B. Instruct the patient to stop taking the prednisone immediately if diarrhea develops. C. Ensure that meals are at bedside so that administration with food reduces gastric irritation. D. Avoid intravenous delivery to prevent adverse effects.

*C. Ensure that meals are at bedside so that administration with food reduces gastric irritation.* Glucocorticoids given in larger pharmacologic doses for nonendocrine causes produce many adverse effects, including gastric irritation and PEPTIC ULCERS. They should be given with food. Doses should be administered before 9 AM to maximize endocrine function. Prednisone should not be stopped abruptly; the patient should contact the healthcare provider before discontinuing the medication. The drug may be administered by many routes, including the parenteral route.

Achalacia increases the risk for? A. GERD B. Peptic Ulcer Disease C. Esophageal cancer D. stomach cancer

*C. Esophageal cancer* Dilated gullet (esophageal body), usually with a stricture (narrowing) at the bottom (LES). There is a lack of peristalsis in lower 2/3rd of esophagus. Food stays caught there and predisposes to cancer of the esophagus.

Which clinical manifestation is most important for you to monitor in a patient with Cushing disease? A. Periorbital edema B. Pitting pedal edema C. Flu with a temperature of 100.4° F (38° C) D. Blood glucose level of 150 mg

*C. Flu with a temperature of 100.4° F (38° C)* Patients with Cushing disease are immunosuppressed and have a blunted response to infection. Periorbital edema and pitting pedal edema are expected findings due to sodium and water retention from the effects of mineralocorticoids. Elevated glucose levels are an expected finding because of the glucose intolerance associated with cortisol-induced insulin resistance and increased gluconeogenesis by the liver. The glucose level may need treatment, but it is not as important as the potential infection

The nurse is planning care for a patient with signs of acute adrenal insufficiency. What is the priority nursing diagnosis? A. Altered comfort B. Altered nutrition C. Fluid volume deficit D. Activity intolerance

*C. Fluid volume deficit* Acute adrenal insufficiency (adrenal crisis) is characterized by hypotension, dehydration, weakness, lethargy, and gastrointestinal (GI) symptoms of nausea and vomiting. Rapid replacement of fluid, salt, and glucocorticoids is essential to prevent shock and death. Comfort, nutrition, and activity are important to address once fluid balance has been restored.

Which nursing diagnosis should be the priority for a patient who is receiving desmopressin (DDAVP)? A. Activity intolerance B. Alteration in comfort C. Fluid volume imbalance D. Deficient knowledge

*C. Fluid volume imbalance* Desmopressin is a form of antidiuretic hormone that increases sodium and water retention, leading to an alteration in fluid volume. Monitoring of urine volumes and body weights is essential to prevent complications. Alteration in comfort, deficient knowledge of the condition, and activity intolerance are important nursing problems; however, they are not priorities according to the Maslow hierarchy of needs

The most appropriate snack for the nurse to order for a patient with acute kidney injury is: A. Raisins B. Yogurt C. Graham crackers D. Slices of honey dew melon

*C. Graham crackers*

A patient with Addison's disease comes to the emergency department with complaints of nausea, vomiting, diarrhea, and fever. The nurse would expect collaborative care to include? A. parenteral injections of adrenocorticotropic hormone (ACTH). B. IV administration of desmopressin acetate (DDVAP). C. IV administration of hydrocortisone. D. IV administration of D5W with 20 mEq KCl.

*C. IV administration of hydrocortisone* Rationale: Nausea, Vomiting and diarrhea are signs of adrenal crisis. Treatment of a crisis requires immediate glucocorticoid replacement, and IV hydrocortisone, fluids, sodium, and glucose are necessary for 24 hours. WHY NOT A? Addison's disease is a primary insufficiency of the adrenal gland (atrophy of adrenal gland), and so adrenocorticotropic hormone (ACTH) would not be effective in stimulating the release of cortisol, nor would vasopressors be effective with the fluid deficiency of Addison's disease. WHY NOT D? Potassium levels are increased in Addison's disease, and KCl would be contraindicated. The shock, hypotension and dehydration associated with adrenal crisis or acute adrenal insufficiency is not responsive to fluid.

A patient who has undergone an esophagectomy for esophageal cancer develops increasing pain, fever, and dyspnea when a full liquid diet is started postoperatively. You recognize that these symptoms indicate which of the following? A. Intolerance to the feedings B. Extension of the tumor into the aorta C. Leakage of fluid or foods into the mediastinum D. Esophageal perforation with fistula formation into the lung

*C. Leakage of fluid or foods into the mediastinum*

The nurse is caring for a patient admitted with suspected hyperparathyroidism. Because of the potential effects of this disease on electrolyte balance, the nurse should assess this patient for which of the following manifestations? A. Neurologic irritability B. Declining urine output C. Lethargy and weakness D. Hyperactive bowel sounds

*C. Lethargy and weakness* Hyperparathyroidism can cause hypercalcemia. Signs of hypercalcemia include polyuria, constipation, nausea and vomiting, lethargy, and muscle weakness

A female college student goes to the university health clinic complaining of pain that started at the umbilicus and moved to the right lower quadrant over the last 12 hours. You notice muscle guarding on examination. What action should you take? A. Administer a PRN laxative per standing orders. B. Ask about the last menstrual period. C. Make the student NPO. D. Assess bowel sounds.

*C. Make the student NPO.* This is a classic description of appendicitis. At the very least, it is an acute abdomen, and the student should be kept NPO until a need for surgery is ruled out. The student should be referred to an emergency department. A laxative should not be given because it can increase peristalsis and cause perforation. Asking about her last menstrual cycle (possibility of a ruptured ectopic pregnancy) is important but the symptoms suggest appendicitis. Bowel sounds should be assessed, but the NPO status is a priority

A patient with mild iatrogenic Cushing syndrome is on an alternate-day regimen of corticosteroid therapy. The nurse explains to the patient that this regimen? A. Maintains normal adrenal hormone balance. B. Prevents ACTH release from the pituitary gland. C. Minimizes hypothalamic-pituitary-adrenal suppression. D. Provides a more effective therapeutic effect of the drug.

*C. Minimizes hypothalamic-pituitary-adrenal suppression.* Rationale: Taking corticosteroids on an alternate-day schedule for pharmacologic purposes is less likely to suppress ACTH production from the pituitary and prevent adrenal atrophy. Normal adrenal hormone balance is not maintained during glucocorticoid therapy because excessive exogenous hormone is used. *THIS IS LIKELY TO BE ON EXAM. IT COORESPONDS TO THE CUSHINGS' SYNDROME CASE STUDY AND WORKSHEETS

A patient is taking fludrocortisone (Florinef). A nurse should recognize that the patient is at risk for developing an electrolyte imbalance if the patient reports which symptom? A. Syncope B. Weight loss C. Muscle weakness D. Numbness and tingling

*C. Muscle weakness* Muscle weakness is a sign of hypokalemia, which can occur because fludrocortisone has mineralocorticoid properties, resulting in sodium and fluid retention and potassium excretion. Syncope and weight loss do not occur because of salt and water retention. Numbness and tingling may be associated with hypocalcemia but are not related to fludrocortisone.

Which of the following is a nursing priority in the care of a patient with a diagnosis of hypothyroidism? A. Providing a dark, low-stimulation environment B. Closely monitoring the patient's intake and output C. Patient teaching related to levothyroxine (Synthroid) D. Patient teaching related to radioactive iodine therapy

*C. Patient teaching related to levothyroxine (Synthroid)* A euthyroid state is most often achieved in patients with hypothyroidism by the administration of levothyroxine (Synthroid). It is not necessary to carefully monitor intake and output, and low stimulation and radioactive iodine therapy are indicated in the treatment of hyperthyroidism

The patient has chronic gastritis. You should monitor the patient for which potential deficiency? A. Iron B. Scurvy C. Pernicious anemia D. Fat-soluble vitamins

*C. Pernicious anemia*

BOOK: It is especially important to assess for which clinical manifestations in a patient that just underwent a total thyroidectomy? A. weight gain B. depressed reflexes C. Positive Chvostek's sign D. confusion and personality changes

*C. Positive Chvostek's sign* Why not B? Without PTH you would have hypocalcemia and depressed reflexes is a sign of hypercalcemia Why not D? confusion and personality changes are signs of hypercalcemia Sign of hypocalcemia. Total thyroidectomy takes the parathyroid too. No PTH so calcium would be low. Signs of hypocalcemia: -Trousseau's sign (carpal spasms during blood pressure) -Chvostek's sign (contraction of facial muscles when facial nerve on cheek is tapped) -hyperactive reflexes -numbness and tingling in the mouth and lips and extremities, "cats go numb"

A patient has been taking oral prednisone for the past several weeks after having a severe reaction to poison ivy. The nurse has explained the procedure for gradual reduction rather than sudden cessation of the drug. What is the rationale for this approach to drug administration? A. Prevention of hypothyroidism B. Prevention of diabetes insipidus C. Prevention of adrenal insufficiency D. Prevention of cardiovascular complications

*C. Prevention of adrenal insufficiency* (secondary adrenal insufficiency) Sudden cessation of corticosteroid therapy can precipitate life-threatening adrenal insufficiency called adrenal crisis. Diabetes insipidus, hypothyroidism, and cardiovascular complications are not common consequences of stopping corticosteroid therapy suddenly.

The patient is diagnosed with E. coli O157:H7 after reporting sudden, bloody diarrhea. What is most important for you to do? A. Administer IV antibiotics. B. Administer loperamide (Imodium). C. Provide IV fluid replacement. D. Obtain a stool culture.

*C. Provide IV fluid replacement.* Treatment is mainly supportive and includes maintaining intravascular volume. *The use of antibiotics for food borne illness remains controversial.* Most persons recover without antibiotics or other specific treatment. There is no evidence that antibiotics improve the course of the disease, and they may precipitate kidney complications. Antidiarrheal agents should be avoided. There is no need for a stool culture because the diagnosis and organism is already identified.

The patient has SIADH with a serum sodium level of 128 mEq/L. What action do you anticipate? A. Increase sodium-rich foods. B. Rapidly infuse hypertonic intravenous (IV) fluids. C. Restrict fluids. D. Administer calcitonin.

*C. Restrict fluids.* When symptoms of SIADH are mild and the serum sodium level is more than 125 mEq/L, the only treatment may be restriction of fluids to 800 to 1000 mL per day. Severe hyponatremia (less than 120 mEq/L) may be treated with slow infusion of hypertonic saline. Calcitonin is used for hypercalcemia.

What clinical manifestations are common in a patient with stomach cancer? A. Shortness of breath and productive cough B. Constipation C. Unexplained weight loss and anemia D. Fat intolerance

*C. Unexplained weight loss and anemia* Clinical manifestations are unexplained weight loss, anorexia, abdominal discomfort, indigestion, and anemia. Anemia is caused by the chronic blood loss as the lesion erodes through the mucosa or from pernicious anemia. The other options are not specific for stomach cancer.

A nurse is caring for a client who is taking levothyroxine (Synthroid). For which of the following should the nurse monitor to identify levothyroxine toxicity? A. Hair loss B. Lethargy C. Weight loss D. Cold intolerance

*C. Weight loss* Overmedication with levothyroxine may result in toxicity. Findings of toxicity include weight loss despite increased appetite, insomnia, nervousness, and palpitations. Hair loss, lethargy, and cold intolerance are findings seen in the client who have hypothyroidism.

The pernicious anemia that may accompany gastritis is caused by A. chronic autoimmune destruction of cobalamin stores in the body. B. progressive gastric atrophy from chronic breakage in the mucosal barrier and blood loss. C. a lack of intrinsic factor normally produced by acid-secreting cells of the gastric mucosa. D. hyperchlorhydria resulting from an increase in acid-secreting parietal cells and degradation of red blood cells.

*C. a lack of intrinsic factor normally produced by acid-secreting cells of the gastric mucosa.*

Measures indicated in the conservative therapy of CKD include A. decreased fluid intake, carbohydrate intake, and protein intake. B. increased fluid intake; decreased carbohydrate intake and protein intake. C. decreased fluid intake and protein intake; increased carbohydrate intake. D. decreased fluid intake and carbohydrate intake; increased protein intake.

*C. decreased fluid intake and protein intake; increased carbohydrate intake.* Water and any other fluids are not routinely restricted in the pre-end-stage renal disease (ESRD) stages. *Patients on hemodialysis have a more restricted diet than patients receiving peritoneal dialysis*. For those receiving hemodialysis, as their urinary output diminishes, fluid restrictions are enhanced. Intake depends on the daily urine output. Generally, 600 mL (from insensible loss) plus an amount equal to the previous day's urine output is allowed for a patient receiving hemodialysis. Patients are advised to limit fluid intake so that weight gains are no more than 1 to 3 kg between dialyses (interdialytic weight gain). For the patient who is undergoing dialysis, protein is not routinely restricted. The beneficial role of protein restriction in CKD stages 1 through 4 as a means to reduce the decline in kidney function is being studied. Historically, dietary counseling often encouraged restriction of protein for CKD patients. Although there is some evidence that protein restriction has benefits, many patients find these diets difficult to adhere to. For CKD stages 1 through 4, many clinicians encourage a diet with normal protein intake. However, you should teach patients to avoid high-protein diets and supplements because they may overstress the diseased kidneys.

The most critical intervention in the prevention of renal calculi is for the patient to? A. urinate frequently. B. eat a diet high in protein. C. maintain a high fluid intake. D. eliminate all calcium from the diet.

*C. maintain a high fluid intake.* The most important factor in the prevention of renal calculi is keeping urine dilute and free flowing. This reduces the risk of recurrent stone formation in many individuals. This is accomplished by drinking about 2000 to 2200 mL/day, with the residual 20% to 30% of fluids gained through consumption of foods. The volume of fluids is higher in the highly active patient who works outdoors or who regularly engages in demanding athletic activities

In planning care for the patient with Crohn's disease, you recognize that a major difference between ulcerative colitis and Crohn's disease is that Crohn's disease A. frequently results in toxic megacolon. B. causes fewer nutritional deficiencies than does ulcerative colitis. C. often recurs after surgery, whereas ulcerative colitis is curable with a colectomy. D. is manifested by rectal bleeding and anemia more frequently than is ulcerative colitis.

*C. often recurs after surgery, whereas ulcerative colitis is curable with a colectomy*

In planning care for the patient with Crohn's disease, you recognize that a major difference between ulcerative colitis and Crohn's disease is that Crohn's disease A. frequently results in toxic megacolon. B. causes fewer nutritional deficiencies than does ulcerative colitis. C. often recurs after surgery, whereas ulcerative colitis is curable with a colectomy. D. is manifested by rectal bleeding and anemia more frequently than is ulcerative colitis.

*C. often recurs after surgery, whereas ulcerative colitis is curable with a colectomy.* Toxic Megacolon IS relatively more common with Crohn's than with UC but it doesn't frequently occur.

Hemodialysis is contraindicated for which of the following clients? a. Client who can't receive anticoagulants b. Client who is unable to ambulate c. Client who is immunocompromised d. Client who is allergic to iodine

*Client who can't receive anticoagulants*

Skip lesions are characteristic of ___ ?

*Crohn's Disease* (patchy areas of the intestine that show involvement of anywhere from the mouth to the anus) This is why surgery cannot prevent it from recurring like it can with Ulcerative Colitis (UC)

All of the following orders were written for a patient who has vomited 1,500 ml of bright red blood. Which order should the nurse implement first? A. Famotidine (Pepcid) 40 mg IV every 8 hours. B. Phytonadione (Vitamin K), 1 mg IV, one time. C. Draw blood for type and crossmatch. D. Administer 1000 ml of Lactated Ringer's solution IV.

*D. Administer 1000 ml of Lactated Ringer's solution IV*

All of the following orders were written for a patient who has vomited 1,500 ml of bright red blood. Which order should the nurse implement first? A. Famotidine (Pepcid) 40 mg IV every 8 hours. B. Phytonadione (Vitamin K), 1 mg IV, one time. C. Draw blood for type and crossmatch. D. Administer 1000 ml of Lactated Ringer's solution IV.

*D. Administer 1000 ml of Lactated Ringer's solution IV.*

Which alterations occur in IBS? A. Ulceration in the gastric mucosa B. Viral infection C. Protozoal infestation D. Altered bowel motility

*D. Altered bowel motility* *The cause of IBS is unknown*, but altered bowel motility, heightened visceral sensitivity, inflammation, and psychological distress are likely to be involved. *Ulceration is associated with inflammatory bowel disease (IBD).* IBS is not associated with infection or protozoal infestation. *LECTURE: IBS-D=diarrhea, IBS-C=Constipation, IBS-A=alternating* alteration differs between men and women

The nurse who inserted a nasogastric tube for a 68-year-old patient with suspected bowel obstruction should write which of the following priority nursing diagnoses on the patient's problem list? A. Anxiety related to nasogastric tube placement B. Abdominal pain related to nasogastric tube placement C. Risk for deficient knowledge related to nasogastric tube placement D. Altered oral mucous membrane related to nasogastric tube placement

*D. Altered oral mucous membrane related to nasogastric tube placement* With nasogastric tube placement, the patient is likely to breathe through the mouth and may experience irritation in the affected nares. For this reason, the nurse should plan preventive measures based on this nursing diagnosis.

Which nursing intervention is most appropriate to decrease postoperative edema and pain after an inguinal herniorrhaphy? A. Applying a truss to the hernia site B. Allowing the patient to stand to void C. Supporting incision during coughing D. Applying a scrotal support with ice bag

*D. Applying a scrotal support with ice bag*

The nurse would monitor for which of the following adverse changes in the patient's laboratory values as a result of being treated with dexamethasone (Decadron)? A. Sodium 130 mEq/L B. Calcium 8.2 mg/dl C. Potassium 4.9 mEq/L D. Blood glucose 162 mg/dl

*D. Blood glucose 162 mg/dl* Hyperglycemia or increased blood glucose level is an adverse effect of corticosteroid therapy. Increased calcium and potassium excretion are complication as well. The normal range for total calcium is 8.6-10.2 so this calcium level is slightly low??? This potassium level is slightly high (3.5-5) and the sodium level is low ( normal is 135-145) which is not consistent with sodium retention. I think 162 mg/dL for glucose is the best answer because hyperglycemia is the main concern.

Which is the best method for evaluation and treatment of large intestine polyps? A. Sigmoidoscopy B. Barium enema C. Digital examination D. Colonoscopy

*D. Colonoscopy* Colonoscopy is preferred because it allows evaluation of the total colon and polyps can be immediately removed. Only polyps in the distal colon and rectum can be detected and removed during sigmoidoscopy. Polyps can be detected but not removed during barium enema and radiography. Digital examination is used for prostate evaluation but not for the diagnosis of colon polyps because it only assesses the rectal area and not the colon.

What causes the edema that occurs in nephrotic syndrome? A. Increased hydrostatic pressure caused by sodium retention B. Decreased aldosterone secretion from adrenal insufficiency C. Increased fluid retention caused by decreased glomerular filtration D. Decreased colloidal osmotic pressure caused by loss of serum albumin Rationale

*D. Decreased colloidal osmotic pressure caused by loss of serum albumin* The increased glomerular membrane permeability found in nephrotic syndrome is responsible for the massive excretion of protein in the urine. This results in a decreased serum protein level and subsequent edema formation. Ascites and anasarca (massive generalized edema) develop if there is severe hypoalbuminemia.

Which discharge teaching should you provide to a patient with a herniorrhaphy for an inguinal hernia repair? A. Wear a truss continually. B. Call the primary provider if scrotal edema occurs. C. Cough frequently to prevent atelectasis. D. Do not do heavy lifting for 6 to 8 weeks.

*D. Do not do heavy lifting for 6 to 8 weeks.*

What is important to teach a patient with gastroesophageal reflux disease (GERD)? A. Drink whole milk before going to bed. B. Lie down and rest 30 minutes after meals. C. Avoid spicy foods. D. Elevate head of the bed on 6-inch blocks.

*D. Elevate head of the bed on 6-inch blocks.* The head of the bed should be elevated to approximately 30 degrees (4- to 6-inch blocks) to prevent reflux through a weakened lower esophageal sphincter during the night. Milk (especially whole milk with higher fat) should be avoided at bedtime because it increases gastric acid secretion. To prevent reflux, patients should not be supine for 2-3 hours after eating. Spicy foods are not particularly related to GERD. The patient should avoid foods known to cause reflux. These include foods that decrease lower esophageal sphincter (LES) pressure such as fatty foods, chocolate, peppermint, tomatoes, coffee, tea, and foods that irritate the esophagus, such as tomato-based products, orange juice, cola, and red wine.

The nurse is caring for a 68-year-old patient admitted with abdominal pain, nausea, and vomiting. The patient has an abdominal mass and a bowel obstruction is suspected. The nurse auscultating the abdomen listens for which of the following types of bowel sounds that is consistent with the patient's clinical picture? A. Low pitched and rumbling above the area of obstruction B. High pitched and hypoactive below the area of obstruction C. Low pitched and hyperactive below the area of obstruction D. High pitched and hyperactive above the area of obstruction

*D. High pitched and hyperactive above the area of obstruction* Early in intestinal obstruction, the patient's bowel sounds are hyperactive and high pitched, sometimes referred to as "tinkling" above the level of the obstruction. This occurs because peristaltic action increases to "push past" the area of obstruction. As the obstruction becomes complete, bowel sounds decrease and finally become absent.

The nurse is developing a plan to minimize the risk of adrenal insufficiency for a patient who is receiving long-term glucocorticoid therapy. Which outcome should be included? A. Increases daily intake of sodium for vascular expansion B. Tapers blood pressure medications to avoid hypotension C. Obtains periodic ultrasound scans of adrenal glands D. Increases or supplements dosage of glucocorticoid at times of stress

*D. Increases or supplements dosage of glucocorticoid at times of stress* Exogenous steroids inhibit the synthesis and release of endogenous steroids by the adrenals, and recovery is variable, taking from days to a year. Failure to increase or supplement doses at times of stress may be life-threatening. Increasing sodium intake and tapering blood pressure medications could cause harm. Adrenal gland ultrasound scans are not a valid way to minimize adrenal insufficiency.

You are conducting a community education session. Which option is correct information related to diverticulosis? A. It is commonly seen in young people. B. The classic presentation is right-sided abdominal pain. C. Adequate protein can prevent its occurrence. D. It is a result of aging and decreased stool size.

*D. It is a result of aging and decreased stool size.* Although the exact cause is unknown, diverticulosis is thought to result from high intraluminal pressure on weakened areas of the bowel, often resulting from inadequate fiber. The decreased stool raises intraluminal pressure. Diverticular disease is common, and the *incidence increases with age*. In Western civilization, the classic anatomic location of diverticulum is in the sigmoid colon or left-sided abdomen. *It usually is asymptomatic unless inflammation (diverticulitis) is present*. Preventive measures include increasing the bulk in diet (fresh fruits and vegetables) and decreasing intake of red meats and fats.

You are answering a patient's questions about celiac or gluten-sensitive enteropathy disease. Which option is the correct information to provide? A. Celiac is also known as tropical sprue. B. Celiac is only seen in children. C. Its symptoms mimic inflammatory bowel disease (IBD). D. It is an autoimmune disease.

*D. It is an autoimmune disease.* *Celiac is an autoimmune disease in people who have a genetic predisposition*, consume gluten, and an immune-mediated response. Celiac disease is different from tropical sprue, which is a chronic disorder acquired in tropical areas and treated with folic acid and tetracycline. Celiac disease is a relatively common disease that occurs in all ages. The symptoms mimic irritable bowel syndrome (IBS).

During the assessment of a patient with acute abdominal pain, what should you do? A. Perform deep palpation before auscultation. B. Obtain blood pressure and pulse rate to determine hypervolemic changes. C. Auscultate bowel sounds because hyperactive bowel sounds suggest paralytic ileus. D. Measure body temperature because an elevated temperature may indicate an inflammatory or infectious process.

*D. Measure body temperature because an elevated temperature may indicate an inflammatory or infectious process.* For the patient complaining of acute abdominal pain, you should take vital signs immediately. Increased pulse and decreasing blood pressure are indicative of hypovolemia. An elevated temperature suggests an inflammatory or infectious process. Intake and output measurements provide essential information about the adequacy of vascular volume. Inspect the abdomen first, and then auscultate bowel sounds. Palpation is performed next and should be gentle.

The nurse would question the use of which of the following cathartic agents in a patient with renal insufficiency? A. Bisacodyl B. Lubiprostone C. Cascara sagrada D. Milk of magnesia

*D. Milk of magnesia* Milk of magnesia *may cause hypermagnesemia* in patients with renal insufficiency. The nurse should question this order with the health care provider before administration.

A nurse monitors the calcium level of a patient who has had a thyroidectomy. The nurse should notice which finding if the patient's calcium level is 7.5 mg/dL? A. Dull, aching bone pain B. Nausea and vomiting C. Lethargy and confusion D. Muscle twitching and tetany

*D. Muscle twitching and tetany* normal range for total serum calcium is 8.6-10.2 mg/dL. Low calcium levels may be the result of inadvertent removal of the parathyroid gland during a thyroidectomy, leading to a lack of PTH. This produces hypocalcemia and symptoms of tetany, muscle twitching, and neuromuscular excitability. Nausea and vomiting, lethargy, and confusion are symptoms of HYPERcalcemia. Dull, aching bone pain may be associated with osteomalacia and vitamin D insufficiency.

Sevelamer (Renagel) is a______binder medication? A. Calcium B. Magnesium C. Postassium D. Phosphate

*D. Phosphate* Treats the hyperphosphatemia/hypercalcemia that occurs with kidney failure

The nurse preparing to administer a dose of Calcium acetate (PhosLo) to a patient with chronic kidney disease would interpret that this medication should have a beneficial effect on which of the following laboratory values of the patient? A. Sodium B. Potassium C. Magnesium D. Phosphorus

*D. Phosphorus* Phosphorus and calcium have inverse or reciprocal relationships, meaning that when phosphorus levels are high, calcium levels tend to be low. Therefore administration of calcium should help to reduce a patient's abnormally high phosphorus level, as seen with chronic kidney disease.

A patient takes glucocorticoids and digoxin (Lanoxin). It is most important for the nurse to monitor which electrolyte? A. Calcium B. Magnesium C. Sodium D. Potassium

*D. Potassium* Because of the mineralocorticoid activity of sodium and water retention and potassium loss, glucocorticoids can increase the risk of toxicity from digoxin. They also can exacerbate hypokalemia caused by thiazide and loop diuretics. Calcium, magnesium, and sodium do not require more frequent monitoring as a result of digoxin and glucocorticoids use.

When caring for a patient during the oliguric phase of acute kidney injury, which of the following would be an appropriate nursing intervention? A. Weigh patient three times weekly. B. Increase dietary sodium and potassium. C. Provide a low-protein, high-carbohydrate diet. D. Restrict fluids according to previous daily loss.

*D. Restrict fluids according to previous daily loss.* Patients in the oliguric phase of acute kidney injury will have fluid volume excess with potassium and sodium retention; hence, they will need to have dietary sodium, potassium, and fluids restricted. Daily fluid intake is based on the previous 24-hour fluid loss (measured output plus 600 ml for insensible loss). The diet also needs to provide adequate, not low, protein intake to prevent catabolism. The patient should also be weighed daily, not just three times a week

When caring for a patient during the oliguric phase of acute kidney injury, what would be an appropriate nursing intervention? A. Weigh patient three times weekly B. Increase dietary sodium and potassium C. Provide a low-protein, high-carbohydrate diet D. Restrict fluids according to the previous day's fluid loss

*D. Restrict fluids according to the previous day's fluid loss* Patients in the oliguric phase of acute kidney injury have fluid volume excess with potassium and sodium retention. They will need to have dietary sodium, potassium, and fluids restricted. Daily fluid intake is based on the previous 24-hour fluid loss (measured output plus 600 mL for insensible loss). *The diet also needs to provide adequate, not low, protein intake to prevent catabolism*. The patient should also be weighed daily, not just three times per week.

A client is starting on levothyroxine (Synthroid) for long-standing hypothyroidism. Which of the following dosage schedules should the nurse expect to administer? A. The client will start at a high dose and the dose will be tapered down as needed. B. The client's initial dosage will be based on body weight. C. The clients dosage will be adjusted daily based on blood levels. D. The client will start on a low dose, which will be gradually increased.

*D. The client will start on a low dose, which will be gradually increased.* This client will be very sensitive to thyroid replacement medications. Therapy should start at low doses and be increased gradually. None of the other dosage schedules are appropriate. "Start low and go slow with levo"

What is a clinical manifestation of hypoparathyroidism? A. Exophthalmos B. Purple striae on abdomen and thighs C. Pernicious anemia D. Tingling in lips and fingertips

*D. Tingling in lips and fingertips* The clinical manifestations of hypoparathyroidism result from hypocalcemia. They can include tetany, characterized by tingling of the lips and stiffness of the extremities, Exophthalmos is caused by hyperthyroidism. Truncal obesity with striae is related to a decreased collagen level found in people with Cushing disease. Pernicious anemia is related to cobalamin (vitamin B12) deficiency.

The patient admitted to the intensive care unit after a motor vehicle accident has been diagnosed with AKI. Which finding indicates the onset of oliguria resulting from AKI? A. Urine output less than 1000 mL for the past 24 hours B. Urine output less than 800 mL for the past 24 hours C. Urine output less than 600 mL for the past 24 hours D. Urine output less than 400 mL for the past 24 hours

*D. Urine output less than 400 mL for the past 24 hours* The most common initial manifestation of AKI is oliguria, a reduction to urine output to less than 400 mL/day. *or less than 30 mL/hr*

LEHNE READING: A patient who has diabetes insipidus is receiving desmopressin (DDAVP). Which laboratory test should a nurse obtain to evaluate the effectiveness of the medication? A. Urine ketones B. Blood urea nitrogen (BUN) C. Creatinine D. Urine specific gravity

*D. Urine specific gravity* Diabetes insipidus is characterized by a decrease in the urine specific gravity because of the excretion of large volumes of dilute urine. Desmopressin acts to prevent fluid loss through the renal tubules and increases the urine specific gravity. Urine ketones are present in type 1 diabetes mellitus. The BUN and creatinine are indicators of renal function but not of the effectiveness of treatment of hypothalamic diabetes insipidus.

The most accurate indicator of fluid loss or gain in an acutely ill patient is: A. Blood pressure B. The presence or absence of peripheral edema C. Serum sodium levels D. Weight change

*D. Weight change*

A renal stone in the pelvis of the kidney will alter the function of the kidney by interfering with the? A. structural support of the kidney. B. regulation of the concentration of urine. C. entry and exit of blood vessels at the kidney. D. collection and drainage of urine from the kidney.

*D. collection and drainage of urine from the kidney.* The outer covering of the kidney is the renal capsule. The inside of the kidney has two parts: the outer cortex and the inner medulla. The medulla consists of medullary pyramids, structures so named because of their shape. The apices (tops) of these pyramids are called papillae, through which urine passes to enter the calyces. The minor calyces widen and merge to form major calyces, which form a funnel-shaped sac called the renal pelvis. The minor and major calyces transport urine to the renal pelvis, from which it drains through the ureter to the bladder.

In a patient with an elevated serum cortisol, the nurse would expect other laboratory findings to reveal A. Hyponatremia. B. Hypoglycemia. C. Decreased serum triglycerides. D. Hypokalemia.

*D. hypokalemia.* Aldosterone release would cause sodium and water retention and potassium and calcium excretion. The break down of glycogen would result in hyperglycemia and the breakdown of fat would increase serum triglycerides.

In a patient with central diabetes insipidus, administration of ADH during a water deprivation test will result in a (n)? a. decrease in body weight. b. increase in urinary output. c. decrease in blood pressure. d. increase in urine osmolality

*D. increase in urine osmolality* Rationale: A patient with diabetes insipidus has a deficiency of ADH with excessive loss of water from the kidney, hypovolemia, hypernatremia, and dilute urine with a low specific gravity. When vasopressin is administered, the symptoms are reversed, with water retention, decreased urinary output that increases urine osmolality, and an increase in BP

During the assessment of a patient with acute abdominal pain, the nurse should? A. perform deep palpation before auscultation. B. obtain blood pressure and pulse rate to determine hypervolemic changes. C. auscultate bowel sounds because hyperactive bowel sounds suggest paralytic ileus. D. measure body temperature because an elevated temperature may indicate an inflammatory or infectious process.

*D. measure body temperature because an elevated temperature may indicate an inflammatory or infectious process.* A-never palpate before auscultating B-HYPO-volemic, why BP and HR are important with acute abdominal pain C-hypoactive suggest paralytic ileus *For the patient complaining of acute abdominal pain, you should take vital signs immediately. Increased pulse and decreasing blood pressure are indicative of hypovolemia. An elevated temperature suggests an inflammatory or infectious process. Intake and output measurements provide essential information about the adequacy of vascular volume. Inspect the abdomen first, and then auscultate bowel sounds. Palpation is performed next and should be gentle.*

You explain to the patient with gastroesophageal reflux disease that this disorder A. results in acid erosion and ulceration of the esophagus caused by the frequent vomiting. B. will require surgical wrapping or repair of the pyloric sphincter to control the symptoms. C. is the protrusion of a portion of the stomach into the esophagus through an opening in the diaphragm. D. often involves relaxation of the lower esophageal sphincter, allowing stomach contents to back up into the esophagus.

*D. often involves relaxation of the lower esophageal sphincter, allowing stomach contents to back up into the esophagus.*

The nurse determines that the patient with oliguria has prerenal oliguria when: A. urinalysis reveals a low specific gravity B. urinalysis reveals casts C. the fractional excretion of sodium (FENa) is high D. the oliguria is reversed with fluid replacement.

*D. the oliguria is reversed with fluid replacement.* A-low specific gravity would mean there is a lot of extra water, which would occur with prerenal AKI. B-casts would indicate internal AKI due to damage of glomeruli C-a high FENa would indicate intrarenal, whereas a low FENa prerenal D-prerenal is hypoperfusion of the kidneys and is treated with IV fluids and drugs to maximize cardiac perfusion.

The nurse determines that the patient with oliguria has prerenal oliguria when? A. urinalysis reveals a low specific gravity B. urinalysis reveals casts C. the fractional excretion of sodium (FENa) is high D. the oliguria is reversed with fluid replacement.

*D. the oliguria is reversed with fluid replacement.* This is different than the question above. Here there is hypovolemia and in the above question there is hypervolemia. Since there is low blood volume and aldosterone is retaining sodium and water there would not be a lot of extra water in this urine.

You explain to the patient with Vincent's infection that treatment will include A. smallpox vaccinations. B. viscous lidocaine rinses. C. Amphotericin B suspension. D. topical application of antibiotics.

*D. topical application of antibiotics.* Vincent's infection is treated with topical applications of antibiotics. Other treatments include rest (physical and mental); avoidance of smoking and alcoholic beverages; a soft, nutritious diet; correct oral hygiene habits; and mouth irrigations with hydrogen peroxide and saline solutions.

*DIALYSIS*

*DIALYSIS*

A nurse is performing an admission assessment of a client who has acute glomerulonephritis. The nurse should expect which of the following findings? a. Low blood pressure b. Polyuria c. Dark-colored urine d. Weight loss

*Dark-colored urine*

The edema that occurs in nephrotic syndrome is due to? a. Increased hydrostatic pressure caused by sodium retention b. Decreased aldosterone secretion from adrenal insufficiency c. Increased fluid retention caused by decreased glomerular filtration d. Decreased colloidal osmotic pressure caused by loss of serum albumin

*Decreased colloidal osmotic pressure caused by loss of serum albumin*

Measures indicated in the conservative therapy of CKD include? a. Decreased fluid intake, carbohydrate intake, and protein intake b. Increased fluid intake, decreased carbohydrate intake and protein intake c. Decreased fluid intake and protein intake, increased carbohydrate intake d. Decreased fluid intake and carbohydrate intake, increased protein intake

*Decreased fluid intake and protein intake, increased carbohydrate intake*

What is the treatment for hyperphosphatemia?

*Dietary phosphate restrictions* and phosphate binder medications taken with all meals. An example is *sevelamer (Renagel)* -or- *fosrenol (lanthanum carbonate powder)*

The nurse would increase comfort of the patient with appendicitis by? a. Having the patient lie prone b. Flexing the patient's right knee c. Sitting the patient up in a chair d. Turning the patient onto his/her left side

*Flexing the patient's right knee*

An elderly 85-year-old woman has been nauseated all day and has vomited three times. Based on this data, what intervention should be taken? a. Administer antispasmodic drugs and observe skin turgor b. Give sips of water and elevate the head of the bed to prevent aspiration c. Offer a high-protein liquid supplement to drink to maintain nutritional needs d. Offer large quantities of Gatorade to drink because elderly people are at risk for sodium depletion

*Give sips of water and elevate the head of the bed to prevent aspiration*

Why is the metabolic acidosis that occurs in the oliguric phase of renal failure associated with hyperkalemia? What does this mean in terms of treatment?

*H+ ions enter cells in exchange for K+ ions*, raising serum K+. This means that *you can treat hyperkalemia by administering sodium bicarbonate*, which treats the acidosis and reduces the H+ ions available to be exchanged. Insulin pushes K+ into the cells.

A nurse is caring for a client who has chronic kidney failure and the following lab results: BUN 196 mg/dL, sodium 152 mEq/L, and potassium 7.3 mEq/L. Which of the following interventions should the nurse implement? a. Initiate an IV infusion of 0.9% sodium chloride b. Give oral spironolactone c. Infuse regular insulin in dextrose 10% in water d. Administer furosemide

*Infuse regular insulin in dextrose 10% in water* **diuretics ineffective in CK failure

A nurse is obtaining a voided urine culture and sensitivity for a client who has manifestations of a UTI. Which of the following actions should the nurse take? a. Collect the client's urine in a clean specimen container b. Instruct the client to initiate flow of urine before collecting specimen c. Obtain the client's first morning voiding on the following day d. Place the client's urine specimen in a container with a preservative

*Instruct the client to initiate flow of urine before collecting specimen*

To assess the patency of a newly placed arteriovenous graft for dialysis, the nurse should? a. Irrigate the graft daily with low-dose heparin b. Monitor for any increase in BP in the affected arm c. Listen with a stethoscope over the graft for the presence of a bruit d. Frequently monitor the pulses and neurovascular status distal to the graft

*Listen with a stethoscope over the graft for the presence of a bruit*

Upper GI bleeding that is esophogeal in origin can be caused by a ______ -________ tear?

*Mallory-Weiss Tear* Mallory-Weiss Tear: collagen tears in lower esophogeal-ascoiated with hyperemesis and upper GI bleeds Espohgeal varices, chronic esophagitis and mallory weiss tears are the cause of upper GI bleeding of esophageal origin

During the assessment of a patient with acute abdominal pain, the nurse should? a. Perform deep palpation before auscultation b. Obtain blood pressure and pulse rate to determine hypervolemic changes c. Auscultate bowel sounds because hyperactive bowel sounds suggest paralytic ileus d. Measure body temperature because an elevated temperature may indicate an inflammatory or infectious process

*Measure body temperature because an elevated temperature may indicate an inflammatory or infectious process*

Patients in renal failure do not show symptoms of hypocalcemia because they are in ________ ?

*Metabolic Acidosis* ABG: pH is less than 7.35 (7.35-7.45 is normal) CO2 is 35-45 (normal) HCO3- is <22 (22-26 is normal so this is low) ROME=Respiratory Opposite Metabolic Equal pH is LOW, and HCO3 is LOW (METABOLIC EQUAL). pH LOW=ACIDOSIS

What are the symptoms of hypermagnesemia (>2.4)?

*Muscular weakness, lethargy, depressed deep tendon reflexes, hypotension, bradycardia, cardiac arrest (from low output)* *remember the symptoms of magnesium sulfate toxicity, which treats seizures in woman with pre-ecplampsia?-low heart rate, low blood pressure and depressed deep tendon reflexes* *these are also similar to the symptoms of HYPERcalcemia, which are depression, lethargy, muscle weakness*

In contrast to diverticulitis, the patient with diverticulosis a. Has rectal bleeding b. Often has no symptoms c. Has localized cramping pain d. Frequently develops peritonitis

*Often has no symptoms*

The nurse explains to the patient with gastroesophageal reflux disease that this disorder? a. Results in acid erosion and ulceration of the esophagus caused by frequent vomiting b. Will require surgical wrapping or repair of the pyloric sphincter to control the symptoms c. Is the protrusion of a portion of the stomach into the esophagus through an opening in the diaphragm d. Often involves relaxation of the lower esophageal sphincter, allowing stomach contents to back up into the esophagus

*Often involves relaxation of the lower esophageal sphincter, allowing stomach contents to back up into the esophagus*

A major difference between ulcerative colitis and Crohn's disease is that Crohn's disease? a. Frequently results in toxic megacolon b. Causes fewer nutritional deficiencies than does ulcerative colitis c. Often recurs after surgery, whereas ulcerative colitis is curable with a colectomy d. Is manifested by rectal bleeding and anemia more frequently than is ulcerative colitis

*Often recurs after surgery, whereas ulcerative colitis is curable with a colectomy*

What is the difference between primary and secondary adrenal insufficiency?

*Primary adrenal insufficiency is called Addison's Disease and it results from insufficient secretion of hormones despite normal or elevated ACTH levels. Addison's is an autoimmune atrophy of the adrenal gland. *Secondary adrenal insufficiency results from inadequate release of ACTH from the pituitary, usually as a result of corticosteroid therapy withdrawal. During therapy HPA suppression causes the pituitary to stop releasing ACTH by way of negative feedback from exogenous corticosteroid. This causes the Adrenal gland to atrophy and stop releasing cortisol as well. If therapy is withdrawn too fast life-threatening adrenal crisis can result.

The nurse is teaching the patient and family about possible causative factors for peptic ulcers. The nurse explains that ulcer formation is? a. Caused by a stressful lifestyle and other acid-producing factors such as H. pylori b. Inherited within families and reinforced by bacterial spread of Staphylococcus aureus in childhood c. Promoted by factors that tend to cause oversecretion of acid, such as excess dietary fats, smoking, and H. pylori d. Promoted by a combination of possible factors that may result in erosion of the gastric mucosa, including certain drugs and alcohol

*Promoted by a combination of possible factors that may result in erosion of the gastric mucosa, including certain drugs and alcohol*

Misoprostol (cytotec) is a ____ analog that decreases gastric acid secretion and increases mucous production. It is used to prevent ulcers induced by ______ ? Its main side effects are diarrhea and abdominal cramps.

*Prostaglandin, NSAIDS and aspirin* NSAIDS inhibit prostaglandins, which protect the stomach by increasing mucous production and decreasing acid

What is the collaborative care for prerenal failure?

*Quickly restore the circulating volume* using IV fluids since hypoperfusion is the cause. Maximize cardiac output by increasing preload and contractility and minimizing afterload. CO=SV X HR, SV=preload+Afterload+contractility

Which of the following findings places a client at risk for seizures? a. Hypokalemia b. Rapid increase of catecholamines c. Rapid decrease in fluid d. Hypercalcemia

*Rapid decrease in fluid*

The nurse determines that the goals of dietary teaching have been met when the patient with celiac disease selects from the menu? a. Scrambled eggs and sausage b. Buckwheat pancakes with syrup c. Oatmeal, skim milk, and orange juice d. Yogurt, strawberries, and rye toast with butter

*Scrambled eggs and sausage*

Which of the following lab findings of a client with AKI should be reported to the provider? a. Serum potassium 5.0 mEq/L b. Serum calcium 9.0 mg/dL c. Serum creatinine 4.0 mg/dL d. Serum amylast 84 IU/L

*Serum creatinine 4.0 mg/dL*

Using RIFLE, the three stages of acute kidney injury are defined based on changes in? a. Blood pressure and urine osmolality b. Fractional excretion of urinary sodium c. Estimation of GFR with the MDRD equation d. Serum creatinine or urine output from baseline

*Serum creatinine or urine output from baseline*

One major advantage of peritoneal dialysis is? a. No medications are required because of the enhanced efficiency of the peritoneal membrane in removing toxins b. The diet is less restricted and dialysis can be performed at home c. The dialysate is biocompatible and causes no long-term consequences d. High glucose concentrations of the dialysate cause a reduction in appetite promoting weight loss

*The diet is less restricted and dialysis can be performed at home*

Is melena a sign of upper or lower GI bleeding?

*Upper. Melena refers to black tarry stools that indicate an upper GI bleed.*

Which of the following client findings indicates a possible delay in kidney function of a newly transplanted kidney? a. Blood pressure 110/55 b. Incisional tenderness c. Pink and bloody urine d. Urine output 30mL/2 hours

*Urine output 30mL/2 hours*

Which of the following should a patient be taught after a hemorrhoidectomy? a. Take mineral oil prior to bedtime b. Eat a low-fiber diet to rest the colon c. Administer oil-retention enemas to empty the colon d. Use prescribed pain medication before a bowel movement

*Use prescribed pain medication before a bowel movement*

What additional medications would you recommend to a patient taking corticosteroids?

*Vitamin D and Calcium to prevent osteoporosis and proton pump inhibitors and histamine H2 blockers to prevent gastric ulcers*

Indicate whether the following characteristics are associated with peritoneal dialysis (PD) or hemodialysis (HD). a. Requires vascular access b. Increased hyperlipidemia c. Lowers serum triglycerides d. Portable system e. Less cardiovascular stress f. More protein loss g. Intensifies anemia h. Rapid fluid and creatinine loss i. Requires fewer dietary restriction

*a-HD b-PD c-HD d-PD e-PD f-PD g-HD h-HD i-PD*

A post-operative client is experiencing acute adrenal insufficiency. Which of the following actions should the nurse take? a. Administer IV hydrocortisone b. Give oral spironolactone c. Infuse 1 unit of platelets d. Restrict daily fluid intake

*a. Administer IV hydrocortisone sodium succinate*

Hemodialysis is contraindicated for which of the following clients? a. Client who can't receive anticoagulants b. Client who is unable to ambulate c. Client who is immunocompromised d. Client who is allergic to iodine

*a. Client who can't receive anticoagulants* Heparin is added to the blood as it enters the machine to prevent it from clotting. Protamine is an antidote given afterwards.

To control the side effects of corticosteroid therapy, the nurse teaches the patient who is taking corticosteroids to? a. Increase calcium intake to 1500 mg/day b. Perform glucose monitoring for hypoglycemia c. Obtain immunizations due to high risk of infections d. Avoid abrupt position changes because of orthostatic hypotension

*a. Increase calcium intake to 1500 mg/day* Osteoporosis is a risk due to excretion of calcium Do not use live virus vaccinations/reactions to skin test may be compromised

The nurse is reviewing a patient's morning lab results. Which of these results is of highest concern? a. Serum K+ of 2.8 mEq/L b. Serum Na+ of 150 mEq/L c. Serum Mg++ of 1.1 mEq/L d. Serum Ca++ (total) of 8.6 mg/dL

*a. Serum K+ of 2.8 mEq/L* Rationale: With a low potassium (normal is 3.5-5) there is an increased risk for dysrhythmias and severe muscle weakness. The sodium and magnesium levels are also not within normal limits. However, the implications are not as life-threatening. The calcium level is normal. The range for total calcium is 8.6-10.2. Some sites say 8.5-10.2

Regardless of the precipitating factor, the injury to mucosal cells in peptic ulcers is caused by? a. acid back-diffusion into the mucosa. b. the release of histamine from GI cells. c. ammonia formation in the mucosal wall. d. breakdown of the gastric mucosal barrie

*a. acid back-diffusion into the mucosa.*

Metabolic acidosis occurs in the oliguric phase of AKI as a result of impaired a. ammonia synthesis. b. excretion of sodium. c. excretion of bicarbonate. d. conservation of potassium.

*a. ammonia synthesis.*

Two effects of hypokalemia on the endocrine system are a. decreased insulin and aldosterone release. b. decreased glucagon and increased cortisol release. c. decreased release of atrial natriuretic factor and increased ADH release. d. decreased release of parathyroid hormone and increased calcitonin release.

*a. decreased insulin and aldosterone release.* Rationale: Aldosterone retains sodium and secretes potassium. Hypokalemia inhibits aldosterone release as well as insulin release; these are the major hormones affected by hypokalemia. *remember from lecture how insulin affects hyper-kalemia and metabolic acidosis

8-Two effects of hypokalemia on the endocrine system are? a. decreased insulin and aldosterone release. b. decreased glucagon and increased cortisol release. c. decreased release of atrial natriuretic factor and increased ADH release. d. decreased release of parathyroid hormone and increased calcitonin release.

*a. decreased insulin and aldosterone release.* both cause hypokalemia-aldosterone wastes

The nurse evaluates that management of the patient with upper GI bleeding is effective when assessment and laboratory findings reveal a? a. decreasing blood urea nitrogen (BUN). b. hematocrit (Hct) of 35%. c. urinary output of 20 mL/hr. d. urine-specific gravity of 1.030.

*a. decreasing blood urea nitrogen (BUN).*

A patient with end-stage renal failure is scheduled for hemodialysis following healing of an AV fistula. The nurse explains that during dialysis? a. he will be able to visit, read, sleep, or watch TV while reclining in a chair. b. he will be placed on a cardiac monitor to detect any adverse effects that might occur. c. the dialyzer will remove and hold part of his blood for 20 to 30 minutes to remove the waste products. d. a large catheter with two lumens will be inserted into the fistula to send blood to and return it from the dialyzer

*a. he will be able to visit, read, sleep, or watch TV while reclining in a chair.* a. Rationale: While patients are undergoing HD, they can perform quiet activities that do not require the limb that has the vascular access. BP is monitored frequently, and the dialyzer monitors dialysis function, but cardiac monitoring is not indicated. The HD machine continuously circulates both the blood and the dialysate past the semipermeable membrane in the machine. Graft and fistula access involve the insertion of two needles into the site to remove blood from and return blood to the dialyzer.

In a patient with an elevated serum cortisol, the nurse would expect other laboratory findings to reveal? a. hypokalemia. b. hyponatremia. c. hypoglycemia. d. decreased serum triglycerides.

*a. hypokalemia* a. Rationale: The mineralocorticoid effects of cortisol causes sodium retention and potassium excretion from the kidney, resulting in hypokalemia. Because water is reabsorbed with the sodium, serum sodium remains normal. In its effect on glucose and fat metabolism, cortisol causes an elevation in blood glucose as well as increases in free fatty acids and triglycerides.

Priority Decision: A patient is admitted to the emergency department with acute abdominal pain. The nursing intervention that should be implemented first is? a. measurement of vital signs. b. administration of prescribed analgesics. c. assessment of the onset, location, intensity, duration, and character of the pain. d. physical assessment of the abdomen for distention, masses, abnormal pulsations, bowel sounds, and pigmentation changes

*a. measurement of vital signs.* STUDY GUIDE QUESTION VERY SIMILAR TO LECTURE QUESTION

During an acute attack of diverticulitis, the patient is? a. monitored for signs of peritonitis. b. treated with daily medicated enemas. c. prepared for surgery to resect the involved colon. d. provided with a heating pad to apply to the left lower quadrant

*a. monitored for signs of peritonitis.* *why not B?* this would increase pressure and cause perforation *why not C?* Surgery is only required for significant perforations, abscesses and obstruction. *why not D?* this would increase blood flow and pressure and cause perforation

A patient is admitted to the hospital with chronic kidney disease. The nurse understands that this condition is characterized by? a. progressive irreversible destruction of the kidneys. b. a rapid decrease in urinary output with an elevated BUN. c. an increasing creatinine clearance with a decrease in urinary output. d. prostration, somnolence, and confusion with coma and imminent death.

*a. progressive irreversible destruction of the kidneys.*

One of the most important roles of the nurse in relation to acute poststreptococcal glomerulonephritis is to? a. promote early diagnosis and treatment of sore throats and skin lesions. b. encourage patients to request antibiotic therapy for all upper respiratory infections. c. teach patients with APSGN that long-term prophylactic antibiotic therapy is necessary to prevent recurrence. d. monitor patients for respiratory symptoms that indicate that the disease is affecting the alveolar basement membrane.

*a. promote early diagnosis and treatment of sore throats and skin lesions.*

The nurse determines that the goals of dietary teaching have been met when the patient with celiac disease selects from the menu? a. scrambled eggs and sausage. b. buckwheat pancakes with syrup. c. oatmeal, skim milk, and orange juice. d. yogurt, strawberries, and rye toast with butter.

*a. scrambled eggs and sausage.*

A nurse is providing discharge teaching for a client who has CKD. Which of the following statements by the client indicates an understanding of the teaching? a. "I will consume foods high in protein" b. "I will decrease my intake of foods high in phosphorus" c. "I will limit my intake of foods high in calcium" d. "I will add salt to the foods I consume

*b. "I will decrease my intake of foods high in phosphorus"*

During discharge teaching for the patient with Addison's disease, the nurse identifies a need for additional instruction when the patient says, a. "I should always call the doctor if I develop vomiting or diarrhea." b. "If my weight goes down, my dosage of steroid is probably too high." c. "I should double or triple my steroid dose if I undergo rigorous physical exercise." d. "I need to carry an emergency kit with injectable hydrocortisone in case I can't take my medication by mouth."

*b. "If my weight goes down, my dosage of steroid is probably too high.* "b. Rationale: A weight reduction in the patient with Addison's disease may indicate a fluid loss and a dose of replacement therapy that is too low rather than too high. Patients with Addison's disease are taught to take two to three times their usual dose of steroids if they become ill, have teeth extracted, or engage in rigorous physical activity and should always have injectable hydrocortisone available if oral doses cannot be taken. Because vomiting and diarrhea are early signs of crisis and because fluid and electrolytes must be replaced, patients should notify their health care provider if these symptoms occur.

*The normal response to increased serum osmolality is the release of?* a. aldosterone from the adrenal cortex, which stimulates sodium excretion by the kidney. b. ADH from the posterior pituitary gland, which stimulates the kidney to reabsorb water. c. mineralocorticoids from the adrenal gland, which stimulate the kidney to excrete potassium. d. calcitonin from the thyroid gland, which increases bone resorption and decreases serum calcium levels.

*b. ADH from the posterior pituitary gland, which stimulates the kidney to reabsorb water.*

When assessing a client diagnosed with DI, the nurse should expect which of the following laboratory findings? a. Decreased heart rate b. Increased hematocrit c. High urine specific gravity d. Decreased BUN

*b. Increased hematocrit* (this causes the serum osmolality to increase, making the blood more viscous since it has less water in it!)

A patient with a head injury develops SIADH. Symptoms the nurse would expect to find include a. Hypernatremia and edema b. Low urinary output and thirst c. Muscle spasticity and hypertension d. Weight gain and decreased glomerular filtration rate

*b. Low urinary output and thirst*

The most common form of malabsorption syndrome is treated with? a. administration of antibiotics. b. avoidance of milk and milk products. c. supplementation with pancreatic enzymes. d. avoidance of gluten found in wheat, barley, oats, and rye.

*b. avoidance of milk and milk products.*

The nurse expects a patient with an ulcer of the posterior portion of the duodenum to experience? a. pain that occurs after not eating all day. b. back pain that occurs 2 to 4 hours following meals. c. midepigastric pain that is unrelieved with antacids. d. high epigastric burning that is relieved with food intake

*b. back pain that occurs 2 to 4 hours following meals.* Duodenal ulcers are often felt as abdominal pain that radiates to the back. This is in contrast to

When taking the blood pressure of a patient who has recently had a thyroidectomy, the nurse notices that the patient has spasms of the hand. The nurse pages the physician. While waiting for the physician to call back the nurse should prepare to replace: a. potassium b. calcium c. bicarbonate d. phosphate

*b. calcium* *This is Trousseau's sign and it indicates hypocalcemia

To provide free water and intracellular fluid hydration for a patient with Diabetes Inspidus who is NPO, the nurse would expect administration of? a. lactated Ringer's solution. b. dextrose 5% in water. c. dextrose 10% in water. d. dextrose 5% in normal saline (0.9%).

*b. dextrose 5% in water.* Rationale: Fluids such as 5% dextrose in water (D5W) allow water to move from the extracellular fluid to the intracellular fluid. Although D5W is physiologically isotonic, the dextrose is rapidly metabolized, leaving free water to shift into cells

A nursing intervention that is indicated for a male patient following an inguinal herniorrhaphy is? a. applying heat to the inguinal area. b. elevating the scrotum with a scrotal support. c. applying a truss to support the operative site. d. encouraging the patient to cough and deep-breathe

*b. elevating the scrotum with a scrotal support.*

The nurse would increase the comfort of the patient with appendicitis by? a. having the patient lie prone. b. flexing the patient's right knee. c. sitting the patient upright in a chair. d. turning the patient onto his or her left side.

*b. flexing the patient's right knee.* LECTURE: knee-chest position is common, patient's don't want to be moved

In replying to a patient's questions about the seriousness of her chronic kidney disease (CKD), the nurse knows that the stage of CKD is based on the a. total daily urine output. b. glomerular filtration rate. c. serum creatinine and urea levels. d. degree of altered mental status.

*b. glomerular filtration rate (GFR)*

A large number of children at a public school have developed profuse diarrhea and bloody stools. The school nurse suspects food poisoning from the school cafeteria and requests analysis and culture of? a. chicken. b. ground beef. c. commercially canned fish. d. salads with mayonnaise dressing.

*b. ground beef.*

An appropriate nursing intervention for the patient with hyperparathyroidism is to? a. pad side rails as a seizure precaution. b. increase fluid intake to 3000 to 4000 mL daily. c. maintain bed rest to prevent pathologic fractures. d. monitor the patient for Trousseau's phenomenon and Chvostek's sign

*b. increase fluid intake to 3000 to 4000 mL daily* Rationale: A high fluid intake is indicated in hyperparathyroidism to dilute the hypercalcemia and flush the kidneys so that calcium stone formation is reduced. Seizures are not associated with hyperparathyroidism, but impending tetany of hypoparathyroidism can be noted with Trousseau's phenomenon and Chvostek's sign. The patient with hyperparathyroidism is at risk for pathologic fractures resulting from decreased bone density, but mobility is encouraged to promote bone calcification.

A patient with SIADH is treated with water restriction and administration of IV fluids. The nurse evaluates that treatment has been effective when the patient experiences a. increased urine output, decreased serum sodium, and increased urine specific gravity. b. increased urine output, increased serum sodium, and decreased urine specific gravity. c. decreased urine output, increased serum sodium, and decreased urine specific gravity. d. decreased urine output, decreased serum sodium, and increased urine specific gravity

*b. increased urine output, increased serum sodium, and decreased urine specific gravity* Rationale: The patient with SIADH has water retention with hyponatremia, decreased urine output, and concentrated urine with high specific gravity. Improvement in the patient's condition is reflected by increased urine output, normalization of serum sodium, and more water in the urine, decreasing the specific gravity.

A patient with cancer of the stomach at the lesser curvature undergoes a total gastrectomy with an esophagojejunostomy. Postoperatively, the nurse teaches the patient to expect? a. rapid healing of the surgical wound. b. lifelong administration of cobalamin. c. to be able to return to normal dietary habits. d. close follow-up for development of peptic ulcers in the jejunum.

*b. lifelong administration of cobalamin.*

Nursing management of the patient with chronic gastritis includes teaching the patient to? a. take antacids before meals to decrease stomach acidity. b. maintain a nonirritating diet with six small meals a day. c. eliminate alcohol and caffeine from the diet when symptoms occur. d. use nonsteroidal antiinflammatory drugs (NSAIDs) instead of aspirin for any minor pain relief.

*b. maintain a nonirritating diet with six small meals a day.*

Following a patient's esophagogastrostomy for cancer of the esophagus, it is important for the nurse to? a. report any bloody drainage from the NG tube. b. maintain the patient in semi-Fowler's or Fowler's position. c. monitor for abdominal distention that may disrupt the surgical site. d. expect to find decreased breath sounds bilaterally because of the surgical approach

*b. maintain the patient in semi-Fowler's or Fowler's position.*

Several patients are seen at an urgent care center with symptoms of nausea, vomiting, and diarrhea that began 2 hours ago while attending a large family reunion potluck dinner. You question the patients specifically about foods they ingested containing? a. beef. b. meat and milk. c. poultry and eggs. d. home-preserved vegetables.

*b. meat and milk*

A patient is admitted to the emergency department with profuse bright-red hematemesis. During the initial care of the patient, the nurse's first priority is to? a. establish two IV sites with large-gauge catheters. b. perform a nursing assessment of the patient's status. c. obtain a thorough health history to assist in determining the cause of the bleeding. d. perform a gastric lavage with cool tap water in preparation for endoscopic examination.

*b. perform a nursing assessment of the patient's status.*

Priority Decision: A patient on a medical unit has a potassium level of K+ 6.8 mEq/L. The priority action for the nurse would be a. check the patient's blood pressure (BP). b. place the patient on a cardiac monitor. c. instruct the patient to avoid high potassium foods. d. call the lab and request a redraw of the lab to verify results.

*b. place the patient on a cardiac monitor.*

A patient with AKI has a serum potassium level of 6.7 mEq/L (6.7 mmol/L) and the following arterial blood gas results: pH 7.28, PaCO2 30 mm Hg, PaO2 86 mm Hg, HCO − 18 mEq/L (18 mmol/L). The nurse recognizes that treatment of the acid-base problem with sodium bicarbonate would cause a decrease in the a. pH. b. potassium level. c. bicarbonate level. d. carbon dioxide level.

*b. potassium level.*

A patient with hypothyroidism is treated with levothyroxin (Synthroid). When teaching the patient about the therapy, the nurse? a. explains that alternate-day dosage may be used if side effects occur. b. provides written instruction for all information related to the medication therapy. c. assures the patient that a return to normal function will occur with replacement therapy. d. informs the patient that medications must be taken until hormone balance is reestablished.

*b. provides written instruction for all information related to the medication therapy*. Rationale: Because of the mental sluggishness, inattentiveness, and memory loss that occur with hypothyroidism, it is important to provide written instructions and repeat information when teaching the patient. Replacement therapy must be taken for life, and alternate-day dosing is not therapeutic. Although most patients return to a normal state with treatment, cardiovascular conditions and psychoses may persist.

One of the major advantages of peritoneal dialysis is that? a. no medications are required because of the enhanced efficiency of the peritoneal membrane in removing toxins. b. the diet is less restricted and dialysis can be performed at home. c. the dialysate is biocompatible and causes no long-term consequences. d. high glucose concentrations of the dialysate cause a reduction in appetite promoting weight loss.

*b. the diet is less restricted and dialysis can be performed at home.*

In teaching patients at risk for upper GI bleeding to prevent bleeding episodes, the nurse stresses that? a. all stools and vomitus must be tested for the presence of blood. b. the use of over-the-counter (OTC) medications of any kind should be avoided. c. antacids should be taken with all prescribed medications to prevent gastric irritation. d. misoprostol (Cytotec) should be used to protect the gastric mucosa in individuals with peptic ulcers.

*b. the use of over-the-counter (OTC) medications of any kind should be avoided.*

To prevent the most common serious complication of peritoneal dialysis (PD), it is important for the nurse to? a. infuse the dialysate slowly. b. use strict aseptic technique in the dialysis procedures. c. have the patient empty the bowel before the inflow phase. d. reposition the patient frequently and promote deep breathing

*b. use strict aseptic technique in the dialysis procedures.* b. Rationale: Peritonitis is a common complication of PD and may require catheter removal and termination of dialysis. Infection occurs from contamination of the dialysate or tubing or from progression of exit- site or tunnel infections, and strict sterile technique must be used by health professionals as well as the patient to prevent contamination. Too-rapid infusion may cause shoulder pain, and pain may be caused if the catheter tip touches the bowel. Difficulty breathing, atelectasis, and pneumonia may occur from pressure of the fluid on the diaphragm, which may be prevented by elevating the head of the bed and promoting repositioning and deep breathing

The immunologic mechanisms involved in glomerulonephritis include? a.tubular blocking by precipitates of bacteria and antibody reactions. b.deposition of immune complexes and complement along the GBM. c.thickening of the GBM from autoimmune microangiopathic changes. d.destruction of glomeruli by proteolytic enzymes contained in the GBM.

*b.deposition of immune complexes and complement along the GBM.*

Measures indicated in the conservative therapy of CKD include? a. Decreased fluid intake, carbohydrate intake, and protein intake b. Increased fluid intake, decreased carbohydrate intake and protein intake c. Decreased fluid intake and protein intake, increased carbohydrate intake d. Decreased fluid intake and carbohydrate intake, increased protein intake

*c. Decreased fluid intake and protein intake, increased carbohydrate intake*

If a patient is in the diuretic phase of AKI, the nurse must monitor for which serum electrolyte imbalances? a. Hyperkalemia and hyponatremia b. Hyperkalemia and hypernatremia c. Hypokalemia and hyponatremia d. Hypokalemia and hypernatremia

*c. Hypokalemia and hyponatremia*

The health care provider prescribes levothyroxine (Synthroid) for a patient with hypothyroidism. Following teaching regarding this drug, the nurse determines further instruction is needed when the patient says? a. I can expect the medication dose may need to be increased. b. I can expect to return to normal function with the use of this drug. c. I only need to take this drug until my symptoms are improved. d. I will report any chest pain or difficulty breathing to the doctor right away.

*c. I only need to take this drug until my symptoms are improved.*

A nurse is caring for a client who has chronic kidney failure and the following lab results: BUN 196 mg/dL, sodium 152 mEq/L, and potassium 7.3 mEq/L. Which of the following interventions should the nurse implement? a. Initiate an IV infusion of 0.9% sodium chloride b. Give oral spironolactone c. Infuse regular insulin in dextrose 10% in water d. Administer furosemide

*c. Infuse regular insulin in dextrose 10% in water*

A major difference between ulcerative colitis and Crohn's disease is that Crohn's disease? a. Frequently results in toxic megacolon b. Causes fewer nutritional deficiencies than does ulcerative colitis c. Often recurs after surgery, whereas ulcerative colitis is curable with a colectomy d. Is manifested by rectal bleeding and anemia more frequently than is ulcerative colitis

*c. Often recurs after surgery, whereas ulcerative colitis is curable with a colectomy*

The nurse determines that a patient with AKI is in the recovery phase when the patient experiences? a. a return to normal weight. b. a urine output of 3700 mL/day. c. decreasing blood urea nitrogen (BUN) and creatinine levels. d. decreasing sodium and potassium levels.

*c. decreasing blood urea nitrogen (BUN) and creatinine levels.* c. Rationale: The blood urea nitrogen (BUN) and creatinine levels remain high during the oliguric and diuretic phases of AKI. The recovery phase begins when the glomerular filtration returns to a rate at which BUN and creatinine stabilize and then decrease. Urinary output of 3 to 5 L/day, decreasing sodium and potassium levels, and fluid weight loss are characteristic of the diuretic phase of acute renal failure (ARF).

A patient is admitted to the hospital in thyrotoxic crisis. On physical assessment of the patient, the nurse would expect to find? a. hoarseness and laryngeal stridor. b. bulging eyeballs and dysrhythmias. c. elevated temperature and signs of heart failure. d. lethargy progressing suddenly to impairment of consciousness.

*c. elevated temperature and signs of heart failure* Rationale: A hyperthyroid crisis results in marked manifestations of hyperthyroidism, with severe tachycardia, heart failure, shock, hyperthermia, agitation, nausea, vomiting, diarrhea, delirium, and coma. Although exophthalmos (bulging eyeballs) may be present in the patient with Graves' disease, it is not a significant factor in hyperthyroid crisis. Hoarseness and laryngeal stridor are characteristic of the tetany of hypoparathyroidism, and lethargy progressing to coma is characteristic of myxedema coma, a complication of hypothyroidism

During the nursing assessment of the patient with renal insufficiency, the nurse asks the patient specifically about a history of? a. angina. b. asthma. c. hypertension. d. rheumatoid arthritis.

*c. hypertension.* c. Rationale: The most common causes of CKD in the United States are hypertension and diabetes mellitus. The nurse should obtain information on long-term health problems that are related to kidney disease. The other disorders are not closely associated with renal disease.

A patient with esophageal cancer is scheduled for a partial esophagectomy. A nursing diagnosis that is likely to be of highest priority preoperatively is? a. deficient fluid volume related to inadequate intake. b. impaired oral mucous membrane related to inadequate oral hygiene. c. imbalanced nutrition: less than body requirements related to dysphagia. d. ineffective self-health management related to lack of knowledge of disease process

*c. imbalanced nutrition: less than body requirements related to dysphagia.*

The nurse identifies a nursing diagnosis of risk for injury: fracture related to alterations in calcium and phosphorus metabolism for a patient with CKD. The pathologic process directly related to the risk for fractures is? a. loss of aluminum through the impaired kidneys. b. deposition of calcium phosphate in soft tissues of the body. c. impaired vitamin D activation resulting in decreased GI absorption of calcium. d. increased release of parathyroid hormone in response to decreased calcium levels

*c. impaired vitamin D activation resulting in decreased GI absorption of calcium.*

The patient with end-stage renal disease tells the nurse that she hates the thought of being tied to the machine but is glad to start dialysis because she will be able to eat and drink what she wants. Based on this information, the nurse identifies the nursing diagnosis of? a. self-esteem disturbance related to dependence on dialysis. b. anxiety related to perceived threat to health status and role functioning. c. ineffective self-health management related to lack of knowledge of treatment plan. d. risk for imbalanced nutrition: more than body requirements related to increased dietary intake

*c. ineffective self-health management related to lack of knowledge of treatment plan.* c. Rationale: A patient on hemodialysis will continue to have dietary and fluid restrictions for which the machine cannot compensate, and the patient needs additional teaching regarding her therapeutic regimen. The other nursing diagnoses are not supported with defining characteristics

To assess the patency of a newly placed arteriovenous graft for dialysis, the nurse should? a. irrigate the graft daily with low-dose heparin. b. monitor for any increase in BP in the affected arm. c. listen with a stethoscope over the graft for the presence of a bruit. d. frequently monitor the pulses and neurovascular status distal to the graft.

*c. listen with a stethoscope over the graft for the presence of a bruit.*

Aldosterone is secreted by the adrenal cortex in response to? a. excessive water intake. b. loss of serum potassium. c. loss of sodium and water. d. increased serum osmolality.

*c. loss of sodium and water* Aldosterone is secreted in response to a decrease in plasma volume (loss of water), decrease in serum sodium or insufficient renal perfusion. B-it is also secreted in response to an increase in serum potassium. A-insufficient water intake would be more likely B-an increase in serum potassium D-a decreased serum osmolality would trigger it to retain sodium and water. ADH is released in response to increased serum osmolality.

The nurse teaches the patient with a hiatal hernia or GERD to control symptoms by? a. drinking 10 to 12 oz of water with each meal. b. spacing six small meals a day between breakfast and bedtime. c. sleeping with the head of the bed elevated on 4- to 6-in blocks. d. performing daily exercises of toe-touching, sit-ups, and weight lifting

*c. sleeping with the head of the bed elevated on 4- to 6-in blocks.*

A patient with a history of peptic ulcer disease is hospitalized with symptoms of a perforation. During the initial assessment, the nurse would expect the patient to report? a. vomiting of bright-red blood. b. projectile vomiting of undigested food. c. sudden, severe upper abdominal pain and shoulder pain. d. hyperactive stomach sounds and upper abdominal swelling.

*c. sudden, severe upper abdominal pain and shoulder pain.* With perforation the patient gets sudden, sever abdominal pain that radiates to the shoulder. They also get a rigid board-like abdomen.

A diagnostic study that indicates renal blood flow, glomerular filtration, tubular function, and excretion is a(n)? a.IVP. b.VCUG. c.renal scan. d.loopogram.

*c.renal scan.* A renal scan evaluates anatomic structures, perfusion, and function of kidneys. The scan shows the location, size, and shape of kidney and can be used to assess blood flow, glomerular filtration, tubular function, and urinary excretion

A patient with Graves' disease asks the nurse what caused the disorder. The best response by the nurse is, a. "The cause of Graves' disease is not known, although it is thought to be genetic." b. "It is usually associated with goiter formation from an iodine deficiency over a long period of time." c. "Antibodies develop against thyroid tissue and destroy it, causing a deficiency of thyroid hormones." d. "In genetically susceptible persons, antibodies are formed that cause excessive thyroid hormone secretion."

*d. "In genetically susceptible persons, antibodies are formed that cause excessive thyroid hormone secretion."* Rationale: In Graves' disease, antibodies to the TSH receptor are formed, attach to the receptors, and stimulate the thyroid gland to release triiodothyronine (T3), thyroxine (T4), or both, creating hyperthyroidism. The disease is not directly genetic, but individuals appear to have a genetic susceptibility to develop autoimmune antibodies. B-Goiter formation from insufficient iodine intake is usually associated with hypothyroidism.

Following thyroid surgery, the nurse suspects damage or removal of the parathyroid glands when the patient develops? a. Muscle weakness and weight loss b. Hyperthermia and severe tachycardia c. Hypertension and difficulty swallowing d. Laryngeal stridor and tingling in the hands and feet

*d. Laryngeal stridor and tingling in the hands and feet*

During the assessment of a patient with acute abdominal pain, the nurse should? a. Perform deep palpation before auscultation b. Obtain blood pressure and pulse rate to determine hypervolemic changes c. Auscultate bowel sounds because hyperactive bowel sounds suggest paralytic ileus d. Measure body temperature because an elevated temperature may indicate an inflammatory or infectious process

*d. Measure body temperature because an elevated temperature may indicate an inflammatory or infectious process*

Which of the following should a patient be taught after a hemorrhoidectomy? a. Take mineral oil prior to bedtime. b. Eat a low-fiber diet to rest the colon. c. Administer oil-retention enema to empty the colon. d. Use prescribed pain medication before a bowel movement.

*d. Use prescribed pain medication before a bowel movement.*

A patient on hemodialysis develops a thrombus of a subcutaneous AV graft requiring its removal. While waiting for a replacement graft or fistula, the patient is most likely to have? a. peritoneal dialysis. b. a percutaneous jugular vein cannula. c. a percutaneous femoral vein cannula. d. a Silastic catheter tunneled subcutaneously to the jugular vein.

*d. a Silastic catheter tunneled subcutaneously to the jugular vein.* d. Rationale: A more permanent, soft, flexible Silastic double-lumen catheter is being used for long-term access when other forms of vascular access have failed. Femoral vein catheters may only remain in place for up to 1 week and jugular catheters for 1 to 3 weeks.

The edema that occurs in nephrotic syndrome is due to? a. increased hydrostatic pressure caused by sodium retention. b. decreased aldosterone secretion from adrenal insufficiency. c. increased fluid retention caused by decreased glomerular filtration. d. decreased colloidal osmotic pressure caused by loss of serum albumin.

*d. decreased colloidal osmotic pressure caused by loss of serum albumin.*

The dialysate for PD contains? a. electrolytes in an equal concentration to that of the blood. b. calcium in a lower concentration than in the blood. c. sodium in a higher concentration than in the blood. d. dextrose in a higher concentration than in the blood.

*d. dextrose in a higher concentration than in the blood.* d. Rationale: Dextrose is added to dialysate fluid to create an osmotic gradient across the membrane to remove excess fluid from the blood. The dialysate fluid has no potassium so that potassium will diffuse into the dialysate from the blood. Dialysate also usually contains higher calcium to promote its movement into the blood. Dialysate sodium is usually less than or equal to that of blood to prevent sodium and fluid retention

When replacement therapy is started for a patient with long-standing hypothyroidism, it is most important for the nurse to monitor the patient for a. insomnia. b. weight loss. c. nervousness. d. dysrhythmias.

*d. dysrhythmias*. Rationale: All these manifestations may occur with treatment of hypothyroidism, but as a result of the effects of hypothyroidism on the cardiovascular system, when thyroid replacement therapy is started, myocardial oxygen consumption is increased, and the resultant oxygen demand may cause angina, cardiac dysrhythmias, and heart failure

The most appropriate snack for the nurse to offer the patient with CKD is? a. raisins. b. ice cream. c. dill pickles. d. hard candy.

*d. hard candy.* d. Rationale: *A patient with CKD may have unlimited intake of sugars and starches (unless the patient is diabetic)*, and hard candy is an appropriate snack and may help relieve the metallic and urine taste common in the mouth. Raisins are a high-potassium food, pickled foods have high sodium contents, and ice cream contains protein.

A 22-year-old patient calls the outpatient clinic complaining of nausea and vomiting and right lower abdominal pain. The nurse advises the patient to? a. use a heating pad to relax the muscles at the site of the pain. b. drink at least 2 quarts of juice to replace the fluid lost in vomiting. c. take a laxative to empty the bowel before examination at the clinic. d. have the symptoms evaluated by a health care provider right away.

*d. have the symptoms evaluated by a health care provider right away.* The age of the patient along with these clinical manifestations suggest appendicitis. Never use heating pads

A patient with end-stage renal failure is scheduled for hemodialysis following healing of an AV fistula. The nurse explains that during dialysis a. he will be placed on a cardiac monitor to detect any adverse effects that might occur. b. the dialyzer will remove and hold part of his blood for 20 to 30 minutes to remove the waste products. c. a large catheter with two lumens will be inserted into the fistula to send blood to and return it from the dialyzer. d. he will be able to visit, read, sleep, or watch TV while reclining in a chair.

*d. he will be able to visit, read, sleep, or watch TV while reclining in a chair.*

That hormones of one gland influence the function of hormones of another gland is demonstrated by the fact that a. increased insulin levels inhibit the secretion of glucagon. b. increased testosterone levels inhibit the release of estrogen. c. increased cortisol levels stimulate the secretion of insulin. d. increased atrial natriuretic peptide (ANP) levels inhibit the secretion of aldosterone

*d. increased atrial natriuretic peptide (ANP) levels inhibit the secretion of aldosterone* Rationale: Atrial natriuretic peptide (ANP) is secreted in response to high blood volume and high serum sodium levels and has an inhibiting effect on ADH and the renin-angiotensin-aldosterone system, the effects of which would make the blood volume even higher. Glucagon secretion inhibits insulin secretion, but insulin does not inhibit glucagon. The relationship between cortisol and and insulin is indirect—cortisol raises blood glucose levels, and insulin secretion is stimulated by the high glucose levels. Testosterone and estrogen have no reciprocal action, and both are secreted by the body in response to tropic hormones

An example of an IV solution that would be appropriate to treat an extracellular fluid volume deficit is? a. D5W. b. 3% saline. c. D5W in 1/2 normal saline (0.45%). d. lactated Ringer's solution.

*d. lactated Ringer's solution.* d. Rationale: An isotonic solution does not change the osmolality of the blood and does not cause fluid shifts between the extracellular fluid and intracellular fluid. In the case of extracellular fluid loss, an isotonic solution, such as lactated Ringer's solution, is ideal because it stays in the extracellular compartment. A hypertonic solution would pull fluid from the cells into the extracellular compartment, resulting in cellular fluid loss and possible vascular overload

On reading the urinalysis results of a dehydrated patient, the nurse would expect to find? a. a pH of 8.4. b. RBCs of 4/hpf. c. color: yellow, cloudy. d. specific gravity of 1.035

*d. specific gravity of 1.035*

Following anal surgery, the nurse advises the patient to? a. use daily laxatives to facilitate bowel emptying. b. use ice packs to the perineum to prevent swelling. c. avoid having a bowel movement for several days until healing occurs. d. take warm sitz baths several times a day to promote comfort and cleaning.

*d. take warm sitz baths several times a day to promote comfort and cleaning.*

While caring for the patient in the oliguric phase of AKI, the nurse monitors the patient for associated collaborative problems, notifying the health care provider when a. urine output is 300 mL/day. b. edema occurs in the feet, legs, and sacral area. c. the cardiac monitor reveals a depressed T wave and a sagging ST segment. d. the patient experiences increasing muscle weakness and abdominal cramping.

*d. the patient experiences increasing muscle weakness and abdominal cramping.*

While caring for the patient in the oliguric phase of AKI, the nurse monitors the patient for associated collaborative problems, notifying the health care provider when a. urine output is 300 mL/day. b. edema occurs in the feet, legs, and sacral area. c. the cardiac monitor reveals a depressed T wave and a sagging ST segment. d. the patient experiences increasing muscle weakness and abdominal cramping.

*d. the patient experiences increasing muscle weakness and abdominal cramping.* d. Rationale: Hyperkalemia is a potentially life- threatening complication of AKI in the oliguric phase. Muscle weakness and abdominal cramping are signs of the neuromuscular impairment that occurs with hyperkalemia, in addition to cardiac conduction abnormalities of peaked T wave, prolonged PR interval, prolonged QRS interval, and depressed ST segment. Urine output of 300 mL/day is expected during the oliguric phase, as is the development of peripheral edema.

Inflammatory bowel disease (IBD) is treated with a ___ protein, ___ calorie, ___ fiber diet?

*high protein, high calorie, low residue/fiber*

The most common cause of death for patients with AKI is ____ while the most common cause of death for CKD is _______ ?

*infection, cardiovascular disease*

A diet for a patient who is in acute kidney injury should include?

*moderate-low protein (want to prevent catabolism), low sodium, low potassium, and low phosphorous* -carbohydrates to avoid catabolism They have hyperkalemia, hypertension, and hyperphosphatemia

ON EXAM: The Hypothalamic-Pituitary-Adrenal Axis results in the release of _____ from the hypothalamus in response to stress, which causes the anterior pituitary to release _____ , which causes the adrenal cortex to release _______ ?

-CRH (hypothalamus) -ACTH (pituitary) -CORTISOL (adrenal gland's cortex)

A short-term stressor would trigger the release of _____ from the adrenal medulla, while a long-term stressor would trigger the release of _____ from the adrenal cortex?

-Catecholamines (epinephrine/norepinephrine) -Mineralocorticoids/Glucocorticoids

The normal response to increased serum osmolality is the release of? a. aldosterone from the adrenal cortex, which stimulates sodium excretion by the kidney. b. ADH from the posterior pituitary gland, which stimulates the kidney to reabsorb water. c. mineralocorticoids from the adrenal gland, which stimulate the kidney to excrete potassium. d. calcitonin from the thyroid gland, which increases bone resorption and decreases serum calcium leve

. b. Rationale: Antidiuretic hormone (ADH) release is controlled by the osmolality of the blood, and as the osmolality rises, ADH is released from the posterior pituitary gland and acts on the kidney to cause reabsorption of water from the kidney tubule, resulting in more dilute blood and more concentrated urine. Aldosterone, the major mineralocorticoid, causes sodium reabsorption from the kidney and potassium excretion. Calcium levels are not a factor in serum osmolality.

Will these events inhibit or stimulate the release of ADH? 1. Hemorrhage? 2. Hypotension? 3. Drinking a liter of water in 10 min? 4. A serum osmolality of 275 mOs/kg?

1-Stimulate 2-Stimulate 3-Inhibit 4-Inhibit (Normal range is 280-295 mOSM/kg)

A patient on peritoneal dialysis has a cloudy effluent (dialysis drainage), WBC present in the effluent a temperature of 101.5 and abdominal pain. This is indicative of which complication of PD? A. Exit site infection B. Peritonitis C. Respiratory distress D. Obstruction to in flow/outflow E. Protein loss

2. Peritonitis

The normal level for total serum calcium ranges from ___ to ___ while the free or ionized serum calcium ranges from ___ to ___ ? How would malnutrition affect total calcium? How would acidosis affect total calcium?

8.6-10.2 4.6-5.2 Malnutrition causes deceased albumin-so total would be lower because the amount of bound calcium would be lower Acidosis results in MORE ionized calcium. Breathing into a paper bag would temporarily increase your serum calcium *You will want to memorize the normal values for both total and ionized. Also know the role that serum albumin levels and serum pH has on ionized calcium levels*

What is Cushing's disease?

A *pituitary tumor* that results in increased secretion of ACTH that targets the adrenal cortex and causes it to release high amounts of corstisol, causing Cushing syndrome

A nurse in an intensive care unit is admitting a client who has myxedema coma. Which of the following should the nurse anticipate in caring for this client? (Select all that apply.) A. Observe cardiac monitor for inverted T wave. B. Observe for evidence of urinary tract infection. C. Initiate IV fluids using 0.9% sodium chloride. D. Expect a prescription for levothyroxine (Synthroid) IV bolus. E. Provide warmth using a heating pad.

A, B, C, D

A nurse is reviewing the laboratory findings of a client who has Cushing's disease. Which of the following findings are expected for this client? (Select all that apply.) A. Sodium 150 mEq/L B. Potassium 3.3 mEq/L C. Calcium 8.0 mg/dL D. Lymphocyte count 35% E. Fasting glucose 145 mg/dL

A, B, C, E A-hypernatremia is expected B-hypokalemia is expected C-hypocalcemia is expected E-hyperglycemia is expected

When assessing a patient who has Cushing's syndrome, a nurse associates which clinical manifestations with this disorder? (Select all that apply.) A. Osteoporosis B. Moon face C. Glycosuria D. Ketonuria E. Mood swings

A, B, C, E Cushing's syndrome results from excess secretion of adrenocorticotropic hormone (ACTH), and these effects result in manifestations such as redistribution of fat to the face and belly, excess blood sugar, mood changes, and calcium loss from bone. Ketoacidosis does not occur.

A nurse is reviewing serum laboratory results for a client who has Addison's disease. Which of the following findings are typical for a client who has this condition? (Select all that apply.) A. Sodium 130 mEq/L B. Potassium 6.1 mEq/L C. Calcium 11.6 mg/dL D. Magnesium 2.5 mg/dL E. Glucose 65 mg/dL

A, B, C, E Everything is the opposite of Cushing's: Hyponatremia, Hyperkalemia, Hypercalcemia and hypoglycemia

A nurse is planning care for a client who has Cushing's disease. In planning care, the nurse should recognize that clients who have Cushing's disease are at increased risk for which of the following? (Select all that apply.) A. Infection B. Gastric ulcer C. Renal calculi D. Bone fractures E. Dysphagi

A, B, D

A nurse in a provider's office is planning care for a client who has a new diagnosis of Graves' disease and a new prescription for methimazole (Tapazole). Which of the following should the nurse include in the plan of care? (Select all that apply.) A. Monitor CBC. B. Monitor triiodothyronine (T3). C. Inform the client that the medication should not be taken for more than 3 months. D. Advise the client to take the medication at the same time every day. E. Inform the client that an adverse effect of this medication is iodine toxicity.

A, B, D Also pregnancy category D

Individuals considered to be at increased risk for CKD include (select all that apply) a. Older African Americans b. Individuals over 60 c. Those with a history of pancreatitis d. Those with a history of hypertension e. Those with a history of type II diabetes

A, B, D, E

A nurse is caring for a client who has primary diabetes insipidus. Which of the following manifestations should the nurse expect to find? (Select all that apply.) A. Serum sodium of 155 mEq/L B. Fatigue C. Serum osmolality of 250 mOsm/L D. Polyuria E. Nocturia

A, B, D, E C-serum osmolality will be greater than 300 as this is concentrated serum.

What does the nurse identify as possible complications of glucocorticoid therapy? (Select all that apply.) A. Impaired carbohydrate and glucose tolerance Correct B. Increased production of glucocorticoids from the adrenals C. Predisposition to gastrointestinal ulceration Correct D. Decrease in serum sodium and glucose levels E. Increase in plaque development in coronary arteries

A, C Adverse effects of glucocorticoids occur in response to pharmacologic doses; they include suppression of adrenal glucocorticoids; glucose and carbohydrate intolerance, elevated blood glucose level; water and sodium retention; and increased gastric acid secretion. Plaque development in arteries is not affected.

erase

A, C *Primary cause of death is infection* *Most common cause is diabetic nephropathy*

What characterizes AKI (select all that apply)? A. Primary cause of death is infection. B. It usually affects older people. C. The disease course is potentially reversible. D. The most common cause is diabetic nephropathy. E. Cardiovascular disease is the most common cause of death.

A, C AKI is potentially reversible CKD is not. It has a high mortality rate, and the *primary cause of death is infection*; the primary cause of death for chronic kidney failure is cardiovascular disease. *AKI commonly results from severe, prolonged hypotension or hypovolemia or exposure to a nephrotoxic agent*. Although it can occur at any age, the older adult is more susceptible to AKI because the number of functioning nephrons decreases with age. Thye most common cause of CKD is diabetes (lecture)*

A nurse is caring for a client who has syndrome of inappropriate antidiuretic hormone (SIADH). Which of the following findings should the nurse expect? (Select all that apply.) A. Decreased serum sodium B. Urine specific gravity 1.001 C. Serum osmolarity 230 mOsm/L D. Polyuria E. Increased thirst

A, C B-in contrast to DI, SIADH has a concentrated urine-greater than 1.030 is caused by excess ADH D-reduced urine output E-increased thirst is DI not SIADH

A nurse is assessing a client who has prerenal acute kidney injury (AKI). Which of the following should the nurse include in the assessment? (Select all that apply.) A. Blood pressure B. Cardiac enzymes C. Urine output D. Serum creatinine E. Serum electrolytes

A, C, D, E A-prerenal is related to hypo-perfusion of the kidneys C-prerenal has several stages and the first is oliguria, output lower than 30mL/hr or 400 mL/day D-Serum creatinine will help determine if Azotemia is present. In prerenal BUN increases more than Creatinine E-Serum electrolytes are important because FENa-will be LOW (below 1%)

In addition to urine function, you recognize that the kidneys perform numerous other functions important to the maintenance of homeostasis. Which physiologic processes are performed by the kidneys (select all that apply)? A. Production of renin B. Hemolysis of old red blood cells (RBCs) C. Activation of vitamin D D. Carbohydrate metabolism E. Erythropoietin production

A, C, E In addition to urine formation, the kidneys activate vitamin D to maintain calcium levels, produce erythropoietin to stimulate RBC production, and release renin to maintain blood pressure. Carbohydrate metabolism and hemolysis of old RBCs are not physiologic functions performed by the kidneys.

A nurse is reviewing the health record of a client who has syndrome of inappropriate antidiuretic hormone (SIADH). Which of the following laboratory findings should the nurse anticipate? (Select all that apply.) A. Low serum sodium B. High serum potassium C. Decreased urine osmolality D. High urine sodium E. Increased urine-specific gravity

A, D, E NOT B-ADH does not affect potassium like mineralcorticoids do NOT-C-increased, you would see decreased with DI

Nurses need to educate patients at risk for CKD. Which individuals are considered to be at increased risk (select all that apply)? A. Older African Americans B. Individuals older than 60 years C. Those with a history of pancreatitis D. Those with a history of hypertension E. Those with a history of type 2 diabetes

A,B, D, E Risk factors for CKD include diabetes mellitus, hypertension, age older than 60 years, cardiovascular disease, family history of CKD, exposure to nephrotoxic drugs, and ethnic minorities (e.g., African American, Native American).

A patient has been given instructions about levothyroxine (Synthroid). Which statement by the patient indicates understanding of these instructions? A. "I'll take this medication in the morning so as not to interfere with sleep." B. "I'll plan to double my dose if I gain more than 1 pound per day." C. "It is best to take the medication with food so I don't have any nausea." D. "I'll be glad when I don't have to take this medication in a few months."

A. "I'll take this medication in the morning so as not to interfere with sleep." Levothyroxine is used to treat hypothyroidism by increasing the basal metabolism and thus wakefulness. It is administered as a once-daily dose and is a lifelong therapy. It is best taken on an empty stomach to enhance absorption.

Which of the following statements by the nurse regarding continuous ambulatory peritoneal dialysis (CAPD) would be of highest priority when teaching a patient new to this procedure? A. "It is essential that you maintain aseptic technique to prevent peritonitis." B. "You will be allowed a more liberal protein diet once you complete CAPD." C. "It is important for you to maintain a daily written record of blood pressure and weight." D. "You will need to continue regular medical and nursing follow-up visits while performing CAPD."

A. "It is essential that you maintain aseptic technique to prevent peritonitis." Peritonitis is a potentially fatal complication of peritoneal dialysis, and thus it is imperative to teach the patient methods of preventing this from occurring. Although the other teaching statements are accurate, they do not have the potential for mortality as does the peritonitis, thus making that nursing action of highest priority.

A patient with a history of peptic ulcer disease has presented to the emergency department with complaints of severe abdominal pain and a rigid, boardlike abdomen, prompting the health care team to suspect a perforated ulcer. Which of the following actions should the nurse anticipate? A. Providing IV fluids and inserting a nasogastric tube B. Administering oral bicarbonate and testing the patient's gastric pH level C. Performing a fecal occult blood test and administering IV calcium gluconate D. Starting parenteral nutrition and placing the patient in a high-Fowler's position

A. *Providing IV fluids and inserting a nasogastric tube* A perforated peptic ulcer requires IV replacement of fluid losses and continued gastric aspiration by NG tube. Nothing is given by mouth and gastric pH testing is not a priority. Calcium gluconate is not a medication directly relevant to the patient's suspected diagnosis and parenteral nutrition is not a priority in the short term.

Which nursing diagnosis is a priority in the care of a patient with renal calculi? A. Acute pain B. Deficient fluid volume C. Risk for constipation D. Risk for powerlessness

A. Acute pain Urinary stones are associated with severe abdominal or flank pain. Deficient fluid volume is unlikely to result from urinary stones, and constipation is more likely to be an indirect consequence rather than a primary clinical manifestation of the problem. The presence of pain supersedes powerlessness as an immediate focus of nursing care.

Which nursing diagnosis is a priority in the care of a patient with renal calculi? A. Acute pain B. Deficient fluid volume C. Risk for constipation D. Risk for powerlessness Rationale

A. Acute pain Urinary stones are associated with severe abdominal or flank pain. Deficient fluid volume is unlikely to result from urinary stones, and constipation is more likely to be an indirect consequence rather than a primary clinical manifestation of the problem. The presence of pain supersedes powerlessness as an immediate focus of nursing care.

A patient who has Paget's disease is receiving calcitonin-salmon (Miacalcin) nasal spray. A nurse should expect the patient to have which therapeutic response if the medication is having the desired effect? A. Decrease in bone pain B. Increase in the serum calcium level C. Lower daily requirement of vitamin D D. Elevated serum alkaline phosphatase level

A. Decrease in bone pain Calcitonin is used to treat both osteoporosis and Paget's disease, which is characterized by increased bone resorption, skeletal abnormalities, and bone pain. Calcitonin acts to inhibit the activity of osteoclasts, thereby reducing bone resorption of calcium and thus bone pain. As a result of the reduced bone turnover, the alkaline phosphatase and calcium levels in the blood are lowered. Treatment should include an adequate intake of vitamin D.

A nurse is caring for a client who has primary adrenal insufficiency. Which of the following findings should the nurse anticipate after an IV injection of ACTH 1.0 mg? A. Decrease in serum plasma cortisol B. Elevated fasting serum blood glucose C. Decrease in serum sodium D. Increase in urinary output

A. Decrease in serum plasma cortisol *Normally this would increase cortisol. The ACTH would not have an effect as this is primary adrenal insufficiency

What is the most likely cause of an addisonian crisis? A. Failure to taper off corticosteroid medication B. Failure to control blood glucose levels below 150 mg/dL C. Low serum calcium levels D. Low serum phosphate levels

A. Failure to taper off corticosteroid medication Gradual tapering of a corticosteroid medication is necessary to prevent adrenal insufficiency. The other options are not related to an induced Addison's disease.

Which are characteristic clinical manifestations of acute poststreptococal glomerulonephritis (APSGN)? A. Hematuria and dependent edema B. Anterior pelvic pain and glycosuria C. Lower back pain and inflamed kidneys D. Dyspnea and dull, aching pain over the urinary bladder Rationale Generalized body edema, hypertension, oliguria, hematuria with a smoky or rusty appearance, and proteinuria may occur. Fluid retention results from decreased glomerular filtration. The edema appears initially in low-pressure tissues, such as around the eyes (periorbital edema), but it later progresses to involve the total body as ascites or peripheral edema in the legs.

A. Hematuria and dependent edema

A patient who has gastroesophageal reflux disease (GERD) is taking magnesium hydroxide (milk of magnesia). Which outcome should a nurse expect if the medication is achieving the desired therapeutic effect? A. Neutralized gastric acid B. Reduced stomach motility C. Increased barrier to pepsin D. Reduced duodenal pH

A. Neutralized gastric acid Antacids work by neutralizing, absorbing, or buffering gastric acid, which raises the gastric pH above 5. For patients with GERD, antacids can produce symptomatic relief. Increased barrier to pepsin is an effect of sucralfate (Carafate). Reduced stomach motility is not an effect of milk of magnesia.

A nurse administers which medication to inhibit an enzyme that makes gastric acid in a patient who has a duodenal ulcer? A. Omeprazole (Prilosec) B. Famotidine (Pepcid) C. Misoprostol (Cytotec) D. Ranitidine (Zantac)

A. Omeprazole (Prilosec) Omeprazole causes irreversible inhibition of the proton pump, the enzyme that generates gastric acid. It is a powerful suppressant of acid secretion. Famotidine and ranitidine block histamine2 receptors on parietal cells. Misoprostol protects against ulcers caused by nonsteroidal anti-inflammatory drugs (NSAIDs) by stimulating the secretion of mucus and bicarbonate to maintain submucosal blood flow.

A nurse administers which medication to inhibit an enzyme that makes gastric acid in a patient who has a duodenal ulcer? A. Omeprazole (Prilosec) B. Famotidine (Pepcid) C. Misoprostol (Cytotec) D. Ranitidine (Zantac)

A. Omeprazole (Prilosec) Omeprazole causes irreversible inhibition of the proton pump, the enzyme that generates gastric acid. It is a powerful suppressant of acid secretion. Famotidine and ranitidine block histamine2 receptors on parietal cells. Misoprostol protects against ulcers caused by nonsteroidal anti-inflammatory drugs (NSAIDs) by stimulating the secretion of mucus and bicarbonate to maintain submucosal blood flow.

When assessing a patient who has Cushing's syndrome, a nurse associates which clinical manifestations with this disorder? (Select all that apply.) A. Osteoporosis B. Moon face C. Glycosuria D. Ketonuria E. Mood swings

A. Osteoporosis B. Moon face C. Glycosuria E. Mood swings Cushing's syndrome results from excess secretion of adrenocorticotropic hormone (ACTH), and these effects result in manifestations such as redistribution of fat to the face and belly, excess blood sugar, mood changes, and calcium loss from bone. Ketoacidosis does not occur.

A patient is admitted to the hospital with CKD. You understand that this condition is characterized by A. Progressive irreversible destruction of the kidneys B. A rapid decrease in urinary output with an elevated BUN level C. Increasing creatinine clearance with a decrease in urinary output D. Prostration, somnolence, and confusion with coma and imminent death

A. Progressive irreversible destruction of the kidneys

Your plan for care of a patient with AKI includes which goal of dietary management? A. Provide sufficient calories while preventing nitrogen excess. B. Deliver adequate calories while restricting fat and protein intake. C. Replace protein intake with enough fat intake to sustain metabolism. D. Restrict fluids, increase potassium intake, and regulate sodium intake. Rationale

A. Provide sufficient calories while preventing nitrogen excess. The challenge of nutrition management in AKI is to provide adequate calories to prevent catabolism despite the restrictions required to prevent electrolyte and fluid disorders and azotemia

A patient with a history of peptic ulcer disease is admitted to the emergency department with complaints of severe abdominal pain and a rigid, board-like abdomen. The health care team suspects a perforated ulcer. Which action should you anticipate? A. Providing IV fluids and monitoring nasogastric tube drainage B. Administering calcium gluconate and testing the patient's gastric pH level C. Performing a fecal occult blood test and administering IV calcium gluconate D. Starting parenteral nutrition and placing the patient in a high Fowler's position Rationale

A. Providing IV fluids and monitoring nasogastric tube drainage A perforated peptic ulcer requires IV replacement of fluid losses and continued gastric aspiration by a nasogastric tube. Nothing is given by mouth, and gastric pH testing is not a priority. Calcium gluconate is not a medication directly relevant to the patient's suspected diagnosis, and parenteral nutrition is not a priority in the short term.

You are caring for a patient in the emergency department with complaints of acute abdominal pain, nausea, and vomiting. When you palpate the patient's left lower abdominal quadrant, the patient complains of pain in the right lower quadrant. You will document this as which diagnostic sign of appendicitis? A. Rovsing sign B. Referred pain C. Chvostek's sign D. Rebound tenderness

A. Rovsing sign

Which urinalysis results most likely indicate glomerular damage? A. Smoky color; 30 mg/dL of protein; pH of 6.2 B. Cloudy, yellow; 50 WBCs/hpf; pH of 8.2; numerous casts C. Cloudy, brown; ammonia odor; specific gravity of 1.030; 3 RBCs/hpf. D. Clear, colorless; trace of glucose; trace of ketones; osmolality of 500 mOsm/kg (500 mmol/kg) Rationale The clinical manifestations of acute poststreptococcal glomerulonephritis (APSGN) appear as a variety of signs and symptoms, including generalized body edema, hypertension, oliguria, hematuria with a smoky or rusty appearance, and proteinuria may occur

A. Smoky color; 30 mg/dL of protein; pH of 6.2

Which urinalysis results most likely indicate glomerular damage? A. Smoky color; 30 mg/dL of protein; pH of 6.2 B. Cloudy, yellow; 50 WBCs/hpf; pH of 8.2; numerous casts C. Cloudy, brown; ammonia odor; specific gravity of 1.030; 3 RBCs/hpf. D. Clear, colorless; trace of glucose; trace of ketones; osmolality of 500 mOsm/kg (500 mmol/kg)

A. Smoky color; 30 mg/dL of protein; pH of 6.2 The clinical manifestations of acute poststreptococcal glomerulonephritis (APSGN) appear as a variety of signs and symptoms, including generalized body edema, hypertension, oliguria, hematuria with a smoky or rusty appearance, and proteinuria may occur.

LECTURE: Which of the following drugs are used for the temporary treatment of hyperkalemia (select all)? A. Sodium polystyrene sulfonate (Kayexalate) B. Calcium acetate (PhosLo) C. IV glucose and insulin D. Sevelamer (Renagel) E. Sodium bicarbonate

A. Sodium polystyrene sulfonate (Kayexalate) C. IV glucose and insulin E. Sodium bicarbonate DICKS-D10W, Insulin, Calcium gluconate, Kayelaxate, Sodium Bicarbonate D. Renegel is a phosphate binder for hyperphsophatemia/hypocalcemia NOT hyperkalemia B. Calcium GLUCONATE is used for hyperkalemia, NOT Calcium ACETATE. Calcium acetate (PhosLo) was used for hyperphosphatemia/hypocalcemia but they can cause complexes to form and harden the arteries so now they treat the hyperphosphatemia instead of the hypocalcemia.

How do you determine that a patient's oliguria is associated with acute renal failure (ARF)? A. Specific gravity of urine at 3 different times is 1.010. B. The serum creatinine level is normal. C. The blood urea nitrogen (BUN) level is normal or below. D. Hypokalemia is identified.

A. Specific gravity of urine at 3 different times is 1.010. A urinalysis may show casts, red blood cells (RBCs), white blood cells (WBCs), a specific gravity fixed at about 1.010, and urine osmolality at about 300 mOsm/kg

The nurse should question an order for glucocorticoids in the treatment of a patient with what? A. Systemic fungal infection B. Diabetes mellitus C. Myasthenia gravis D. Glaucoma

A. Systemic fungal infection Glucocorticoids are contraindicated in the treatment of a patient with a systemic fungal infection or in patients receiving live vaccines. Glucocorticoids should be used with caution in patients with diabetes mellitus, myasthenia gravis, and glaucoma.

Which manifestations should a nurse investigate first when monitoring a patient who is taking levothyroxine (Synthroid)? A. Tachycardia B. Tremors C. Insomnia D. Irritability

A. Tachycardia -A pulse or heart rate greater than 100 beats/min must be reported promptly. High doses of levothyroxine may cause thyrotoxicosis, a condition of profound excessive thyroid activity. Tachycardia is the priority assessment, because it can lead to severe cardiac dysfunction. Tremors, insomnia, and irritability are other symptoms of thyrotoxicosis and should be assessed after tachycardia. ALL of these symptoms must be reported promptly.

Which manifestations should a nurse investigate first when monitoring a patient who is taking levothyroxine (Synthroid)? A. Tachycardia B. Tremors C. Insomnia D. Irritability

A. Tachycardia High doses of levothyroxine may cause thyrotoxicosis, a condition of profound excessive thyroid activity. Tachycardia is the priority assessment, because it can lead to severe cardiac dysfunction. Tremors, insomnia, and irritability are other symptoms of thyrotoxicosis and should be assessed after tachycardia.

A patient who is on corticosteroid therapy for treatment of an autoimmune disorder has the following additional drugs ordered. How is the need for these drugs related to the effects of corticosteroids? a. Furosemide (Lasix) b. Pantoprazole (Protonix) c. Alendronate (Fosamax) d. Insulin e. Potassium

A. This treats the sodium and fluid retention because of mineralocorticoid effect. Can worsen hypokalemia though. B. This is a proton pump inhibitor. It helps with the gastrointestinal (GI) irritation, increase in secretion of pepsin and increased hydrochloric acid in the stomach. C. This helps treat corticosteroid-induced osteoporosis due to increased calcium excretion. D. Insulin helps with the glucose intolerance with hyperglycemia. Increased amounts of insulin are usually required. E. Potassium is needed to supplement for the hypokalemia because of increased secretion due to the mineralocorticoid effect of aldosterone

What is one of your most important roles in relation to APSGN? A. To promote early diagnosis and treatment of sore throats and skin lesions B. To encourage patients to request antibiotic therapy for all upper respiratory infections C. To teach patients with APSGN that long-term prophylactic antibiotic therapy is necessary to prevent recurrence D. To monitor patients for respiratory symptoms that indicate that the disease is affecting the alveolar basement membrane Rationale

A. To promote early diagnosis and treatment of sore throats and skin lesions APSGN develops 5 to 21 days after an infection of the tonsils, pharynx, or skin (e.g., streptococcal sore throat, impetigo) by nephrotoxic strains of group A β-hemolytic streptococci. One of the most important ways to prevent APSGN is to encourage early diagnosis and treatment of sore throats and skin lesions.

What is one of your most important roles in relation to APSGN? A. To promote early diagnosis and treatment of sore throats and skin lesions B. To encourage patients to request antibiotic therapy for all upper respiratory infections C. To teach patients with APSGN that long-term prophylactic antibiotic therapy is necessary to prevent recurrence D. To monitor patients for respiratory symptoms that indicate that the disease is affecting the alveolar basement membrane Rationale

A. To promote early diagnosis and treatment of sore throats and skin lesions APSGN develops 5 to 21 days after an infection of the tonsils, pharynx, or skin (e.g., streptococcal sore throat, impetigo) by nephrotoxic strains of group A β-hemolytic streptococci. One of the most important ways to prevent APSGN is to encourage early diagnosis and treatment of sore throats and skin lesions.

You are caring for a patient admitted with an exacerbation of asthma. After several treatments, the ABG results are pH 7.40, PaCO2 40 mm Hg, HCO3 24 mEq/L, PaO2 92 mm Hg, O2 saturation 99%. You interpret these results as which of the following? A. Within normal limits B. Slight metabolic acidosis C. Slight respiratory acidosis D. Slight respiratory alkalosis

A. Within normal limits The normal pH is 7.35 to 7.45. Normal PaCO2 levels are 38 to 48 mm Hg and HCO3 is 22 to 26 mEq/L. Normal PaO2 is >80 mm Hg. Normal oxygen saturation is >95%. Since the patient's results all fall within these normal ranges, the nurse can conclude that the patient's blood gas results are within normal limits.

A patient has a total serum calcium level of 3 mg/dL (1.5 mEq/L). If this finding reflects hypoparathyroidism, you would expect further diagnostic testing to reveal A. decreased serum PTH level. B. increased serum ACTH level. C. increased serum glucose level. D. decreased serum cortisol level. Rationale

A. decreased serum PTH level. A normal serum calcium level is 8.6 to 10.2 mg/dL. A low serum calcium level is found in patients with hypoparathyroidism, and the serum PTH level is decreased.

To help reduce the incidence of acute glomerulonephritis, public health teaching can stress the importance of A. early treatment of strep throat. B. obtaining a yearly screening urinalysis. C. proper urinary hygiene to prevent cystitis. D. early immunizations against streptococcal pneumonia. Rationale One of the most important ways to prevent APSGN is to encourage early diagnosis and treatment of sore throats and skin lesions. If streptococci are found in the culture, treatment with appropriate antibiotic therapy (usually penicillin) is essential. The patient must be encouraged to take the full course of antibiotics to ensure that the bacteria have been eradicated.

A. early treatment of strep throat.

To help reduce the incidence of acute glomerulonephritis, public health teaching can stress the importance of A. early treatment of strep throat. B. obtaining a yearly screening urinalysis. C. proper urinary hygiene to prevent cystitis. D. early immunizations against streptococcal pneumonia. Rationale

A. early treatment of strep throat.

When caring for a patient with primary hyperaldosteronism, the nurse would question a health care provider's order for the use of? A. furosemide (Lasix) B. amiloride (Midamor) C. spironolactone (Aldactone) D. aminoglutethimide (Cytadren)

A. furosemide (Lasix) Aldosterone causes excretion of potassium, resulting in hypokalemia. Lasix also causes hypokalemia.

The advantage of continuous replacement therapy over hemodialysis is its ability to A. remove fluid without the use of a dialysate. B. remove fluid in less than 24 hours. C. allow the patient to receive the therapy at the work site. D. be administered through a peripheral line.

A. remove fluid without the use of a dialysate. Several features of continuous replacement therapy are different from those of hemodialysis. Solute removal can occur by convection (no dialysate required) in addition to osmosis and diffusion. The process can take days or weeks. The patient cannot receive the therapy at work and a vascular access device is required.

Changes in serum pH affect ionized calcium levels Acidosis=____ionized calcium Alkalosis=____ ionized calcium

Acidosis=more ionized calcium Alkalosis=less ionized calcium *think Al-Kal-losis=Ionic Kalcium-Losses

The most common cause of Acute Kidney Injury is ______ while the most common cause of Chronic Kidney Injury is _______ ?

Acute Tubular Necrosis (ATN), Diabetic Nephropathy

Occurs when the adrenal glands produce an insufficient amount of corticosteroid?

Adrenal Insufficiency

A patient who is anxious starts to hyperventilate and develops tetany. Why? What do you do to treat this?

Alkalosis->less ionized calcium->Tetany (sign of hypocalcemia) breathing out of paper bag is the treatment. It increases CO2 which makes the patient more acidotic and causes the ionized calcium levels to rise.

All patients with documented gastric/duodenal ulcers and h.pylori should receive _____ therapy?

Antibiotic therapy

The triple drug therapy for PUD includes 2 ____ 's and a ___?

Antibiotics, PPI *TRIPLE DRUG THERAPY:* -Amoxicillin -Clarithromycin -Proton pump inhibitor

During the oliguric phase of AKI, the nurse monitor the patient for (select all that apply) a. Hypotension b. ECG changes c. Hypernatremia d. Pulmonary edema e. Urine with high specific gravity

B, C Why not A? there is HYPERtension from fluid overload. Why not C? There is fluid overload so dilution hyponatremia would occur Why not E? The body is producing urine but not concentrating it well in the oliguric stage of internal failure. *The Urine Specific Gravity is fixed at 1.010* Prerenal and postrenal failure leads to intrarenal failure, which causes a different set of physiological problems.

The teaching plan for the patient being discharged following an acute episode of upper GI bleeding will include information concerning the importance of (select all that apply)? a. Only taking aspirin with milk or bread products b. Avoiding taking aspirin and drugs containing aspirin c. Taking only drugs prescribed by the health care provider d. Taking all drugs 1 hour before mealtime to prevent further bleeding e. Reading all OTC drug labels to avoid those containing stearic acid and calcium

B, C b. Avoiding taking aspirin and drugs containing aspirin c. Taking only drugs prescribed by the health care provider

A nurse is reinforcing teaching with a client who has been prescribed levothyroxine (Synthroid) to treat hypothyroidism. Which of the following should the nurse include in the teaching? (Select all that apply.) A. Weight gain is expected while taking this medication. B. Medication should not be discontinued without the advice of the provider. C. Follow-up serum TSH levels should be obtained. D. Take the medication on an empty stomach. E. Use fiber laxatives for constipation.

B, C, D

A nurse is caring for a client who has Addison's disease and is taking hydrocortisone (Cortef). Which of the following medication instructions is appropriate for the nurse to include? (Select all that apply.) A. Take the medication on an empty stomach. B. Notify the provider of any illness or stress. C. Report any symptoms of weakness or dizziness. D. Do not discontinue the medication suddenly. E. Eat a low-sodium diet

B, C, D NOT A-take with food to reduce risk of peptic ulcers NOT E-patinets with addisons are hyponatremic

A nurse is admitting a client who has acute adrenal insufficiency to the intensive care unit. Which of the following prescriptions should the nurse anticipate? (Select all that apply.) A. IV therapy with 0.45% sodium chloride B. Regular insulin C. Hydrocortisone sodium succinate (Solu-Cortef) D. Sodium polystyrene sulfonate (Kayexalate) E. Furosemide (Lasix)

B, C, D, E Why not A? People with adrenal insufficiency are hyponatremic, so isotonic solution is the best way. B-Patient's with adrenal insufficiency are hyperkalemic so insulin will help reduce that. C-This will provide replacement therapy D-Kayexalate absorbs potassium, reducing hyperkalemia E-Diuretics waste potassium, reducing hyperkalemia NOTE: these are similar to the drugs that are used to treat hyperkalemia in the oliguric stage of kidney failure-rememeber the DICKS mnemonic. K-ayexal-ate-backwards is ate-K absorbs K+

Which of the following conditions causes PRERENAL acute renal failure? A. Benign prostatic hyperplasia B. Extensive burns C. Gun shot injury with hemorrhage D. Myocardial infarction with subsequence heart failure E. Renal calculi F. Contrast Dye G. Acute glomerulonephritis H. Septic shock

B, C, D, H (impaired blood flow to the kidneys) A-postrenal (obstruction distal to the kidney) E-postrenal (obstruction distal to the kidney) F-Intrarenal (nephrotoxic drugs, which damage glomeruli) G-Intrarenal (damage to the glomeruli or the tubules) KNOW FOR EXAM: causes of prerenal, intrarenal and postrenal kidney failure

A nurse is reviewing the clinical manifestations of hyperthyroidism with a client. Which of the following findings should the nurse include? (Select all that apply.) A. Dry skin B. Heat intolerance C. Constipation D. Palpitations E. Weight loss F. Bradycardia

B, D, E

Important nursing intervention(s) when caring for a patient with Cushing syndrome include (select all that apply)? a. restricting protein intake. b. monitoring blood glucose levels. c. observing for signs of hypotension. d. administering medication in equal doses. e. protecting patient from exposure to infection.

B, E

The patient is prescribed oral prednisone. Which statement indicates the patient correctly understands the teaching regarding the medication? A. "I will take my pulse before taking the drug and withhold it if the pulse is above 100 beats/minute." B. "I will take the medication in the morning with food." C. "I will carry glucose tablets with me in case I get a reaction from the medication." D. "I need to take the pill with a full glass of water and remain upright for 1 hour."

B. "I will take the medication in the morning with food." Rationale *The medication is given in the morning, usually with food to mimic the body's normal secretion of corticosteroids and prevent gastric irritation*. It is not necessary to monitor the pulse before administration; the drug is taken regardless of the pulse value. Hyperglycemia, not hypoglycemia, is the common response. The last option is the instruction given to patients taking bisphosphonates such as alendronate.

Which of the following statements by the nurse regarding continuous ambulatory peritoneal dialysis (CAPD) would be of highest priority when teaching a patient new to this procedure? A. "You will be allowed a more liberal protein diet once you complete CAPD." B. "It is essential that you maintain aseptic technique to prevent peritonitis." C. "It is important for you to maintain a daily written record of blood pressure and weight." D. "You will need to continue regular medical and nursing follow-up visits while performing CAPD."

B. "It is essential that you maintain aseptic technique to prevent peritonitis Peritonitis is a potentially fatal complication of peritoneal dialysis, and thus it is imperative to teach the patient methods of preventing this from occurring. Although the other teaching statements are accurate, they do not have the potential for mortality as does the peritonitis, thus making that nursing action of highest priority.

A 61-year-old patient with suspected bowel obstruction has had a nasogastric tube inserted at 4:00 am. The nurse shares in the morning report that the day shift staff should check the tube for patency at which of the following times? A. 7:00 am, 10:00 am, and 1:00 pm B. 8:00 am and 12:00 pm C. 9:00 am and 3:00 pm D. 9:00 am, 12:00 pm, and 3:00 pm

B. 8:00 am and 12:00 pm A nasogastric tube should be checked for patency routinely at 4-hour intervals. Thus if the tube were inserted at 4:00 am, it would be due to be checked at 8:00 am and 12:00 pm.

Which patient is at greatest risk for Goodpasture syndrome? A. An older adult with a history of urinary tract infections (UTIs) B. A young adult man who smokes C. A young adult woman who takes contraceptives D. An older adult male with benign prostatic hypertrophy Rationale

B. A young adult man who smokes Goodpasture syndrome is a rare disease that is seen mostly in young male smokers. The clinical manifestations include flu-like symptoms with pulmonary symptoms that include cough, mild shortness of breath, hemoptysis, crackles, rhonchi, and pulmonary insufficiency. Renal involvement causes hematuria, weakness, pallor, anemia, and renal failure. Pulmonary hemorrhage usually occurs and may precede glomerular abnormalities by weeks or months. Abnormal diagnostic findings include low hematocrit and hemoglobin levels, elevated BUN and serum creatinine levels, hematuria, and proteinuria. Circulating serum anti-GBM antibodies parallel the activity of the renal disease and are diagnostic of this syndrome.

Which approach should a nurse take to administer sucralfate (Carafate) to a patient with a duodenal ulcer? A. Crush the tablet into a fine powder before mixing it with water. B. Administer the tablet with sips of water 1 hour before meals. C. Allow the tablet to dissolve in water before administering it. D. Administer the tablet with an antacid for maximum benefit.

B. Administer the tablet with sips of water 1 hour before meals. Sucralfate acts through a compound that is a sticky gel, which adheres to an ulcer crater, creating a barrier to back-diffusion. The drug is best taken on an empty stomach. The tablet form does not dissolve in water when crushed, and crushing it could reduce the effectiveness of the drug. *Sucralfate acts under mildly acidic conditions; antacids raise the gastric pH above 4 and may interfere with the effects of sucralfate.*

Adrenocorticotropin hormone (ACTH) and Thyroid Stimulating Hormone (TSH) are products of which structure? A. Hypothalamus B. Anterior pituitary gland C. Posterior pituitary gland D. Thyroid gland E. Adrenal gland

B. Anterior pituitary gland A-The only thing synthesized in the hypothalamus is ADH, which is made in the neuro-secretory cells and then released from their axon terminals in the posterior pituitary. C-ADH and oxytocin are the only things that are released from the posterior pituitary D-This is the target organ of TSH, triggering the release of T3 and T4 E-This is the target organ of ACTH, triggering the release of glucocorticoids (cortisol) and mineralocorticoids (aldosterone).

A patient with a history of end-stage renal disease (ESRD) resulting from diabetes mellitus has presented to the outpatient dialysis unit for his scheduled hemodialysis. Which assessment should you prioritize before, during, and after his treatment? A. Level of consciousness B. Blood pressure and fluid balance C. Temperature, heart rate, and blood pressure D. Assessment for signs and symptoms of infection

B. Blood pressure and fluid balance Although all of the assessments are relevant to the care of a patient receiving hemodialysis, the nature of the procedure indicates a particular need to monitor blood pressure and fluid balance.

A patient with a history of end-stage renal disease secondary to diabetes mellitus has presented to the outpatient dialysis unit for his scheduled hemodialysis. Which of the following assessments should the nurse prioritize before, during, and after his treatment? A. Level of consciousness B. Blood pressure and fluid balance C. Temperature, heart rate, and blood pressure D. Assessment for signs and symptoms of infection

B. Blood pressure and fluid balance Although all of the assessments are relevant to the care of a patient receiving hemodialysis, the nature of procedure indicates a particular need to monitor patients' blood pressure and fluid balance.

What are the immunologic mechanisms involved in glomerulonephritis? A. Tubular blocking by precipitates of bacteria and antibody reactions B. Deposition of immune complexes and complement along the glomerular basement membrane (GBM) C. Thickening of the GBM from autoimmune microangiopathic changes D. Destruction of glomeruli by proteolytic enzymes contained in the GBM Rationale

B. Deposition of immune complexes and complement along the glomerular basement membrane (GBM) All forms of immune complex disease are characterized by an accumulation of antigen, antibody, and complement in the glomeruli, which can result in tissue injury. The immune complexes activate complement. Complement activation results in the release of chemotactic factors that attract polymorphonuclear leukocytes, histamine, and other inflammatory mediators. The result of these processes is glomerular injury.

Which finding in a patient taking levothyroxine (Synthroid) and warfarin (Coumadin) would require follow-up by a nurse? A. Cardiac dysrhythmias B. Excessive bruising C. Weight loss of 5 kg D. Shortness of breath

B. Excessive bruising Levothyroxine intensifies the effect of warfarin, an anticoagulant that increases the patient's risk for bleeding. The warfarin dose may need to be reduced. Bruising, weight loss, and shortness of breath are not effects associated with interactions of levothyroxine and warfarin.

The patient who is admitted with a diagnosis of diverticulitis and a history of irritable bowel disease and gastroesophageal reflux disease (GERD) has received a dose of Mylanta 30 ml PO. The nurse would evaluate its effectiveness by questioning the patient as to whether which of the following symptoms has been resolved? A. Diarrhea B. Heartburn C. Constipation D. Lower abdominal pain

B. Heartburn Mylanta is an antacid that contains both aluminum and magnesium. It is indicated for the relief of GI discomfort, such as with heartburn associated with GERD.

Which assessment finding is commonly found in the oliguric phase of acute kidney injury (AKI)? A. Hypovolemia B. Hyperkalemia C. Hypernatremia D. Thrombocytopenia

B. Hyperkalemia

Which assessment finding is commonly found in the oliguric phase of acute kidney injury (AKI)? A. Hypovolemia B. Hyperkalemia C. Hypernatremia D. Thrombocytopenia

B. Hyperkalemia In AKI, the serum potassium levels increase because the normal ability of the kidneys to excrete potassium is impaired. Sodium levels are typically normal or diminished, whereas fluid volume is normally increased due to decreased urine output. Thrombocytopenia is not a consequence of AKI, although altered platelet function may occur in AKI

In addition to urinary symptoms, patients with APSGN may also present with which symptom? A. Rash over the upper trunk B. Hypertension C. Halos around lights D. Disorientation

B. Hypertension Rationale Hypertension primarily results from increased extracellular fluid volume.

To normalize a low serum calcium level, the body releases parathyroid hormone (PTH); this results in which therapeutic effect? A. Decrease in the intestinal absorption of calcium B. Increase in bone resorption of calcium C. Increase in renal calcium excretion D. Increase in plasma levels of phosphate

B. Increase in bone resorption of calcium PTH restores normal calcium levels by promoting calcium resorption from bone and transferring it to the blood. PTH also activates vitamin D, which increases, not decreases, calcium absorption from the intestine. Renal losses of calcium are reduced by PTH. PTH reduces plasma levels of phosphate. Calcium and phosphate are inversely related so that when PTH increases serum calcium it also decreases serum potassium.

What is the action of calcitonin produced by the thyroid gland? A. Increases absorption of calcium B. Inhibits calcium resorption C. Regulates T3 and T4 secretion D. Stimulates the secretion of oxytocin

B. Inhibits calcium resorption Calcitonin is a hormone produced by C cells of the thyroid gland in response to high circulating calcium levels. Calcitonin inhibits calcium resorption (loss of substance) for bone, increases calcium storage in bone, and increases renal excretion of calcium and phosphorus, thereby lowering serum calcium levels. The other options are incorrect.

A patient with a head injury develops SIADH. What symptoms do you expect to find? A. Hypernatremia and edema B. Low urinary output and thirst C. Muscle spasticity and hypertension D. Weight gain and decreased glomerular filtration rate Rationale

B. Low urinary output and thirst

You are admitting a patient with complaints of abdominal pain, nausea, and vomiting. A bowel obstruction is suspected. You assess this patient for which of the following anticipated primary acid-base imbalances if the obstruction is high in the intestine? A. Metabolic acidosis B. Metabolic alkalosis C. Respiratory acidosis D. Respiratory alkalosis

B. Metabolic alkalosis (Because gastric secretions are rich in hydrochloric acid, the patient who is vomiting will lose a significant amount of gastric acid and be at an increased risk for metabolic alkalosis)

You are caring for a patient admitted with a diagnosis of COPD who has the following arterial blood gas results: pH 7.33, PaO2 47 mm Hg, PaCO2 60 mm Hg, HCO3 32 mEq/L, and O2 saturation of 92%. Which of the following is the correct interpretation of these results? A. Fully compensated respiratory alkalosis B. Partially compensated respiratory acidosis C. Normal acid-base balance with hypoxemia D. Normal acid-base balance with hypercapnia

B. Partially compensated respiratory acidosis Fully compensated means pH is back to normal. A low pH (normal 7.35-7.45) indicates acidosis. In the patient with respiratory disease such as COPD, the patient retains carbon dioxide (normal 38-48 mm Hg), which acts as an acid in the body. For this reason, the patient has respiratory acidosis. The elevated HCO3 indicates a partial compensation for the elevated CO2.

Which finding is important for you to report in a patient with primary Addison's disease? A. Bronze discoloration over the joints B. Peaked T waves on the electrocardiogram (ECG) C. Irritability D. Hoarse voice

B. Peaked T waves on the electrocardiogram (ECG) Hyperkalemia can be a significant abnormal laboratory finding for patients with Addison's disease. Hyperkalemia can cause peaked T waves, and it is most significantly related to effects on cardiac muscle. Bronzing or hyperpigmentation over sun-exposed areas, including joints, is an expected finding in primary Addison's disease. It is most likely caused by increased secretion of β-lipotropin, which contains melanocyte-stimulating hormone. Irritability is an expected finding. Hoarse voice is usually associated with hypothyroidism.

When caring for a patient with acute kidney injury who is hyperkalemic, which of the following prescribed actions should the nurse take first? A. AdministerSevelamer (Renagel) B. Place the patient on a cardiac monitor. C. Administer sodium polystyrene sulfonate (Kayexalate). D. Look in the EHR for the patient's current blood urea nitrogen (BUN) and creatinine levels.

B. Place the patient on a cardiac monitor *ALWAYS Assess BEFORE implementing!!!!! Hyperkalemia is >5.0 mEq/L and >6.0 causes cardiac dysrhythmias. The patients cardiac rhythm should be monitored.

A nurse is providing instructions to a client who has Graves' disease and has a new prescription for propranolol (Inderal). Which of the following information should the nurse include? A. An adverse effect of this medication is jaundice. B. Take your pulse before each dose. C. The purpose of this medication is to decrease production of thyroid hormone. D. You should stop taking this medication if you have a sore throat

B. Propranolol can cause bradycardia. The client should take his pulse before each dose. If there is a significant change, he should withhold the dose and consult his provider.

The patient had pituitary surgery yesterday. Which symptom is most important for you to monitor? A. Urine specific gravity: 1.005 B. Voids 10 L/day C. Crackles auscultated in lung bases D. Temperature: 100.4° F (38° C)

B. Voids 10 L/day Diabetes insipidus (DI) is a deficiency of production or secretion of antidiuretic hormone (ADH) or a decreased renal response to ADH. It is characterized by polydipsia (5 to 20 L/day) with low specific gravity (less than 1.005). The last two options are more likely related to atelectasis and are less important than DI.

A nurse is providing discharge instructions to a client who had a transsphenoidal hypophysectomy. Which of the following instructions should the nurse include? (Select all that apply.) A. Brush teeth after every meal or snack. B. Avoid bending at the knees. C. Eat a high-fiber diet. D. Notify the provider if he has sweet-tasting drainage. E. Notify the provider if he has diminished sense of smell.

C and D A-avoid brushing teeth to allow wound to heal B-avoid bending at waist not knees C-this helps to avoid constipation, which contributes to ICP D-CSF contains glucose E-expected

A nurse is assessing a client who is 12 hr postoperative following a thyroidectomy. Which of the following findings are indicative of thyroid crisis? (Select all that apply.) A. Bradycardia B. Hypothermia C. Tremors D. Abdominal pain E. Mental confusion

C, D, E A-Tachycardia B-Hypothermia

A nurse teaches a patient who has ulcerative colitis about the side effects of the treatment medication, sulfasalazine (Azulfidine). Which statement by the patient would indicate understanding of the information? A. "My tongue may become discolored and my taste altered." B. "I may have constipation, so I'll increase my fluid intake." C. "I'll report any fatigue or sore throat and fever to my doctor." D. "I'll immediately report any chest pain or shortness of breath."

C. "I'll report any fatigue or sore throat and fever to my doctor."

A patient with Graves' disease is treated with iodine-131 (Iodope) therapy. Which statement by the patient would indicate understanding of the treatment's effects? A. "I'll have to isolate myself from my family so I don't expose them to radiation." B. "I'm looking forward to feeling better immediately after this treatment." C. "I'll tell my doctor if I have fatigue, hair loss, or cold intolerance." D. "I'll need to take this drug on a daily basis for at least 1 year."

C. "I'll tell my doctor if I have fatigue, hair loss, or cold intolerance." Iodine-131 usually is given as a single treatment to produce remission of Graves' disease. Fatigue, hair loss, and cold intolerance are signs of hypothyroidism, which is a complication of the treatment. Iodine-131 has a quick radioactive decay and half-life; therefore, isolation is not needed, but it can take up to 2 months for the desired response to develop.

The patient in the oliguric phase of AKI excreted 300 mL of urine in addition to 100 mL of other losses during the past 24 hours. With appropriate calculations, you determine that for the next 24 hours the patient's fluid allocation is? A. 600 mL. B. 800 mL. C. 1000 mL. D. 1200 mL.

C. 1000 mL. Fluid intake must be closely monitored during the oliguric phase. The rule for calculating the fluid restriction is to add all losses for the previous 24 hours to 600 mL for insensible losses. 400 (losses) + 600=1000 mL

When planning care for adult patients, you conclude that which of the following oral intakes is adequate to meet daily fluid needs of a stable patient? A. 500 to 1500 ml B. 1200 to 2200 ml C. 2000 to 3000 ml D. 3000 to 4000 ml

C. 2000 to 3000 ml Daily fluid intake and output is usually 2000 to 3000 ml. This is sufficient to meet the needs of the body and replace both sensible and insensible fluid losses. These would include urine output and fluids lost through the respiratory system, skin, and GI tract. 2-3 Liters a day

A nurse is planning to teach a client who is being evaluated for Addison's disease about the adrenocorticotropic hormone (ACTH) stimulation test. The nurse should base her instructions to the client on which of the following? A. The ACTH stimulation test measures the response by the kidneys to ACTH. B. In the presence of primary adrenal insufficiency, plasma cortisol levels rise in response to administration of ACTH. C. ACTH is a hormone produced by the pituitary gland. D. The client is instructed to take a dose of ACTH by mouth the evening before the test.

C. ACTH is a hormone produced by the pituitary gland. Why not A? The ACTH stimulation test measures the response by the adrenal glands NOT kidneys to ACTH. They are located on top of the kidneys. Why not B? In the presence of primary adrenal insufficiency the adrenal glands do not produce enough cortisol even when ACTH is present. Its the glands themselves that are damaged. Why not D? Measured during process.

A nurse should consider which diagnostic test a priority to obtain before a patient receives iodine-131 (Iodotope)? A. White blood cell (WBC) count B. Electrocardiogram (ECG) C. Beta human chorionic gonadotropin (hCG) test D. Creatinine level

C. Beta human chorionic gonadotropin (hCG) test Any female patient of reproductive age requires a negative result on a beta hCG (pregnancy hormone) test before iodine-131 (131I) can be administered. 131I is a radioactive isotope used to treat hyperthyroidism and is contraindicated in pregnancy and lactation. A WBC count, ECG, and creatinine level are not indicated before treatment with iodine-131.

Which is a risk for developing hemorrhoids? A. Body mass index (BMI) of 17 kg/m2 B. Younger than 30 years C. Chronic constipation D. History of lactase deficiency

C. Chronic constipation Chronic constipation increases intraluminal pressure, which can predispose to hemorrhoids. Overweight is a risk, not underweight (normal BMI is 18.5 to 24.9 kg/m2). Older adults are at higher risk. Lactase deficiency is not related to hemorrhoids.

All of the laboratory results for a patient are significantly high. Which one is most important for you to respond to? A. Thyroid-stimulating hormone (TSH) B. Growth hormone C. Free T4 (thyroxin) D. Phosphate

C. Free T4 (thyroxin) An excess of thyroid hormone can be life threatening because of the cardiac effects. The patient may be having a thyroid storm. An elevated TSH level indicates the patient has hypothyroidism. Growth hormone produces acromegaly with bone and soft tissue abnormalities, but the high level is not an emergent finding. The phosphate level is also not life threatening.

A nurse is caring for a client who is 6 hr postoperative following a transsphenoidal hypophysectomy. The nurse should test the client's nasal drainage for the presence of which of the following? A. RBCs B. Ketones C. Glucose D. Streptococcus

C. Glucose (is in CSF)

LECTURE: The most appropriate snack for the nurse to order for a patient with acute kidney injury is: A. Raisins B. Yogurt C. Graham crackers D. Slices of honeydew melon

C. Graham crackers In the oliguric phase you would want foods that are LOW in phosphate due to the hyperphosphatemia/hypocalcemia. You wouldn't want anything with high protein due to Azotemia in the oliguric phase.

Which is the best category of food to encourage prevention of diverticulosis? A. High iron, such as organ meats B. Low gluten C. High fiber, such as raw vegetables D. No nuts or popcorn

C. High fiber, such as raw vegetables

If a patient is in the diuretic phase of AKI, the nurse must monitor for which serum imbalances? A. Hyperkalemia and hyperphosphatemia B. Hyperkalemia and azotemia C. Hypokalemia and hypomagnesemia D. Hypokalemia and metabolic acidosis

C. Hypokalemia and hypomagnesemia

LECTURE: If a patient is in the diuretic phase of AKI, the nurse must monitor for which serum imbalances? A. Hyperkalemia and hyperphosphatemia B. Hyperkalemia and azotemia C. Hypokalemia and hypomagnesemia D. Hypokalemia and metabolic acidosis

C. Hypokalemia and hypomagnesemia (A,B,D-oliguric phase)

If a patient is in the diuretic phase of AKI, you must monitor for which serum electrolyte imbalances? A. Hyperkalemia and hyponatremia B. Hyperkalemia and hypernatremia C. Hypokalemia and hyponatremia D. Hypokalemia and hypernatremia

C. Hypokalemia and hyponatremia *In the diuretic phase of AKI, the kidneys have recovered their ability to excrete wastes but not to concentrate the urine. Hypovolemia and hypotension can result from massive fluid losses. Because of the large losses of fluid and electrolytes, the patient must be monitored for hyponatremia, hypokalemia, and dehydration.*

You are caring for a patient admitted with suspected hyperparathyroidism. Because of the potential effects of this disease on electrolyte balance, for what should you assess this patient? A. Neurologic irritability B. Declining urine output C. Lethargy and weakness D. Hyperactive bowel sounds

C. Lethargy and weakness Hyperparathyroidism can cause hypercalcemia. Signs of hypercalcemia include polyuria, constipation, nausea, vomiting, lethargy, and muscle weakness.

How should you assess the patency of a newly placed arteriovenous graft for dialysis? A. Irrigate the graft daily with low-dose heparin. B. Monitor for any increase in blood pressure in the affected arm. C. Listen with a stethoscope over the graft for presence of a bruit. D. Frequently monitor the pulses and neurovascular status distal to the graft. Rationale

C. Listen with a stethoscope over the graft for presence of a bruit. *A thrill can be felt by palpating the area of anastomosis of the arteriovenous graft, and a bruit can be heard with a stethoscope. The bruit and thrill are created by arterial blood rushing into the vein*

Which is the best food for the patient with lactase deficiency? A. Skim milk B. Low-fat ice cream C. Live-culture yogurt D. Cheddar cheese

C. Live-culture yogurt

The surgeon was unable to spare a patient's parathyroid gland during a thyroidectomy. Which of the following assessments should the nurse prioritize when providing postoperative care for this patient? A. Assessing the patient's white blood cell levels and assessing for infection B. Monitoring the patient's hemoglobin, hematocrit, and red blood cell levels C. Monitoring the patient's serum calcium levels and assessing for signs of hypocalcemia D. Monitoring the patient's level of consciousness and assessing for acute delirium or agitation

C. Monitoring the patient's serum calcium levels and assessing for signs of hypocalcemia Loss of the parathyroid gland is associated with hypocalcemia. Infection and anemia are not associated with loss of the parathyroid gland, whereas cognitive changes are less pronounced than the signs and symptoms of hypocalcemia. PTH-increases serum calcium levels when they are low so no PTH means calcium levels will be lower than normal.

Which is a risk of long-term Cushing disease? A. Polycythemia B. Pheochromocytoma C. Osteoporosis D. Rheumatoid arthritis

C. Osteoporosis Protein wasting is caused by the catabolic effects of cortisol on peripheral tissue. This leads to bone loss, which leads to osteoporosis and subsequent pathologic fractures. The other conditions are not related to Cushing disease.

A nurse is planning care for a client who has prerenal acute kidney injury following abdominal aortic aneurysm repair. The client's urinary output is 80 mL in the past 4 hr, and blood pressure is 92/58 mm Hg. Which of the following should be included in the plan of care? A. Prepare the client for a CAT scan with contrast dye. B. Anticipate urine specific gravity to be 1.010. C. Plan to administer a fluid challenge. D. Place client in Trendelenburg position

C. Plan to administer a fluid challenge. The nurse should plan to administer a fluid challenge for hypovolemia, which is indicated by the client's low urinary output and blood pressure. A-contrast dye is contraindicated in a person who has suspected acute kidney injury due to nephrotoxicity B-urine specific gravity will be 1.030 C-fluid challenge for hypovolemia, which is indicated by the client's low urinary output and blood pressure D-reverse trendelenburg, with feet up and head down

A patient has been taking oral prednisone for the past several weeks after having a severe reaction to poison ivy. The nurse has explained the procedure for gradual reduction rather than sudden cessation of the drug. What is the rationale for this approach to drug administration? A. Prevention of hypothyroidism B. Prevention of diabetes insipidus C. Prevention of adrenal insufficiency D. Prevention of cardiovascular complications

C. Prevention of adrenal insufficiency Sudden cessation of corticosteroid therapy can precipitate life-threatening secondary adrenal insufficiency from HPA suppression. Diabetes insipidus, hypothyroidism, and cardiovascular complications are not common consequences of stopping corticosteroid therapy suddenly.

The elderly patient reports lethargy, dry skin, constipation, and difficulty thinking. What is your first response? A. Teach about the normal changes of aging. B. Encourage more raw fruits and vegetables. C. Recommend testing for hypothyroidism. D. Recommend testing for low iodine.

C. Recommend testing for hypothyroidism. In the older adult, the typical manifestations of hypothyroidism are often mistakenly attributed to normal aging. Older adults who have confusion, lethargy, and depression should be evaluated for thyroid disease. That takes priority over teaching about normal processes or attempting treatment of constipation. Although low iodine levels can cause hypothyroidism, it has many other causes, and the condition itself should be ruled out. Low iodine levels are not a major problem in the United States due to the use of iodized salt.

The elderly patient reports lethargy, dry skin, constipation, and difficulty thinking. What is your first response? A. Teach about the normal changes of aging. B. Encourage more raw fruits and vegetables. C. Recommend testing for hypothyroidism. D. Recommend testing for low iodine. Rationale

C. Recommend testing for hypothyroidism. In the older adult, the typical manifestations of hypothyroidism are often mistakenly attributed to normal aging. Older adults who have confusion, lethargy, and depression should be evaluated for thyroid disease. That takes priority over teaching about normal processes or attempting treatment of constipation. Although low iodine levels can cause hypothyroidism, it has many other causes, and the condition itself should be ruled out. Low iodine levels are not a major problem in the United States due to the use of iodized salt.

The patient has SIADH with a serum sodium level of 128 mEq/L. What action do you anticipate? A. Increase sodium-rich foods. B. Rapidly infuse hypertonic intravenous (IV) fluids. C. Restrict fluids. D. Administer calcitonin.

C. Restrict fluids. When symptoms of SIADH are mild and the serum sodium level is more than 125 mEq/L, the only treatment may be restriction of fluids to 800 to 1000 mL per day. Severe hyponatremia (less than 120 mEq/L) may be treated with slow infusion of hypertonic saline. Calcitonin is used for hypercalcemia

A nurse is reinforcing teaching for a client with hyperthyroidism who has a new prescription for propranolol (Inderal). The nurse should teach the client that propranolol should perform which of the following actions? A. Increase blood flow to the thyroid gland B. Prevent thyroid hormone synthesis C. Suppress tachycardia D. Promote conversion of T4 to T3

C. Suppress tachycardia Propranolol is a beta-adrenergic antagonist that suppresses tachycardia, a common finding in hyperthyroidism. Propranolol lowers blood pressure, but does not increase blood flow to the thyroid gland. Propranolol does not have any effects on thyroid hormone synthesis.

You identify a risk for urinary calculi in a patient who relates a health history that includes? A. adrenal insufficiency. B. serotonin deficiency. C. hyperaldosteronism. D. hyperparathyroidism.

C. hyperaldosteronism. Excessive levels of circulating parathyroid hormone (PTH) usually lead to hypercalcemia and hypophosphatemia. In the kidneys excess calcium in the filtrate cannot be reabsorbed, leading to increased levels of calcium in the urine (hypercalciuria). This increased urinary calcium level along with a large amount of urinary phosphate can lead to calculi formation.

The most critical intervention in the prevention of renal calculi is for the patient to A. urinate frequently. B. eat a diet high in protein. C. maintain a high fluid intake. D. eliminate all calcium from the diet.

C. maintain a high fluid intake. The most important factor in the prevention of renal calculi is keeping urine dilute and free flowing. This reduces the risk of recurrent stone formation in many individuals. This is accomplished by drinking about 2000 to 2200 mL/day, with the residual 20% to 30% of fluids gained through consumption of foods. The volume of fluids is higher in the highly active patient who works outdoors or who regularly engages in demanding athletic activities.

You teach the patient that the best time to take corticosteroids for replacement purposes is? A. once each day at bedtime. B. every other day on awakening. C. on arising and in the late afternoon. D. at consistent intervals every 6 to 8 hours.

C. on arising and in the late afternoon. *Glucocorticoids are usually given in divided doses: two thirds in the morning and one third in the afternoon*. This dosage schedule reflects normal circadian rhythm in endogenous hormone secretion and decreases the side effects associated with corticosteroid replacement therapy.

CALCIUM/PARATHYROID

CALCIUM/PARATHYROID

What is the mnemonic for emergency treatment of hyperkalemia secondary to kidney failure?

Cardiac monitor then... DICKS: (D)-10w IV solution with (I)nsulin, (C)alcium gluconate (K)ayaxalate and (S)odium bicarbonate *Make sure they don't get potassium in their IV, such as lactated ringers

Why do glucocorticoid medications need to be taken with food?

Cortisol increases gastric acid secretion, which puts patients at *increased risk of peptic ulcers*. In order to mitigate the risks of gastric irritation and ulceration they should be taken with food.

What is the difference between Cushing syndrome and Cushing's Disease?

Cushing Syndrome: An array of symptoms caused by high levels of cortisol in the blood. *Prolonged exogenous exposure to high doses of glucocorticoid medications is the MOST COMMON CAUSE*. Can be caused by Cushing's disease. *Cushing's Disease is a pituitary tumor that causes excess secretion of ACTH*, resulting in increased secretion of cortisol from the adrenal cortex

An excess of corticosteroids is known as ______ and a deficit is known as _______ ?

Cushings, Adrenal Insufficiency (Addison's) Think CU-shings (Cortisol Up), ADD-i-son's-need to "add in some" cortisol-treated by adding LOW doses of corticosteroid medications

Describe what hormone each of these is associated with and whether there is too much or too little? Cushings Addisons Graves Hashimoto's thyroiditis Diabetes Insipidus Syndrome of Inappropriate Antidiuretic Hormone (SIADH)

Cushings-too much cortisol Addisons-too little cortisol Graves-too much thyroid hormone Hashimotos Thyroiditis-too little thyroid hormone Diabetes Insipidus-too little ADH SIADH-too much ADH

A patient has been given instructions about levothyroxine (Synthroid). Which statement by the patient indicates understanding of these instructions? A. "I'll be glad when I don't have to take this medication in a few months." B. "I'll plan to double my dose if I gain more than 1 pound per day." C. "It is best to take the medication with food so I don't have any nausea." D."I'll take this medication in the morning so as not to interfere with sleep."

D. "I'll take this medication in the morning so as not to interfere with sleep." Levothyroxine is used to treat hypothyroidism by increasing the basal metabolism and thus wakefulness. It is administered as a once-daily dose and is a lifelong therapy. It is best taken on an empty stomach to enhance absorption.

THIS WILL BE ON TEST: The nurse notes that a patient gained 2 lbs in 24 hours. This is equivalent to a fluid gain of approximately: A. 250 mL B. 500 mL C. 750 mL D. 1,000 mL

D. 1,000 mL THIS WILL BE ON TEST: 2.2 pounds=1 kg=1000 grams =1000mL REMEMBER THAT 1 mL=1 GRAM 2 pounds= .9 kg or 900 grams, which is 900 mL

A patient with CKD has fluid volume excess. When getting peritoneal dialysis, he would want to use a dextrose solution of? A. o.5% B. 1.5% C. 2.5% D. 4%

D. 4% (the more hypertonic the dextrose solution the more fluid it will pull off)

During a physical assessment, you notice that the patient has a visibly enlarged thyroid gland. What action should you take? A. Place your thumb horizontally with the upper edge along lover border of the cricoids cartilage. B. Stand behind patient, and use the index and middle fingers of both hands to feel the neck. C. Ask the patient to swallow water, and ask about discomfort. D. Document findings and delay palpation. Rationale

D. Document findings and delay palpation. Rationale Palpation can trigger the release of thyroid hormones. *Palpation should be delayed if the patient has a visibly enlarged thyroid gland.* The first option describes anterior palpation for a normal thyroid examination (*this is the normal procedure for a thyroid inspection when there is no goiter*), and the second option describes a posterior thyroid examination. Neither option should be done with a visibly enlarged gland. Swallowing water can be used during a normal thyroid examination, but is not necessary with an enlarged thyroid.

Which assessment finding is associated with renal calculi? A. Urinary bladder distention B. Hypertension C. Proteinuria D. Flank pain

D. Flank pain Clinical manifestations of renal calculi include abdominal or flank pain (typically severe), hematuria, and renal colic.

A nurse monitors the calcium level of a patient who has had a thyroidectomy. The nurse should notice which finding if the patient's calcium level is 7.5 mg/dL? A. Dull, aching bone pain B. Nausea and vomiting C. Lethargy and confusion D. Muscle twitching and tetany

D. Muscle twitching and tetany The normal calcium level is 8.6-10.2 mg/dL. This is hypocalcemia. Low calcium levels may be the result of inadvertent removal of the parathyroid gland during a thyroidectomy, leading to a lack of PTH. This produces hypocalcemia and symptoms of tetany, muscle twitching, and neuromuscular excitability. Nausea and vomiting, lethargy, and confusion are symptoms of hypercalcemia. Dull, aching bone pain may be associated with osteomalacia and vitamin D insufficiency.

The person with lactase deficiency is at risk for which condition? A. Colorectal cancer B. Pancreatitis C. Osteoarthritis D. Osteoporosis

D. Osteoporosis

You are caring for a patient receiving calcium carbonate for the treatment of osteopenia. Which of the following serum laboratory results would you identify as an adverse effect related to this therapy? A. Sodium falling to 138 mEq/L B. Potassium rising to 4.1 mEq/L C. Magnesium rising to 2.9 mg/dl D. Phosphorus falling to 2.1 mg/dl

D. Phosphorus falling to 2.1 mg/dl Calcium has an inverse relationship with phosphorus in the body. When phosphorus levels fall, calcium rises, and vice versa. Since hypercalcemia rarely occurs as a result of calcium intake, the patient's phosphorus falling to 2.1 mg/dl (normal 2.4-4.4 mg/dl), may be a result of the phosphate-binding effect of calcium carbonate.

When caring for a patient during the oliguric phase of acute kidney injury, which of the following would be an appropriate nursing intervention? A. Weigh patient three times weekly. B. Increase dietary sodium and potassium. C. Provide a low-protein, high-carbohydrate diet. D. Restrict fluids according to previous daily loss.

D. Restrict fluids according to previous daily loss. Patients in the oliguric phase of acute kidney injury will have fluid volume excess with potassium and sodium retention; hence, they will need to have dietary sodium, potassium, and fluids restricted. Daily fluid intake is based on the previous 24-hour fluid loss (measured output plus 600 ml for insensible loss). The diet also needs to provide adequate, not low, protein intake to prevent catabolism. The patient should also be weighed daily, not just three times a week.

What should a patient be taught after a hemorrhoidectomy? A. Do not use the Valsalva maneuver. B. Eat a low-fiber diet to rest the colon. C. Administer oil-retention enema to empty the colon. D. Use prescribed pain medication before a bowel movement.

D. Use prescribed pain medication before a bowel movement.

A patient's recent medical history is indicative of diabetes insipidus. The nurse would perform patient education related to which of the following diagnostic tests? A. Thyroid scan B. Fasting glucose test C. Oral glucose tolerance

D. Water deprivation test A water deprivation test is used to diagnose the polyuria that accompanies diabetes insipidus. Glucose tests and thyroid tests are not directly related to the diagnosis of diabetes insipidus.

LECTURE: The most accurate indicator of fluid loss or gain in an acutely ill patient is: A. Blood pressure B. The presence or absence of peripheral edema C. Serum sodium levels D. Weight changes

D. Weight changes

LECTURE QUESTION The nurse determines that the patient with oliguria has prerenal oliguria when: A. urinalysis reveals a low specific gravity B. urinalysis reveals casts C. the fractional excretion of sodium (FENa) is high D. the oliguria is reversed with fluid replacement.

D. the oliguria is reversed with fluid replacement.

The immunological mechansims involved in glomerulonephritis include?

Deposition of immune complexes and compliment along the GBM

A patient had a closed head injury. He has a foley catheter that was placed in the ER and you notice that the bag is full of clear light yellow urine. You empty 1,250 mL from the foley bag. Several chemistry labs were drawn in the ER: -Sodium level 149 mEq/L (reference range 135- 145 mEq/L) -Serum osmolality 306 mOsm/kg (reference range 275-295 mOsm/kg) What is the problem here? What would be the treatment for this?

Diabetes Insipidus (DI) ,not making AHD so large amounts of dilute urine and high serum osmolarity. The patient would receive a synthetic ADH called Desmopressin (DDVAP) and D5W (free water)-you wouldn't 1ml/hr for every 1 mL of urine loss. You would;t give sodium chloride or NS 0.9% (isotonic) because the patient already has hypernatremia.

The most common cause of CKD in the united states is_____, followed by _____?

Diabetes, Hypertension

_______ is indicated when diet and medications can no longer control the life-threatening effects of renal failure?

Dialysis Stage 5 CKD starts dialysis

A GFR of 15 mL/min or less is stage 5 CKD and is called?

ESRD-end stage renal disease

Hyperparathyroidism is treated with _________ and hypoparathyroidism is treated with ________?

Hyperparathyroidism: this is associated with Hypercalcemia. The initial treatment involves *increasing fluids to promote diuresis* and excretion of calcium + bisphosphonates to prevent bone resorption Hypoparathyroidism: *Vitamin D and Calcium supplements*

ON EXAM: Prolonged stress, such as being in the hospital for a surgery, triggers the release of cortisol from the _____ ?

Hypothalamic-Pituitary-Adrenal Axis

When calcium levels are high the thyroid gland secretes calcitonin to lower the serum level of calcium. How does it do that?

It inhibits bone resorption by osteoclasts and enhances calcium excretion from the kidneys

KIDNEYS

KIDNEYS

What causes the hyperphosphatemia and corresponding hypocalcemia during the oliguric phase of Acute Kidney Injury?

Kidney failure leads to a decreased GFR, resulting in decreased secretion of PO4 (phosphate). *Kidney failure also causes decreased activation of vitamin D, resulting in decreased absorption of calcium in the gut*, causing decreased serum calcium or hypocalcemia.

Levothyroxine (Synthroid) • When should it be administered? • Should it be administered with food? • What assessments should be performed before administering levothyroxine? • How long will it take before the patient will notice improvement in thyroid function? • What side effects should the patient report to the physician immediately? • Will the patient be on this medication for the rest of their life? See Table 50-11 in Lewis (2011) Patient and Caregiver Teaching Guide (Hypothyroidism)

Levothyroxine (Synthroid) • When should it be administered? In the morning at least 30-60 minutes before breakfast • Should it be administered with food? NO, absorption of oral Synthroid is reduced by food. Since it is almost always oral food should be AVOIDED. • What assessments should be performed before administering levothyroxine? Assess fro coronary artery disease. This drug can increase the oxygen demands of the heart. Start low and go slow with older adults. Drugs the patient is on: This drug increases the effects of Warfarin and increases the need for insulin and digoxin. • How long will it take before the patient will notice improvement in thyroid function? Lewis: -"levothyroxine has a peak of 1-3 weeks" - "TSH is tested 4-6 weeks after chnaging the dose -Lehne: -Long half-life means that it takes " about 1 month for the drug to plateau and reach steady state, delaying the full effect • What side effects should the patient report to the physician immediately? Lewis: Tachycardia, Chest pain, Weight Loss, nervousness, tremors and insomnia Lehne: thyrotoxicosis-tachycardia, angina, hyperthermia, tremor, nervousness, insomnia, heat intolerance, sweating-notify provider Weight loss only occurs when the dose is high enough to produce a pathological state (hyperthryoid), which is why its not used to treat obesity. • Will the patient be on this medication for the rest of their life? YES, "lifelong replacement therapy is usually required" and "emphasize the need for lifelong replacement" are on page 1271

In the change of shift report the night nurse tells you that your patient just had a coffee ground emesis. Does this indicate that the patient is experiencing active acute gastrointestinal bleeding? Why?

NOT ACUTE BECAUSE IT WOULD BE BRIGHT RED COLOR-COULD BE ACTIVE BUT NOT ACUTE. BLACK COFFEE GROUND EMESIS MEANS IT HAS BEEN ACTED ON BY DIGESTIVE ENZYMES THAT TURN IT BACK-COFFEE GROUND EMESIS

Calcium is regulated by two hormones: Thyrocalcitonin (calcitonin) and Parathyroid hormone (PTH) Which one causes calcium levels to go up? Which one causes calcium to go down?

PTH/Calcitonin

When calcium levels are low the parathyroid gland releases PTH to raise the calcium level. How does it do that?

PTH: -promotes bone resorption of calcium by osteoclasts -stimulates the kidney to conserve calcium and excrete phosphorus (important since having high levels of both would cause complexes) -activates vitamin D to enhance absorption of calcium in the gut

A patient is recovering in the intensive care unit (ICU) after receiving a kidney transplant approximately 24 hours ago. Which of the following is an expected assessment finding for this patient during this early stage of recovery? Hypokalemia Hyponatremia Large urine output Correct Leukocytosis with cloudy urine output

Patients frequently experience diuresis in the hours and days immediately following a kidney transplant. Electrolyte imbalances and signs of infection are unexpected findings that warrant prompt intervention.

erase

Patients frequently experience diuresis in the hours and days immediately following a kidney transplant. Electrolyte imbalances and signs of infection are unexpected findings that warrant prompt intervention.

You are providing care for a patient who has been admitted to the hospital for the treatment of nephrotic syndrome. What is a priority nursing assessment in the care of this patient? A. Assessment of pain and level of consciousness B. Assessment of serum calcium and phosphorus levels C. Blood pressure and assessment for orthostatic hypotension D. Daily weights and measurement of the patient's abdominal girth Rationale

Peripheral edema is characteristic of nephrotic syndrome, and a key nursing responsibility in the care of patients with the disease is close monitoring of abdominal girth, weight, and extremity size. Pain, level of consciousness, and blood pressure are less important in the care these patients. Abnormal calcium and phosphorus levels are not commonly associated with nephrotic syndrome

A Tenckhoff catheter is used with ____ dialysis?

Peritoneal

The elderly patient with hypothyroidism is prescribed levothyroxine (Synthroid). What should you monitor the patient for (select all that apply)? A. Pulse rate greater than 100 beats/minute B. Chest pain C. Constipation D. Positive Trousseau's sign E. Tremors

Rationale The drug increases metabolism and is often withheld because of a pulse greater than 100 beats/minute. The drug can increase myocardial oxygen demand, which can result in angina and cardiac dysrhythmias. Tremors, insomnia, or nervousness indicate the dose is too high for a patient. Constipation and inability to pass flatus are signs of hypothyroidism. A carpal spasm identified while taking the blood pressure indicates latent tetany or hypocalcemia, which is related to the parathyroid gland.

Which laboratory result is most likely for a patient diagnosed with Cushing disease? A. Hypokalemia B. Decreased serum cortisol level C. Eosinophilia D. Thrombocytopenia

Rationale The excessive adrenocortical activity produces hyperglycemia, hypokalemia, hypercalcemia, and elevated plasma cortisol levels. Eosinophils are commonly associated with an allergic response, and levels would decrease rather than increase. Decreased platelet levels are not associated with this disorder.

You are screening a patient for a procedure that requires contrast dye. What should you look for when determining whether or not to use contrast or if contrast is necessary, follow the contrast dye protocol?

Risks for kidney injury: Age over 60, Diabetes, hypertension, nephrotoxic drugs like Metformin, patient with hypoperfusion of the kidneys (can progress to intrarenal AKI) or postrenal (hydronephrosis can progress to AKI)

What treatments are used to treat the clinical manifestations of prerenal oliguria?

Since prolonged ischemia can lead to intrarenal failure restore the circulating volume as quickly as possible using IV fluids. Then-->optimize cardiac performance to maximize cardiac output. Cardiac output is affected by preload, after load and contractility.

A drug for PUD that binds to the surface of the ulcer and coats it is _____ it can interfere with the absorption of many drugs and should be taken ___ minutes before an antacid?

Sucralfate (Carafate) -Protects ulcer from further injury -Should be taken 30 minutes before antacid

RIFLE defines stages of AKI based on changes in A. blood pressure and urine osmolality. B. fractional excretion of urinary sodium. C. estimation of GFR with the MDRD equation. D. serum creatinine and urine output from baseline.

The RIFLE classification is used to describe the stages of AKI and standardize the diagnosis. Risk (R) is the first stage of AKI, followed by injury (I), which is the second stage, and increasing severity to the final or third stage of failure (F). The two outcome variables are loss (L) and end-stage kidney disease (E). The three stages that are determined by the serum creatinine level and urine output are R, I, and F.

Why are magnesium-containing laxatives and antacids contraindicated for patients with renal failure in the oliguric stage?

They are already in hypermegnesemia and cannot excrete the magnesium so Maalox and milk of magnesia are not good for those with impaired renal function

*Why are patients with renal failure who have hypocalcemia usually not symptomatic?*

They are in a state of *metabolic acidosis*, which creates *more ionized calcium* (physiologically active form). Therefore, although their total serum calcium is low their ionized calcium is normal due to acidosis.

In the change of shift report the night nurse tells you that your patient just had a coffee ground emesis. Does this indicate that the patient is experiencing active acute gastrointestinal bleeding? Why?

This is not acute. Acute or sudden GI bleeding results in hematemesis, bright red blood. Bright red blood means active bleeding while coffee ground emesis means enzymes in the stomach have acted on the blood.

Why would you not treat the hyperphosphatemia/hypocalcemia secondary to AKI with a calcium binder like calcium acetate (PhosLo)?

This raises the calcium and causes calcium-phosphate precipitates to form in blood vessels, causing hypertension and stroke. They treat the high phosphate not the low calcium. DO NOT USE *CALCIUM ACETATE (PHOSLO)*

Which is the best explanation of the pathophysiology that occurs in celiac or gluten-sensitive enteropathy? A. Intestinal inflammation, especially in the duodenum B. Full-thickness ulcers throughout the small and large intestines C. Lack of enzyme necessary to digest gluten D. Irregular peristalsis with heightened sensitivity

Tissue destruction occurs as a result of chronic inflammation, activated by gluten. Damage is most severe in the duodenum, probably because it is the site of the highest concentration of gluten. Intermittent, full-thickness ulcers are characteristic of IBD. Lack of the necessary enzyme for digestion of milk is characteristic of lactase deficiency. Irregular peristalsis with enhanced sensitivity is characteristic of irritable bowel syndrome (IBS).

What are the normal ranges for total and ionized calcium?

Total serum calcium: 8.6 - 10.2 mg/dL 􏰀Combination of ionized, complexed, and protein bound 􏰀Ionized serum calcium: 4.6 - 5.2 mg/dL (biologically active form)

Why might someone who is malnourished have a total serum calcium below 8.6 mg/dL?

Total=ionized+complexed+protein bound If not enough protein then the albumin level will be low and that person will have low protein bound, which is part of the total amount 8.6-10.2 TOTAL 4.6-5.2 IONIZED

How might you treat hypocalcemia R/T to kidney disease without using calcium acetate or dietary calcium?

Treat the high phosphate. Phosphate and Calcium are inversely related so using dietary phosphorus restriction and phosphate binders taken with meals (to bind phosphate in the food) will lower the phosphate and thus raise the calcium, balancing them again. D. Sevelamer (Renagel) and Fosrenol (lanthanum carbonate powder) are 2 examples of phosphate binders.

Taking prednisone for too long can cause Cushing's syndrome? A. True B. False

True

Is Melena (tarry stools) a sign of upper or lower GI bleeding?

UPPER, lower GI bleeds are bright red blood

An appropriate technique to use during physical assessment of the thyroid gland is A. asking the patient to hyperextend the neck during palpation. B. percussing the neck for dullness to define the size of the thyroid. C. having the patient swallow water during inspection and palpation of the gland. D. using deep palpation to determine the extent of a visibly enlarged thyroid gland. Rationale

Water should be available for the patient to swallow as part of inspection and palpation of the thyroid gland.

Which of the following urinalysis results would the nurse recognize as an abnormal finding? A. pH 6.0 B. Amber yellow color C. Specific gravity 1.025 D. White blood cells (WBCs) 9/hpf

White blood cells (WBCs) 9/hpf Normal WBC levels in urine are below 5/hpf, with levels exceeding this indicative of inflammation or urinary tract infection. Amber yellow is normal coloration, whereas a pH of 6.0 is average. Reference ranges for specific gravity are 1.003 to 1.030

A patient has a low serum T3 level, and the health care provider orders measurement of the TSH level. If the TSH A level is elevated, this indicates that? a. the cause of the low T3 level is most likely primary hypothyroidism. b. the negative feedback system is failing to stimulate the anterior pituitary gland. c. the patient has an underactive thyroid gland that is not receiving TSH stimulation. d. there is most likely a tumor of the anterior pituitary gland that is causing increased production of TSH.

a. Rationale: Endocrine disorders related to hormone secretion from glands that are stimulated by tropic hormones can be caused by a malsecretion of the tropic hormone or of the target gland. If the problem is in the target gland, it is known as a primary endocrine disorder, and a problem with tropic hormone secretion is known as a secondary endocrine disorder. Serum levels of tropic hormones can illustrate the status of the negative feedback system in relation to target-organ hormone levels. ***If a target organ produces low amounts of hormone, tropic hormones will be increased; if a target organ is overproducing hormones, tropic hormones will be low or undetectable***

Metabolic acidosis occurs in the oliguric phase of AKI as a result of impaired? a. ammonia synthesis. b. excretion of sodium. c. excretion of bicarbonate. d. conservation of potassium.

a. ammonia synthesis. a. Rationale: Metabolic acidosis occurs in acute kidney injury (AKI) because the kidneys cannot synthesize ammonia needed to excrete H+, resulting in an increased acid load. Sodium is lost in urine because the kidneys cannot conserve sodium, and impaired excretion of potassium results in hyperkalemia. Bicarbonate is normally generated and reabsorbed by the functioning kidney to maintain acid-base balance.

The teaching plan for the patient being discharged following an acute episode of upper GI bleeding will include information concerning the importance of? (select all that apply) a. only taking aspirin with milk or bread products. b. avoiding taking aspirin and drugs containing aspirin. c. taking only drugs prescribed by the health care provider. d. taking all drugs 1 hour before mealtime to prevent further bleeding. e. reading all OTC drug labels to avoid those containing stearic acid and calcium.

a. avoiding taking aspirin and drugs containing aspirin. c. taking only drugs prescribed by the health care provider.

When obtaining subjective data from a patient during assessment of the endocrine system, the nurse asks specifically about? a. energy level. b. intake of vitamin C. c. employment history. d. frequency of sexual intercourse.

a. energy level.

In a patient with an elevated serum cortisol, the nurse would expect other laboratory findings to reveal? a. hypokalemia. b. hyponatremia. c. hypoglycemia. d. decreased serum triglycerides.

a. hypokalemia.

To control the side effects of corticosteroid therapy, the nurse teaches the patient who is taking corticosteroids to? a. increase calcium intake to 1500 mg/day. b. perform glucose monitoring for hypoglycemia. c. obtain immunizations due to high risk of infections. d. avoid abrupt position changes because of orthostatic hypotension.

a. increase calcium intake to 1500 mg/day.

A patient is admitted to the hospital with chronic kidney disease. The nurse understands that this condition is characterized by? a. progressive irreversible destruction of the kidneys. b. a rapid decrease in urinary output with an elevated BUN. c. an increasing creatinine clearance with a decrease in urinary output. d. prostration, somnolence, and confusion with coma and imminent death.

a. progressive irreversible destruction of the kidneys.

During the immediate postoperative care of the recipient of a kidney transplant, the nurse expects to a. regulate fluid intake hourly based on urine output. b. find urine-tinged drainage on the abdominal dressing. c. medicate the patient frequently for incisional flank pain. d. remove the urinary catheter to evaluate the ureteral implant.

a. regulate fluid intake hourly based on urine output.

A patient has a serum sodium level of 152 mEq/L (152 mmol/L). The normal hormonal response to this situation is? a. release of ADH. b. release of renin. c. secretion of aldosterone. d. secretion of corticotropin-releasing hormone.

a. release of ADH.

Endocrine disorders often go unrecognized in the older adult because? a. symptoms are often attributed to aging. b. older adults rarely have identifiable symptoms. c. endocrine disorders are relatively rare in the older adult. d. older adults usually have subclinical endocrine disorders that minimize symptoms.

a. symptoms are often attributed to aging.

A patient has a low serum T3 level, and the health care provider orders measurement of the TSH level. If the TSH level is elevated, this indicates that a. the cause of the low T3 level is most likely primary hypothyroidism. b. the negative feedback system is failing to stimulate the anterior pituitary gland. c. the patient has an underactive thyroid gland that is not receiving TSH stimulation. d. there is most likely a tumor of the anterior pituitary gland that is causing increased production of TSH.

a. the cause of the low T3 level is most likely primary hypothyroidism.

Important nursing intervention(s) when caring for a patient with Cushing syndrome include (select all that apply)? a. Restricting protein intake b. Monitoring blood glucose levels c. Observing for signs of hypotension d. Administering medication in equal doses e. Protecting patient from exposure to infection

b. Monitoring blood glucose levels e. Protecting patient from exposure to infection

In contrast to diverticulitis, the patient with diverticulosis? a. Has rectal bleeding b. Often has no symptoms c. Has localized cramping pain d. Frequently develops peritonitis

b. Often has no symptoms

One major advantage of peritoneal dialysis is? a. No medications are required because of the enhanced efficiency of the peritoneal membrane in removing toxins b. The diet is less restricted and dialysis can be performed at home c. The dialysate is biocompatible and causes no long-term consequences d. High glucose concentrations of the dialysate cause a reduction in appetite promoting weight loss

b. The diet is less restricted and dialysis can be performed at home

The nurse evaluates the patency of an AV graft by? a. palpating for pulses distal to the graft site. b. auscultating for the presence of a bruit at the site. c. evaluating the color and temperature of the extremity. d. assessing for the presence of numbness and tingling distal to the site.

b. auscultating for the presence of a bruit at the site.

In replying to a patient's questions about the seriousness of her chronic kidney disease (CKD), the nurse knows that the stage of CKD is based on the? a. total daily urine output. b. glomerular filtration rate. c. serum creatinine and urea levels. d. degree of altered mental status.

b. glomerular filtration rate. b. Rationale: Stages of chronic kidney disease are based on the glomerular filtration rate (GFR) or the presence of kidney damage over a period of 3 months. No specific markers of urinary output, azotemia, or urine output classify the degree of CKD.

An appropriate nursing intervention for the patient with hyperparathyroidism is to a. pad side rails as a seizure precaution. b. increase fluid intake to 3000 to 4000 mL daily. c. maintain bed rest to prevent pathologic fractures. d. monitor the patient for Trousseau's phenomenon and Chvostek's sign

b. increase fluid intake to 3000 to 4000 mL daily.

A patient with a head injury develops SIADH. Symptoms the nurse would expect to find include? a. hypernatremia and edema. b. low urinary output and thirst. c. muscle spasticity and hypertension. d. weight gain and decreased glomerular filtration rate.

b. low urinary output and thirst. Excess ADH increases the permeability of the renal distal tubule and collecting duct, which leads to the reabsorption of water into the circulation. Consequently, extracellular fluid volume expands, plasma osmolality declines, the glomerular filtration rate increases, and sodium levels decline (dilutional hyponatremia). Hyponatremia causes muscle cramping, pain, and weakness. Initially, the patient displays thirst, dyspnea on exertion, and fatigue. The patient with SIADH experiences low urinary output and increased body weight. As the serum sodium level falls (usually to less than 120 mEq/L), manifestations become more severe and include vomiting, abdominal cramps, muscle twitching, and seizures. As plasma osmolality and serum sodium levels continue to decline, cerebral edema may occur, leading to lethargy, anorexia, confusion, headache, seizures, and coma.

In contrast to diverticulitis, the patient with diverticulosis? a. has rectal bleeding. b. often has no symptoms. c. has localized cramping pain. d. frequently develops peritonitis.

b. often has no symptoms.

Priority Decision: A patient on a medical unit has a potassium level of K+ 6.8 mEq/L. The priority action for the nurse would be? a. check the patient's blood pressure (BP). b. place the patient on a cardiac monitor. c. instruct the patient to avoid high potassium foods. d. call the lab and request a redraw of the lab to verify results.

b. place the patient on a cardiac monitor. b. Rationale: Dysrhythmias may occur with an elevated potassium and they are potentially lethal. Monitor the rhythm while contacting the physician or calling the rapid response team. Vital signs should be checked. Depending on the patient history and cause of increased potassium, instruct the patient on diet sources of potassium but this would not help at this point. The nurse may want to recheck the value but until then, he or she should monitor the heart rhythm.

A patient with AKI has a serum potassium level of 6.7 mEq/L (6.7 mmol/L) and the following arterial blood gas results: pH 7.28, PaCO 30 mm Hg, PaO 86 mm Hg, HCO − 18 mEq/L (18 mmol/L). The nurse recognizes that treatment of the acid-base problem with sodium bicarbonate would cause a decrease in the a. pH. b. potassium level. c. bicarbonate level. d. carbon dioxide level.

b. potassium level. b. Rationale: During acidosis, potassium moves out of the cell in exchange for H+ ions, increasing the serum potassium level. Correction of the acidosis with sodium bicarbonate will help lower the potassium levels. A decrease in pH and the bicarbonate and PaCO2 levels would indicate worsening acidosis.

Tubular damage is indicated in the patient with AKI by a urinalysis finding of a. hematuria. b. specific gravity fixed at 1.010. c. urine sodium of 12 mEq/L (12 mmol/L). d. osmolality of 1000 mOsm/kg (1000 mmol/kg

b. specific gravity fixed at 1.010.

Tubular damage is indicated in the patient with AKI by a urinalysis finding of? a. hematuria. b. specific gravity fixed at 1.010. c. urine sodium of 12 mEq/L (12 mmol/L). d. osmolality of 1000 mOsm/kg (1000 mmol/kg).

b. specific gravity fixed at 1.010.

One of the major advantages of peritoneal dialysis is that? a. no medications are required because of the enhanced efficiency of the peritoneal membrane in removing toxins. b. the diet is less restricted and dialysis can be performed at home. c. the dialysate is biocompatible and causes no long-term consequences. d. high glucose concentrations of the dialysate cause a reduction in appetite promoting weight loss.

b. the diet is less restricted and dialysis can be performed at home. *Lewis*

The nurse is involved in health promotion related to oral cancer. Teaching young adults about behaviors that put them at risk for oral cancer includes? a. Discouraging use of chewing gum b. Avoiding use of perfumed lip gloss c. Avoiding use of smokeless tobacco d. Discouraging drinking of carbonated beverages

c. Avoiding use of smokeless tobacco

A nurse is performing an admission assessment of a client who has acute glomerulonephritis. The nurse should expect which of the following findings? a. Low blood pressure b. Polyuria c. Dark-colored urine d. Weight loss

c. Dark-colored urine

If a patient is in the diuretic phase of AKI, the nurse must monitor for which serum electrolyte imbalances? a. Hyperkalemia and hyponatremia b. Hyperkalemia and hypernatremia c. Hypokalemia and hyponatremia d. Hypokalemia and hypernatremia

c. Hypokalemia and hyponatremia

A patient is receiving radiation therapy for cancer of the kidney. The nurse monitors the patient for signs and symptoms of damage to the? a. pancreas. b. thyroid gland. c. adrenal glands. d. posterior pituitary gland.

c. adrenal glands.

An appropriate technique to use during physical assessment of the thyroid gland is? a. asking the patient to hyperextend the neck during palpation. b. percussing the neck for dullness to define the size of the thyroid. c. having the patient swallow water during inspection and palpation of the gland. d. using deep palpation to determine the extent of a visibly enlarged thyroid gland.

c. having the patient swallow water during inspection and palpation of the gland.

The patient with end-stage renal disease tells the nurse that she hates the thought of being tied to the machine but is glad to start dialysis because she will be able to eat and drink what she wants. Based on this information, the nurse identifies the nursing diagnosis of? a. self-esteem disturbance related to dependence on dialysis. b. anxiety related to perceived threat to health status and role functioning. c. ineffective self-health management related to lack of knowledge of treatment plan. d. risk for imbalanced nutrition: more than body requirements related to increased dietary intake.

c. ineffective self-health management related to lack of knowledge of treatment plan.

To assess the patency of a newly placed arteriovenous graft for dialysis, the nurse should? a. irrigate the graft daily with low-dose heparin. b. monitor for any increase in BP in the affected arm. c. listen with a stethoscope over the graft for the presence of a bruit. d. frequently monitor the pulses and neurovascular status distal to the graft.

c. listen with a stethoscope over the graft for the presence of a bruit. *Lewis: 47*

A patient with mild iatrogenic Cushing syndrome is on an alternate-day regimen of corticosteroid therapy. The nurse explains to the patient that this regimen? a. maintains normal adrenal hormone balance. b. prevents ACTH release from the pituitary gland. c. minimizes hypothalamic-pituitary-adrenal suppression. d. provides a more effective therapeutic effect of the drug.

c. minimizes hypothalamic-pituitary-adrenal suppression.

The health care provider prescribes levothyroxine for a patient with hypothyroidism. Following teaching regarding this drug, the nurse determines that further instruction is needed when the patient says? a."I can expect the medication dose may need to be increased." b."I can expect to return to normal function with the use of this drug." c."I only need to take this drug until my symptoms are improved." d."I will report any chest pain or difficulty breathing to the doctor right away."

c."I only need to take this drug until my symptoms are improved."

Causes of primary hypothyroidism in adults include a. malignant or benign thyroid nodules. b. surgical removal or failure of the pituitary gland. c. surgical removal or radiation of the thyroid gland. d. autoimmune-induced atrophy of the thyroid gland

d. Rationale: Both Graves' disease and *Hashimoto's thyroiditis* are autoimmune disorders that eventually destroy the thyroid gland, leading to primary hypothyroidism. Thyroid tumors most often result in hyperthyroidism. Secondary hypothyroidism occurs as a result of pituitary failure, and iatrogenic hypothyroidism results from thyroidectomy or radiation of the thyroid gland.

The dialysate for PD contains? a. electrolytes in an equal concentration to that of the blood. b. calcium in a lower concentration than in the blood. c. sodium in a higher concentration than in the blood. d. dextrose in a higher concentration than in the blood.

d. dextrose in a higher concentration than in the blood.

A patient rapidly progressing toward end-stage renal disease asks about the possibility of a kidney transplant. In responding to the patient, the nurse knows that contraindications to kidney transplantation include? a. hepatitis C infection. b. coronary artery disease. c. refractory hypertension. d. extensive vascular disease.

d. extensive vascular disease. d. Rationale: Extensive vascular disease is a contraindication of renal transplantation, primarily because adequate blood supply is essential to both the health of the new kidney and the circulation of immunosuppressive drugs. Other contraindications include disseminated malignancies, refractory or untreated cardiac disease, chronic respiratory failure, chronic infection, or unresolved psychosocial disorders. CAD may be treated with bypass surgery before transplantation, and transplantation can relieve hypertension. Hepatitis B or C infection is not a contraindication

*A patient with AKI is a candidate for continuous renal replacement therapy (CRRT). The most common indication for use of CRRT is?* a. azotemia. b. pericarditis. c. hyperkalemia. d. fluid overload.

d. fluid overload. d. Rationale: *Continuous renal replacement therapy (CRRT) is indicated for the patient with AKI as an alternative or adjunct to HD to slowly remove solutes and fluid in the hemodynamically unstable patient. It is especially useful for treatment of fluid overload, but HD is indicated for treatment of hyperkalemia, pericarditis, or other serious effects of uremia*

Following thyroid surgery, the nurse suspects damage or removal of the parathyroid glands when the patient develops? a. muscle weakness and weight loss. b. hyperthermia and severe tachycardia. c. hypertension and difficulty swallowing. d. laryngeal stridor and tingling in the hands and feet.

d. laryngeal stridor and tingling in the hands and feet.

After an adrenalectomy for pheochromocytoma, the patient is most likely to experience? a. hypokalemia. b. hyperglycemia. c. marked sodium and water retention. d. marked fluctuations in blood pressure.

d. marked fluctuations in blood pressure.

The nurse is teaching the patient and family about possible causative factors for peptic ulcers. The nurse explains that ulcer formation is? a. caused by a stressful lifestyle and other acid-producing factors such as H. pylori. b. inherited within families and reinforced by bacterial spread of Staphylococcus aureus in childhood. c. promoted by factors that tend to cause oversecretion of acid, such as excess dietary fats, smoking, and H. pylori. d. promoted by a combination of possible factors that may result in erosion of the gastric mucosa, including certain drugs and alcohol.

d. promoted by a combination of possible factors that may result in erosion of the gastric mucosa, including certain drugs and alcohol.

The nurse determines that a patient with oliguria has prerenal oliguria when a. urine testing reveals a low specific gravity. b. the causative factor is malignant hypertension. c. urine testing reveals a high sodium concentration. d. reversal of the oliguria occurs with fluid replacement

d. reversal of the oliguria occurs with fluid replacement d. Rationale: In prerenal oliguria, the oliguria is caused by a decrease in circulating blood volume, and there is no damage yet to the renal tissue. It can be reversed potentially by correcting the precipitating factor, such as fluid replacement for hypovolemia. Prerenal oliguria is characterized by urine with a high specific gravity and a low sodium concentration, whereas oliguria of intrarenal failure is characterized by urine with a low specific gravity and a high sodium concentration. Malignant hypertension causes damage to renal tissue and intrarenal oliguria.

The nurse determines that a patient with oliguria has prerenal oliguria when? a. urine testing reveals a low specific gravity. b. the causative factor is malignant hypertension. c. urine testing reveals a high sodium concentration. d. reversal of the oliguria occurs with fluid replacement

d. reversal of the oliguria occurs with fluid replacement (Why not C-hypoperfusion of the kidneys results in sodium retention due to aldosterone release, resulting in salt and water being retained.

All cells in the body are believed to have intracellular receptors for? a. insulin. b. glucagon. c. growth hormone. d. thyroid hormone.

d. thyroid hormone.

An optimal teaching plan for an outpatient with stomach cancer receiving radiation therapy should include information about? a. cancer support groups, alopecia, and stomatitis. b. avitaminosis, ostomy care, and community resources. c. prosthetic devices, skin conductance, and grief counseling. d. wound and skin care, nutrition, drugs, and community resources.

d. wound and skin care, nutrition, drugs, and community resources.

A patient with graves disease will have ______ TSH levels?

decreased

How is gastritis linked to pernicious anemia?

destruction of the parietal cells

Aldosterone results in ______ retention and ______ secretion?

sodium, potassium

Diabetes Insipidus is a disorder that results from _____ of ADH? A. Overproduction B. Underproduction

underproduction

A nurse is providing discharge teaching for a client who has CKD. Which of the following statements by the client indicates an understanding of the teaching? a. "I will consume foods high in protein" b. "I will decrease my intake of foods high in phosphorus" c. "I will limit my intake of foods high in calcium" d. "I will add salt to the foods I consume"

*"I will decrease my intake of foods high in phosphorus"*

A nurse is providing teaching for a client who has CKD. Which of the following client statements indicates an understanding of the teaching? a. "I will monitor my blood pressure on the same day each week" b. "I will take milk of magnesia when I'm constipated" c. "I will weigh myself each morning" d. "I will use a salt substitute in my diet"

*"I will weigh myself each morning"*

The results of a patient's recent endoscopy indicate the presence of peptic ulcer disease (PUD). Which of the following teaching points should the nurse provide to the patient in light of his new diagnosis? A. "You'll need to drink at least two to three glasses of milk daily." B. "It would likely be beneficial for you to eliminate drinking alcohol." C. "Many people find that a minced or pureed diet eases their symptoms of PUD." D. "Your medications should allow you to maintain your present diet while minimizing symptoms."

*"It would likely be beneficial for you to eliminate drinking alcohol."* Although there is no specific recommended dietary modification for PUD, most patients find it necessary to make some sort of dietary modifications to minimize symptoms. Milk may exacerbate PUD and alcohol is best avoided because it can delay healing

Which statement regarding continuous ambulatory peritoneal dialysis (CAPD) is of highest priority when teaching a patient new to this procedure? A. "It is essential that you maintain aseptic technique to prevent peritonitis." B. "You will be allowed a more liberal protein diet after you complete CAPD." C. "It is important for you to maintain a daily written record of blood pressure and weight." D. "You must continue regular medical and nursing follow-up visits while performing CAPD."

*A. "It is essential that you maintain aseptic technique to prevent peritonitis."* Peritonitis is a potentially fatal complication of peritoneal dialysis, and it is imperative to teach the patient methods to prevent it from occurring. Although the other teaching statements are accurate, they do not address the potential for mortality by peritonitis, making that nursing action the highest priority

Which of the following statements by the nurse regarding continuous ambulatory peritoneal dialysis (CAPD) would be of highest priority when teaching a patient new to this procedure? A. "It is essential that you maintain aseptic technique to prevent peritonitis." B. "You will be allowed a more liberal protein diet once you complete CAPD." C. "It is important for you to maintain a daily written record of blood pressure and weight." D. "You will need to continue regular medical and nursing follow-up visits while performing CAPD."

*A. "It is essential that you maintain aseptic technique to prevent peritonitis."* Peritonitis is a potentially fatal complication of peritoneal dialysis, and thus it is imperative to teach the patient methods of preventing this from occurring. Although the other teaching statements are accurate, they do not have the potential for mortality as does the peritonitis, thus making that nursing action of highest priority.

LEHNE READING: A patient is receiving desmopressin (DDAVP) for the treatment of diabetes insipidus. Which instruction is the priority for a nurse to give the patient? A. "Reduce your water intake to prevent water intoxication." B. "Rotate the nostril you use daily to prevent irritation." C. "Weigh yourself several times each week." D. "You'll quickly see the results of a lower urine amount."

*A. "Reduce your water intake to prevent water intoxication."* Failure to reduce the fluid intake while using desmopressin results in water intoxication, leading to seizures and coma. DDAVP is administered intranasally, therefore rotating the nostril used is important to prevent irritation. Monitoring weekly weights for volume status and understanding that a rapid treatment response occurs also are important; however, they are not as important as reducing the fluid intake to prevent water intoxication.

The nurse is preparing to insert a nasogastric tube into a 68-year-old patient with an abdominal mass and suspected bowel obstruction. The patient asks the nurse why this procedure is necessary. Which of the following responses is most appropriate? A. "The tube will help to drain the stomach contents and prevent further vomiting." B. "The tube will push past the area that is blocked, and thus help to stop the vomiting." C. "The tube is just a standard procedure before many types of surgery to the abdomen." D. "The tube will let us measure your stomach contents, so that we can plan what type of IV fluid replacement would be best."

*A. "The tube will help to drain the stomach contents and prevent further vomiting."* The nasogastric tube is used to decompress the stomach by draining stomach contents, and thereby prevent further vomiting.

Which statement is the most important for a nurse to make to a patient who is taking methimazole, a drug that is used to treat hyperthyroidism? A. "You need to notify your doctor if you have a sore throat and fever." B. "Another medication can be given if you experience any nausea." C. "You may experience some muscle soreness with this medicine." D. "Headache and dizziness may occur but not very frequently."

*A. "You need to notify your doctor if you have a sore throat and fever." * Agranulocytosis (the absence of granulocytes to fight infection) is the most serious toxicity associated with methimazole. Sore throat and fever may be the earliest signs. Nausea, muscle soreness, and headache and dizziness are other adverse effects of methimazole that are not as serious as agranulocytosis. LECTURE: *Methimazole-Pregnancy Category D*-human studies demonstrate a risk. Potential benefits may outweigh the risks.

Which of the following nursing diagnoses is a priority in the care of a patient with renal calculi? A. Acute pain B. Deficient fluid volume C. Risk for constipation D. Risk for powerlessness

*A. Acute pain* Urinary stones are associated with severe abdominal or flank pain. Deficient fluid volume is unlikely to result from urinary stones, whereas constipation is more likely to be an indirect consequence rather than a primary clinical manifestation of the problem. The presence of pain supersedes powerlessness as an immediate focus of nursing care.

A nurse is developing a plan of care for a patient who has Addison's disease and is taking hydrocortisone (Cortef). Which of these outcomes should receive priority in the plan? A. At times of stress, the patient increases the glucocorticoid dose. B. The patient wears a Medic Alert bracelet at all times. C. The patient carries an injectable form and an oral form of glucocorticoid. D. The patient divides the daily dose, taking two-thirds of it in the morning and one-third in the afternoon.

*A. At times of stress, the patient increases the glucocorticoid dose.* Patients with adrenal insufficiency require lifelong replacement doses of glucocorticoids. Failure to increase the dosage at times of stress and illness can be life-threatening. Wearing a Medic Alert bracelet, carrying injectable and oral forms of glucocorticoid, and dividing the daily glucocorticoid dose are important for a patient taking hydrocortisone, but they are not priorities over understanding the need to increase the dose during stress.

What is a classic diagnostic finding in a patient with appendicitis? A. Elevated white blood cell (WBC) count B. Elevated level of lipase C. Left lower quadrant tenderness D. Positive Kernig's sign

*A. Elevated white blood cell (WBC) count* The WBC count is mildly to moderately elevated in about 90% of cases. The lipase level is elevated in patients with pancreatitis. Left lower quadrant tenderness is typically seen in diverticulitis. *The classic location for appendicitis is McBurney's point in the right lower quadrant.* Positive Kernig's sign indicates meningitis.

A patient has developed a paralytic ileus following a recent abdominal surgery. What is the most important nursing consideration when caring for this patient? A. Ensure that the nasogastric tube (SalemSump) is patent. B. Maintain the patient on a clear liquid diet. C. Maintain the patient on strict bed rest. D. Monitor motor strength in bilateral extremities.

*A. Ensure that the nasogastric tube (Salem Sump) is patent.*

After a hypophysectomy, what should be the priority of postoperative nursing care? A. Frequent monitoring of serum and urine osmolarity B. Parenteral administration of a growth hormone receptor antagonist C. Keeping the patient in a recumbent position for 2 days D. Patient teaching relate to lifelong ACTH and TSH hormone replacement Rationale

*A. Frequent monitoring of serum and urine osmolarity* A possible postoperative complication after hypophysectomy is transient diabetes insipidus (DI). This may occur because of loss of antidiuretic hormone (ADH), which is stored in the posterior lobe of the pituitary, or cerebral edema related to manipulation of the pituitary during surgery. To assess for DI, monitor urine output and serum and urine osmolarity closely.

Which is the best food choice for a patient with celiac disease? A. Whole-wheat bread B. Corn mush C. Rye bread D. Barley soup

*B. Corn mush*

The mnemonic for hypocalcemia, which occurs with hyperphosphatemia during the oliguric phase of AKI, is CATS go numb. What does this stand for?

*CATS go numb* Convulsions-Arythmias-Tetany-Seizures (CATS) go numb=numbness and tingling around the lips, mouth and extremities

Which assessment finding is associated with renal calculi? A. Urinary bladder distention B. Hypertension C. Proteinuria D. Flank pain

*D. Flank pain*

Hyperkalemia is frequently associated with? a. hypoglycemia. b. metabolic acidosis. c. respiratory alkalosis. d. decreased urine potassium levels.

*b. metabolic acidosis*

A nurse is providing teaching for a client who has CKD. Which of the following client statements indicates an understanding of the teaching? a. "I will monitor my blood pressure on the same day each week" b. "I will take milk of magnesia when I'm constipated" c. "I will weigh myself each morning" d. "I will use a salt substitute in my diet"

*c. "I will weigh myself each morning"* -Magnesium toxcity is common *PROBABLE EXAM QUESTION*

The pernicious anemia that may accompany gastritis is due to which of the following? a. Chronic autoimmune destruction of cobalamin stores in the body b. Progressive gastric atrophy from chronic breakage in the mucosal barrier and blood loss c. A lack of intrinsic factor normally produced by acid-secreting cells of the gastric mucosa d. Hyperchlorhydria resulting from an increase in acid-secreting parietal cells and degradation of RBCs

*c. A lack of intrinsic factor normally produced by acid-secreting cells of the gastric mucosa*

The nurse teaches the patient that the best time to take corticosteroids for replacement purposes is? a. Once a day at bedtime b. Every other day on awakening c. On arising and in the late afternoon d. At consistent intervals every 6 to 8 hours

*c. On arising and in the late afternoon*

Which of the following findings places a client at risk for seizures? a. Hypokalemia b. Rapid increase of catecholamines c. Rapid decrease in fluid d. Hypercalcemia

*c. Rapid decrease in fluid* *WHY NOT D?* Hypocalcemia NOT hypercalcemia places a person at risk for seizures

A nurse is assessing a client who has adrenal insufficiency. Which of the following findings should the nurse expect? a. Moon face b. Weight gain c. Serum calcium 12.8 mg/dL d. Serum sodium 150 mEq/L

*c. Serum calcium 12.8 mg/dL*

Which of the following lab findings of a client with AKI should be reported to the provider? a. Serum potassium 5.0 mEq/L b. Serum calcium 9.0 mg/dL c. Serum creatinine 4.0 mg/dL d. Serum amylast 84 IU/L

*c. Serum creatinine 4.0 mg/dL* normal is .07-1.2

A patient with diabetes insipidus is treated with nasal desmopressin (DDAVP). The nurse determines that the drug is not having an adequate therapeutic effect when the patient experiences? a. headache and weight gain. b. nasal irritation and nausea. c. a urine specific gravity of 1.002. d. an oral intake greater than urinary output.

*c. a urine specific gravity of 1.002.* Rationale: Normal urine specific gravity is 1.003 to 1.030, and urine with a specific gravity of 1.002 is very dilute, indicating that there continues to be excessive loss of water and that treatment of diabetes insipidus is inadequate. Headache, weight gain, and oral intake greater than urinary output are signs of volume excess that occur with overmedication. Nasal irritation and nausea may also indicate overdosage.

Short bowel syndrome is most likely to occur in the patient with? a. ulcerative colitis. b. irritable bowel syndrome. c. an extensive resection of the ileum. d. a colectomy performed for cancer of the bowel.

*c. an extensive resection of the ileum.*

Diminished ability to concentrate urine, associated with aging of the urinary system, is attributed to? a. a decrease in bladder sensory receptors. b. a decrease in the number of functioning nephrons. c. decreased function of the loop of Henle and tubules. d. thickening of the basement membrane of Bowman's capsule.

*c. decreased function of the loop of Henle and tubules.*

The nurse determines that a patient with AKI is in the recovery phase when the patient experiences a. a return to normal weight. b. a urine output of 3700 mL/day. c. decreasing blood urea nitrogen (BUN) and creatinine levels. d. decreasing sodium and potassium levels.

*c. decreasing blood urea nitrogen (BUN) and creatinine levels.*

The nurse teaches a patient with newly diagnosed peptic ulcer disease to? a. maintain a bland, soft, low-residue diet. b. use alcohol and caffeine in moderation and always with food. c. eat as normally as possible, eliminating foods that cause pain or discomfort. d. avoid milk and milk products because they stimulate gastric acid production.

*c. eat as normally as possible, eliminating foods that cause pain or discomfort.* LECTURE: diets were thought to cause at one point, now just eat what doesn't cause pain

A patient with chronic kidney disease has hyperphosphatemia. A commonly associated electrolyte imbalance is a. hypokalemia. b. hyponatremia. c. hypocalcemia. d. hypomagnesemia.

*c. hypocalcemia*

The nurse teaches the patient with diverticulosis to? a. use anticholinergic drugs routinely to prevent bowel spasm. b. have an annual colonoscopy to detect malignant changes in the lesions. c. maintain a high-fiber diet and use bulk laxatives to increase fecal volume. d. exclude whole grain breads and cereals from the diet to prevent irritating the bowel.

*c. maintain a high-fiber diet and use bulk laxatives to increase fecal volume.* lecture

In planning care for the patient with Crohn's disease, the nurse recognizes that a major difference between ulcerative colitis and Crohn's disease is that Crohn's disease? a. frequently results in toxic megacolon. b. causes fewer nutritional deficiencies than does ulcerative colitis. c. often recurs after surgery, whereas ulcerative colitis is curable with a colectomy. d. is manifested by rectal bleeding and anemia more frequently than is ulcerative colitis.

*c. often recurs after surgery, whereas ulcerative colitis is curable with a colectomy.* likely to show up

A potential adverse effect of palpation of an enlarged thyroid gland is a. carotid artery obstruction. b. damage to the cricoid cartilage. c. release of excessive thyroid hormone into circulation. d. hoarseness from pressure on the recurrent laryngeal nerve.

*c. release of excessive thyroid hormone into circulation.*

Using RIFLE, the three stages of acute kidney injury are defined based on changes in? a. Blood pressure and urine osmolality b. Fractional excretion of urinary sodium c. Estimation of GFR with the MDRD equation d. Serum creatinine or urine output from baseline

*d. Serum creatinine or urine output from baseline*

A nursing intervention that is most appropriate to decrease postoperative edema and pain following an inguinal herniorrhaphy is? a. applying a truss to the hernia site. b. allowing the patient to stand to void. c. supporting the incision during coughing. d. applying a scrotal support with ice bag.

*d. applying a scrotal support with ice bag.*

In a patient with a positive Chvostek's sign, the nurse would anticipate the IV administration of? a. calcitonin. b. vitamin D. c. loop diuretics. d. calcium gluconate.

*d. calcium gluconate.*

The nurse identifies a risk for urinary calculi in a patient who relates a past health history that includes? a. adrenal insufficiency. b. serotonin deficiency. c. hyperaldosteronism. d. hyperparathyroidism.

*d. hyperparathyroidism.* Excessive levels of circulating parathyroid hormone (PTH) usually lead to hypercalcemia and hypophosphatemia. In the kidneys excess calcium in the filtrate cannot be reabsorbed, leading to increased levels of calcium in the urine (hypercalciuria). This increased urinary calcium level along with a large amount of urinary phosphate can lead to calculi formation

*That hormones of one gland influence the function of hormones of another gland is demonstrated by the fact that?* a. increased insulin levels inhibit the secretion of glucagon. b. increased testosterone levels inhibit the release of estrogen. c. increased cortisol levels stimulate the secretion of insulin. d. increased atrial natriuretic peptide (ANP) levels inhibit the secretion of aldosterone.

*d. increased atrial natriuretic peptide (ANP) levels inhibit the secretion of aldosterone.*

You should take antacids ____ hour (s) before or ___ hour (s) after you take any other medications?

1, 1 (interfere with absorption of many other medications)

The elderly patient with hypothyroidism is prescribed levothyroxine (Synthroid). What should you monitor the patient for (select all that apply)? A. Pulse rate greater than 100 beats/minute B. Chest pain C. Constipation D. Positive Trousseau's sign E. Tremors

A, B, E The drug increases metabolism and is often withheld because of a pulse greater than 100 beats/minute. The drug can increase myocardial oxygen demand, which can result in angina and cardiac dysrhythmias. Tremors, insomnia, or nervousness indicate the dose is too high for a patient. Constipation and inability to pass flatus are signs of hypothyroidism. A carpal spasm identified while taking the blood pressure indicates latent tetany or hypocalcemia, which is related to the parathyroid gland.

What does the nurse identify as a possible adverse effect of long-term glucocorticoid therapy? (Select all that apply.) A. Adrenal insufficiency B. Osteoporosis C. Hypoglycemia D. Hyperkalemia E. Cataracts

A, B, E Adverse effects of long-term glucocorticoid therapy include adrenal insufficiency, osteoporosis, hyperglycemia, hypokalemia, and cataracts

What does the nurse identify as a possible adverse effect of long-term glucocorticoid therapy? (Select all that apply.) A. Adrenal insufficiency B. Osteoporosis C. Hypoglycemia D. Hyperkalemia E. Cataracts

A, B, E *Adverse effects of long-term glucocorticoid therapy include adrenal insufficiency, osteoporosis, hyperglycemia, hypokalemia and cataracts* C-hyperglycemia D-hypokalemia

An upper GI bleed originating in the stomach is often drug-induced peptic ulcer disease caused by?

Aspirin, NSAID, corticosteroids

erase

C. Suspend treatment immediately. The ultrafiltrate should be clear yellow, and specimens may be obtained for evaluation of serum chemistries. If the ultrafiltrate becomes bloody or blood tinged, a possible rupture in the filter membrane should be suspected, and treatment is suspended immediately to prevent blood loss and infection.

KIDNEY LECTURE

KIDNEYS LECTURE

With duodenal ulcers pain is relieved by meals? T/F

TRUE

Why do glucocorticoid medications need to be taken in the morning before 9 am?

This mimics the natural circadian rhythms that trigger bursts of cortisol in the morning

The nurse evaluates the patency of an AV graft by? a. palpating for pulses distal to the graft site. b. auscultating for the presence of a bruit at the site. c. evaluating the color and temperature of the extremity. d. assessing for the presence of numbness and tingling distal to the site

b. auscultating for the presence of a bruit at the site.

To prevent the most common serious complication of PD, it is important for the nurse to? a. infuse the dialysate slowly. b. use strict aseptic technique in the dialysis procedures. c. have the patient empty the bowel before the inflow phase. d. reposition the patient frequently and promote deep breathing.

b. use strict aseptic technique in the dialysis procedures.

A patient with renal disease has oliguria and a creatinine clearance of 40 mL/min. These findings most directly reflect abnormal function of? a.tubular secretion. b.glomerular filtration. c.capillary permeability. d.concentration of filtrate.

b.glomerular filtration. The amount of blood filtered each minute by the glomeruli is expressed as the glomerular filtration rate (GFR). The normal GFR is about 125 mL/minute

A characteristic common to all hormones is that they a.circulate in the blood bound to plasma proteins. b.influence cellular activity of specific target tissues. c.accelerate the metabolic processes of all body cells. d.enter a cell to alter the cell's metabolism or gene expression.

b.influence cellular activity of specific target tissues.

Sucralfate (Carafate) should be taken _____ meals and antacids _____ meals?

before, after

Causes of primary hypothyroidism in adults include a. malignant or benign thyroid nodules. b. surgical removal or failure of the pituitary gland. c. surgical removal or radiation of the thyroid gland. d. autoimmune-induced atrophy of the thyroid gland.

d. autoimmune-induced atrophy of the thyroid gland. *Hashimoto's thyroiditis=autoimmune primary hypothyroidism*

What is used to suppress the immune and inflammatory response?

high doses of glucocorticoids

What is the main trigger for ADH release?

high serum osmolality (normal range is 275-295 mOSM/kg)-300 or more is considered "high"

Whats used to treat Addison's disease?

low doses of glucocorticoids

Patients in renal failure have ___ total calcium and ____ ionized calcium?

low, high Acidosis=more ionized calcium total normal is 8.6-10.2 normal ionized is 4.6-5.2

Uremia encephalopathy occurs during renal failure and is the result of ____ ?

metabolic waste products affecting the brain

The oliguric phase of acute kidney injury is characterized by hyperphosphatemia and hypocalcemia. These 2 things are inversely related so treating one balances the other out. Part of the treatment for this is a LOW phosphate diet not a HIGH calcium diet because adding calcium creates atherosclerotic plaques. What are some foods that are high in phosphate that should be AVOIDED?

•Cheese and milk •Pudding and ice cream •Beer and cola •Oysters and sardines •Lentils and kidney beans •Soy beans •bran cereal •Nuts and seeds •Whole grain products


Ensembles d'études connexes

Unit 2 Physical Properties of Water-QUESTIONS

View Set

Functional Assessment and Behavior Intervention Plans

View Set

OB CH 8 Intrapartum Assessment and Interventions

View Set

MH Exam 3: Chapters 20, 21, & 25

View Set

Autosomal Reciprocal Translocations

View Set

Period 6: 1/17-1/21, Industrialization in the Gilded Age

View Set

Penny Chapter 30 - Chromosomal Abnormalities

View Set

Insurance information and privacy protection act

View Set

ASAP PACE Certificate (2022 updated)

View Set